Você está na página 1de 148

=: ltem_1of32

? 61 � � , � I)
Question Id: 3398
:.
- ?
Mark <J
Previous Next
[:> Tutorial Lab Values Notes Calculator Reverse Color Text Zoom

4 A 29-year-old woman comes to the emergency department after spilling hot coffee on her left forearm.
5
Medications include a daily oral contraceptive, and the patient has no other medical issues. Evaluation shows a
6
full-thickness burn, and she is discharged with analgesics, topical antibiotics, and wound care instructions. Three
7

8
days later, the patient returns due to worsening pain and swelling of the left hand. She describes the pain in her
9 hand as severe and aching. Repeat examination shows previous burn injury healing with a circumferential eschar
10 formation. Her left hand is tense and tender. Which of the following is the most likely cause of this patient's
11
cond ition?
12
13
Q A. Clostridial myonecrosis
14
Q B. Inflammatory arthritis

17
Q C. Subcutaneous bacterial invasion
18 Q D. Vascular compromise
19
20 Q E. Venous thrombosis
21
22
23 Submit
24
25
26

27

Block Time Remaining: 00:47:20


28
29 TIMEDTUTOR

01:52 �
T• l/vl l/1 •
=: ltem 1of32
_ :. ? 61 � � , � I)
Question Id: 3398 - ?
Mark <J
Previous Next
[:> Tutorial Lab Values Notes Calculator Reverse Color Text Zoom

4 A 29-year-old woman comes to the emergency department after spilling hot coffee on her left forearm.
5
Medications include a daily oral contraceptive, and the patient has no other medical issues. Evaluation shows a
6
full-thickness burn, and she is discharged with analgesics, topical antibiotics, and wound care instructions. Three
7

8
days later, the patient returns due to worsening pain and swelling of the left hand. She describes the pain in her
9 hand as severe and aching. Repeat examination shows previous burn injury healing with a circumferential eschar
10 formation. Her left hand is tense and tender. Which of the following is the most likely cause of this patient's
11
cond ition?
12
13
X@ A. Clostridial myonecrosis [8%]
14
B. Inflammatory arthritis [0%)

17
C. Subcutaneous bacterial invasion [12%]
18
D. Vascular compromise [73%>]
19

I
20 E. Venous thrombosis [4%]
21
22
23
lncorrft
l1o. 73% II\ 48 Seconds i==l 08/19/2018
orr�t answer
24
� L!!!. Answere<I correctly TimeSpent
'-:::.J l.!!!.I Last Update<!
25
26

27 •
Block Time Remaining: 00:47:12
28
29 TIMEDTUTOR

01:52 �
T• l/vl l/1 •
1M
2

=:
ltem 1of32
_
Question Id: 3398
:.
- ? Mark <J
Previous Next
[:>
?
Tutorial
61
Lab Values

Notes

Calculator
,
Reverse Color

Text Zoom
I)
3 •
4 Clinical features of compartment syndrome
5
• Pain out of proportion to injury
6
7 • Pain ion passive stretch
Common
8 • Rapidly increasing & tense swelling
9 • Paresthesia (early)
10
11 • ! Sensation
12 • Motor weakness (within hours)
Uncommon
13 • Paralysis (late)
14
• ! Distal pulses (uncommon)
6
� Acute compartment syndrome (ACS) occurs when excessive fluid accumulation in a confined compartment of
17
the body (eg, upper leg, lower leg, upper arm, forearm, hand, foot, abdomen) causes an increase in compartment
18
19
pressure to the point that blood flow is severely impaired. Pain out of proportion to clinical findings is often the
20 first presenting symptom, and worsening pain may be elicited with passive stretch of the affected muscle
21 compartment. Tissue tension, pallor, and decreased sensation may also be present. If elevated compartment
22 pressure is allowed to persist, tissue ischemia and eventual tissue death will occur. The eschar that results from
23
a circumferential, full-thickness (third degree) burn often leads to constriction of venous and lymphatic
24
25
drainage, fluid accumulation, and resulting distal ACS.

.... _ ..,-. . -
26 Overall, clinical findings for ACS can be unreliable, and a high clinical suspicion should be maintained in patients
27
• I III1111::J._.. • I- - - II eI • I e
Block Time Remaining: 00:47:12
28
29 TIMEDTUTOR

01:53 �
T• l/vl l/1 •
=: ltem 1of32
_ ? 61 � � , � I)
Question Id: 3398
:.
- ?
Mark <J
Previous Next
[:> Tutorial Lab Values Notes Calculator Reverse Color Text Zoom

Overall, clinical findings for ACS can be unreliable, and a high clinical suspicion should be maintained in patients
4
with significant injuries to the extremities, especially the distal extremities. Serial compartment pressure monitoring
5
6
is useful in determining the need for definitive operative management, which consists of compartment
7 decompression by fasciotomy or, in the case of circumferential burns, escharotomy.
8
(Choices A and C) Subcutaneous bacterial invasion may lead to cellulitis or, in the case of Clostridium
9
10
perfringens, gas gangrene with myonecrosis. However, cellulitis is typically characterized by skin warmth and gas
11 gangrene is characterized by crepitus.
12
(Choice B) Inflammatory arthritis (eg, rheumatoid arthriti s) is characterized by pain and swelling often involving
13
14 the bilateral metacarpal phalangeal joints. Unilateral swelling of the left hand following a forearm burn is not
consistent with inflammatory arthritis.
� (Choice E) Venous thrombosis of the upper extremity can lead to pain and swelling of the hand, and burn trauma
17
18
may increase the ri sk of venous thrombosis development. However, the combination of swelling, tissue tension,
19 and severe pain in the setting of a circumferential, full-thickness burn with an eschar is more consistent with ACS.
20
Educational objective:
21
22 Circumferential, full-thickness (third degree) burns can result in eschar formation that restricts venous and
23 lymphatic drainage, leading to acute compartment syndrome.
24
25
References
26
27 •
28
Block Time Remaining: 00:47:12
29 TIMEDTUTOR

01:53 �
T• l/vl l/1 •
1M
1 •
=: ltem_2of32
? 61 � � , � I)
Question Id: 12417
:.
- ?Mark <J
Previous Next
[:> Tutorial Lab Values Notes Calculator Reverse Color Text Zoom

� A 15-month-old boy is brought to the emergency department with cyanosis. Over the past week, the patient has
4
5
been teething and has had mild nasal congestion and low-grade fevers. He has been eating and drinking
6 normally. The patient was extremely fussy today, despite his mother giving him acetaminophen and a topical
7 anesthetic that she used when she had a toothache. He was hospitalized for bronchiolitis 6 months ago but is
8 otherwise healthy, and immunizations are up to date. Blood pressure is 90/60 mm Hg, pulse is 158/min, and
9
respirations are 50/min. Pulse oximetry is 85% on room air . Physical examination shows a sleepy child with blue
10
11
discoloration of the skin and nails. The lungs are clear to auscultation bilaterally. Cardiac examination reveals no
12 murmurs or gallops. The abdomen is soft, nontender, and has no hepatosplenomegaly. Blood samples are
13 obtained for laboratory testing, and 100% oxygen is administered but fails to increase the pulse oximetry reading or
14 the cyanosis.

Item 1 of 2

6
17
Which of the following laboratory findings would most likely be seen in this patient?
18
19
20 QA. Elevated PaCO2
21 0 8. Elevated plasma osmolal gap
22
23 0 C. Elevated transaminase levels
24
25
0 D. Normal co-oximetry results
26 0 E. Normal PaO2
27

Block Time Remaining: 00:47:08


28
29 TIMEDTUTOR

01:53 �
T• l/vl l/1 •
1M
1 •
=: ltem_2of32
? 61 � � , � I)
Question Id: 12417
:.
- ?Mark <J Previous Next
[:> Tutorial Lab Values Notes Calculator Reverse Color Text Zoom


4
anesthetic that she used when she had a toothache. He was hospitalized for bronchiolitis 6 months ago but is
5 othervvise healthy, and immunizations are up to date. Blood pressure is 90/60 mm Hg, pulse is 158/min, and
6 respirations are 50/min. Pulse oximetry is 85%> on room air. Physical examination shows a sleepy child with blue
7 discoloration of the skin and nails. The lungs are clear to auscultation bilaterally. Cardiac examination reveals no
8
murmurs or gallops. The abdomen is soft, nontender, and has no hepatosplenomegaly. Blood samples are
9
10
obtained for laboratory testing, and 100% oxygen is administered but fails to increase the pulse oximetry reading or
11 the cyanosis.
12
Item 1 of 2
13
14 Which of the following laboratory findings would most likely be seen in this patient?


6 X@ A. Elevated PaCO2 [32%>]
17
18 B. Elevated plasma osmolal gap [15%]
19
C. Elevated transaminase levels [7%]
20
21 D. Normal co-oximetry results (10%]
22
23 E. Normal PaO2 [34%>]
24
25
26
Incorrect
(1,, 34%
L!!!. Answered correctl
27
II • •I
Block Time Remaining: 00:47:oo
28
29 TIMEDTUTOR

EN • @ ,j > .,,II
01:53 �
i"• T• l/vl l/1 •
1M
1 •
=: ltem 2of32
_ :. ? 61 � � , � I)
Question Id: 12417 - ? Mark <J
Previous Next
[:> Tutorial Lab Values Notes Calculator Reverse Color Text Zoom


4
Clinical presentation of methemoglobinemia
5 Exposure to oxidizing substances (eg, dapsone, nitrites,
6 History
local/topical anesthetic)
7
8 • Cyanosis
9 Clinical
• Pulse oximetry saturation ~85%>
10 examination
• Dark chocolate-colored blood
11
12
• Saturation gap (>5% difference between oxygen saturation on pulse oximetry &
13 Laboratory
ABG)
14 findings
• Normal PaO2


6
ABG = arterial blood gas
17
18 This patient has methemoglobinemia, an uncommon complication that can occur after excessive exposure to an
19
oxidizing agent (eg, dapsone, nitrites, loca l/topical anesthetics) and results in cyanosis. Additional clinical
20
features may include lethargy, respiratory depression, se izures, and death. In normal hemoglobin, iron is in the
21
22 ferrous (Fe2•) state. When exposed to an oxidizing agent, at least one of the four iron molecules is oxidized to the
23 ferric (Fe3•) state, resulting in acute methemoglobinemia. Ferric iron is unable to bind oxygen; in addition, the
24
ferric state changes the hemoglobin structure and causes ferrous sites to have an increased affinity for oxygen (ie,
25
"left shift" on oxygen-dissociation curve). The increased oxygen affinity prevents oxygen release in peripheral


26
27 tissues (ie, decreased oxygen delivery). •
28
Block Time Remaining: 00:47:oo
TIMEDTUTOR � Feedback
©
Suspend End Block

..-••.
29

r§1 EN . 00 ,j, .,,II I"•


01:53 �
� T• l/vl l/1 •
1M
1 •
=: ltem 2of32
_ :. ? 61 � � , � I)
Question Id: 12417 - ?
Mark <J
Previous Next
[:> Tutorial Lab Values Notes Calculator Reverse Color Text Zoom

� Because methemoglobin absorbs light at a different wavelength than hemoglobin, patients have pulse oximetry
4
5 readings of -85% regardless of the true oxygen saturation level (as seen in this patient). Supplemental oxygen
6 has no effect on the altered methemoglobin structure; therefore, it does not improve cyanosis, blood color, or pulse
7 oximetry readings. In contrast, arterial blood gas testing analyzes on ly unbound arterial oxygen (as opposed to
8
hemoglobin-bound oxygen) and displays a falsely elevated oxygen saturation level, shown as normal Pa02.
9
10 (Choice A) PaC02 increases in respiratory failure such as in severe bronchiolitis or asthma. In a tachypneic
11
individual with healthy lungs, as in this case, PaC02 will be low due to hyperventilation from tachypnea.
12
13 (Choice B) An elevated plasma osmolal gap is seen after ethanol, methanol, or ethylene glycol ingestion.
14 Patients with this condition may have signs of alcohol intoxication but are not cyanotic.

(Choice C) Elevated transaminase levels signify liver inflammation and can be due to an overdose of

6
17 acetaminophen, but these patients have abdominal pain, nausea, and vomiting, not cyanosis.
18
19
(Choice D) Co-oximetry testing analyzes hemoglobin absorption wavelengths and can identify hemoglobin,
20 methemoglobin, and carboxyhemoglobin. Elevated levels of methemoglobin would confirm the diagnosis in this
21 patient, and therefore co-oximetry testing would not be normal.
22
23 Educational objective:
24 Methemoglobinemia presents with cyanosis after exposure to an oxidizing agent (eg, dapsone, nitrites,
25 anesthetics). Pulse oximetry saturation is typical ly low and does not improve with the administration of oxygen;


26
however, Pa02 is normal.
27 •
28
29
Block Time Remaining: 00:47:oo
TIMEDTUTOR � Feedback
©
Suspend End Block

.... r§1
...- �
EN . 00: ,j, .,,ii 1•• 01:53 �
T• l/vl l/1 •
1M

-

1
= Item 3of32
- Question Id: 12418
_
� ?
Mark <J
Previous
C>
Next
?
Tutorial
61
Lab Values

N�tes

Calculator
,
Reverse Color
� 0
Text z oom

4 Item 2 of 2
5
6 Which of the following should be administered to this patient?
7
8 0 A. Dimercaprol
10
9
0 B. Fomepizole
11 0 C. Glucagon
12
13 0 D. Methylene blue
14
0 E. N-acetylcysteine

6 0 F. Pralidoxime
17
18
19 Submit
20
21
22
23
24
25


26
27
28
Block Time Remaining: 00:46:58
TIMEDTUTOR � Feedback
©
Suspend End Block

..-....
29

r§1 �
EN � 00 ,j, .,,ii 1•• 01:54 �
T• l/vl l/1 •
-
1M
1 •
=: ltem_3of32
? 61 � � , � I)
Question Id: 12418
:.
- ?Mark <J Previous Next
[:> Tutorial Lab Values Notes Calculator Reverse Color Text Zoom

4 Item 2 of 2
5
6 Which of the following should be administered to this patient?
7
8 X@ A. Dimercaprol [6%>]
9
B. Fomepizole [2°/o]
10
11 C. Glucagon [1%]
12
13 D. Methylene blue [75%]
14
E. N-acetylcysteine [11 %]

F. Pralidoxime [2%]

I

6
17
18
19
Incorrect
(1,, 75% (T\
6 Seconds i::==I 09/27/2018
Correct answer
20
L!!!. Answered correctly "-.::.; TimeSpent I.!!!.! Last Updated
21 D
22
23
24 Explanation
25
26 This patient has methemoglobinemia, a medical emergency. Any medication (eg, topical anesthetic) suspected
27
I - II- I -II• • • • I -II ... - •1•111ac,..,1t1 II - • .. - • • I I ... • I
Block Time Remaining: 00:46:54
28
29 TIMEDTUTOR

EN • @ ,j > .,,II I"•


01:54 �
T• l/vl l/1 •
1M

-
1 •
=: ltem 3of32
_ ? 61 � � , � I)
Question Id: 12418
:.
- ?
Mark <J
Previous Next
[:> Tutorial Lab Values Notes Calculator Reverse Color Text Zoom

This patient has methemoglobinemia, a medical emergency. Any medication (eg, topical anesthetic) suspected
4
to be the cause of the methemoglobinemia requires immediate discontinuation.
5
6 The antidote for acquired symptomatic methemoglobinemia or high levels of methemoglobin (as measured by co­
7 oximetry) is methylene blue. Methylene blue acts as an electron acceptor for NADPH and is reduced to
8
leucomethylene blue, which in turn reduces methemoglobin to hemoglobin. High-dose ascorbic acid (vitamin
9
10 C) acts as a reducing agent and can be used when methylene blue is unavailable or contraindicated (eg, glucose-
11 6-phosphate deficiency).
12
(Choice A) Dimercaprol, a chelating agent, is most commonly used in the treatment of lead poisoning to increase
13
14 the urinary and fecal excretion of lead.

(Choice B) Fomepizole is used for the treatment of ethylene glycol or methanol ingestion. Through the inhibiti on

6
of alcohol dehydrogenase, fomepizole prevents the metabolism of these alcohols to their toxic metabolites.
17
18 (Choice C) Glucagon can be used after the ingestion of a beta blocker or calcium channel blocker by activating
19
adenylate cyclase, which increases intracellular calcium and improves cardiac contractility.
20
21 (Choice E) N-acetylcysteine is the antidote for acetaminophen poisoning. It acts by restoring glutathione, which
22 metabolizes acetaminophen's toxic metabolite, N-acetyl-p-benzoquinone imine (NAPQI), to nontoxic metabolites.
23
24 (Choice F) Pralidoxime is an antidote for cholinergic toxicity (eg, acetylcholinesterase inhibitors such as
25 organophosphates). Pralidoxime is given with atropine and reactivates the cholinesterase enzyme.


26
27
Educational objective:
28
Block Time Remaining: 00:46:54
TIMEDTUTOR � Feedback
©
Suspend End Block

..-.
29

r§1 EN . 00: ,j, .,,ii 1•• 01:54 �


� T• l/vl l/1 •
1M

-
1 •
=: ltem_3of32
? 61 � � , � I)
Question Id: 12418
:.
- ?Mark <J
Previous Next
[:> Tutorial Lab Values Notes Calculator Reverse Color Text Zoom

4 6-phosphate deficiency).
5
(Choice A) Dimercaprol, a chelating agent, is most commonly used in the treatment of lead poisoning to increase
6
the urinary and fecal excretion of lead.
7
8 (Choice B) Fomepizole is used for the treatment of ethylene glycol or methanol ingestion. Through the inhibiti on
9
of alcohol dehydrogenase, fomepizole prevents the metabolism of these alcohols to their toxic metabolites.
10
11 (Choice C) Glucagon can be used after the ingestion of a beta blocker or calcium channel blocker by activating
12 adenylate cyclase, which increases intracellular calcium and improves cardiac contractility.
13
14 (Choice E) N-acetylcysteine is the antidote for acetaminophen poisoning. It acts by restoring glutathione, which
metabolizes acetaminophen's toxic metabolite, N-acetyl-p-benzoquinone imine (NAPQI), to nontoxic metabolites.

6
17
(Choice F) Pralidoxime is an antidote for cholinergic toxicity (eg, acetylcholinesterase inhibitors such as
18 organophosphates). Pralidoxime is given with atropine and reactivates the cholinesterase enzyme.
19
Educational objective:
20
21
Patients with acquired symptomatic methemoglobinemia or high levels of methemoglobin can be treated with
22 methylene blue or high-dose ascorbic acid.
23
24
References
25
26 • A 12-day-old boy with methemoglobinemia after circumcision with local anesthesia {Lidocaine/Prilocaine).
27

Block Time Remaining: 00:46:54


28
29 TIMEDTUTOR

EN • @: ,j > .,,11 1•• 01:54 �


T• l/vl l/1 •
1M
1 �
=: ltem4of32 ? 61 � � , � I)
Question Id: 2655 •
- ? Mark <J
Previous Next Tutorial Lab Values Notes Calculator Reverse Color Text Zoom
2
3 .. . . gp g y . [:>
g y . p .
several episodes of vomiting and abdominal pain for the past two hours. His vomit is coffee ground in appearance.
5
He is irritable and lethargic. His blood pressure is 80/50 mm Hg and pulse rate is 120/min. Examination shows a
6
7
normal oropharynx; chest auscultation is within normal limits. Abdomen is soft and mildly tender at the
8 epigastrium; there is no hepatosplenomegaly. Extremities are cold to touch. Initial laboratory studies show:
9 Hemoglobin 10.3 g/L
10 Leukocyte count 14,500/mm3
11
Bicarbonate 1 8 mEq/L
12
13
Chest x-ray is within normal limits. Abdominal imaging shows radioopaque tablets in the stomach. Intravenous
14 normal saline is started. Which of the following is the most appropriate next step in management?

6 Q A. Sodium bicarbonate

17
18
0 B. Deferoxamine

19 0 C. Magnesium sulfate
20
21 0 D. Hemodialysis
22
23
0 E. Calcium EDTA

24 0 F. Calcium gluconate
25
26 0 G. Oral succimer
27
• ■ Ill -,-..·- n·,,..,..,,..,,,.-,,1 -
28
Block Time Remaining: 00:46:44
29 TIMEDTUTOR

EN � @: ,j > .,,11 1•• 01:54 �


T• l/vl l/1 •
1M
1 •
=: ltem4of32 ? 61 � � , � I)
2 Question Id: 2655 •
- ?
Mark <J
Previous Next
[:> Tutorial Lab Values Notes Calculator Reverse Color Text Zoom
3 •
epigastrium; there is no hepatosplenomegaly. Extremities are cold to touch. Initial laboratory studies show:
5 Hemoglobin 10.3 g/L
6
Leukocyte count 14,500/mm3
Bicarbonate 1 8 mEq/L
7
8
Chest x-ray is within normal limits. Abdominal imaging shows radioopaque tablets in the stomach. Intravenous
9
normal saline is started. Which of the following is the most appropriate next step in management?
10
11
12
X @:' A. Sodium bicarbonate [8%]
13 � B. Deferoxamine [64%]
14
C. Magnesium sulfate [0%]
6
� D. Hemodialysis [1 %)
17
18 E. Calcium EDTA [6%]
19
20 F. Calcium gluconate (1%]
21
G. Oral succimer [7%]
22
23 H. N-acetylcysteine [8%]
24
25
26
27 I•I• I:
Block Time Remaining: 00:46:39
28
29 TIMEDTUTOR

EN • @: ,j > .,,11 1•• 01:54 �


T• l/vl l/1 •
1M
1 •
=: ltem4of32 ? 61 � � , � I)
2 Question Id: 2655 •
- ? Mark <J
Previous Next
[:> Tutorial Lab Values Notes Calculator Reverse Color Text Zoom
3 •
This 3-year-old boy is suffering from acute iron poisoning. Pre-natal vitamins, which are rich in iron appear as
5 radiopaque tablets seen on abdominal x-ray. The mechanism of iron poisoning is free radical production and lipid
6 peroxidation, which impairs various cell processes, leading to systemic manifestations. These include abdominal
7 pain and hematemesis, hypovolemic shock, and metabolic acidosis. This boy's low bicarbonate level, hypotension,
8
and cool extremities indicate that he should be given intravenous volume resuscitation as well as intravenous
9
10
deferoxamine, the antidote for serious iron poisoning.
11 (Choice A) Sodium bicarbonate is used in cases of tricyclic anti depressant and aspirin overdose.
12
13 (Choice C) Magnesium sulfate is used in patients suffering from torsades de points induced by a prolonged QT
14 interval.

6
(Choice D) Hemodialysis is the treatment choice for severe lithium toxicity because it is the most dialyzable toxin.

17 (Choice E) Calcium EDTA is a lead chelator and is used for moderate to severe lead poisoning. Children with lead
18
poisoning present with irritability, poor appetite, headaches, abdominal pain and anemia.
19
20 (Choice F) Calcium gluconate is cardio-protective in cases of hyperkalemia.
21
22 (Choice G) Oral succimer can be used as a chelating agent in mild or moderate lead poisoning.
23
(Choice H) N-Acetylcysteine is used in cases of acetaminophen toxicity.
24
25 Educational Objective:
26 Iron poisoning disrupts basic cell processes, causing systemic manifestations including abdominal pain,
27
1 ·11- - .. - I• .: ... I� •IIII• I I I � - 1 • � - ■ 1 ... 1
Block Time Remaining: 00:46:39
28
29 TIMEDTUTOR

EN • @: ,j > .,,11 1•• 01:54 �


T• l/vl l/1 •
1M
1 •
=: ltem4of32
? 61 � � , � I)
2 Question Id: 2655

- ?
Mark <J
Previous Next
[:> Tutorial Lab Values Notes Calculator Reverse Color Text Zoom
3 •
and cool extremities indicate that he should be given intravenous volume resuscitation as well as intravenous
deferoxamine, the antidote for serious iron poisoning.
5
6 (Choice A) Sodium bicarbonate is used in cases of tricyclic antidepressant and aspirin overdose.
7
8 (Choice C) Magnesium sulfate is used in patients suffering from torsades de points induced by a prolonged QT
9 interval.
10
(Choice 0) Hemodialysis is the treatment choice for severe lithium toxicity because it is the most dialyzable toxin.
11
12 (Choice E) Calcium EDTA is a lead chelator and is used for moderate to severe lead poisoning. Children with lead
13
poisoning present with irritability, poor appetite, headaches, abdominal pain and anemia.
14
(Choice F) Calcium gluconate is cardio-protective in cases of hyperkalemia.
6
� (Choice G) Oral succimer can be used as a chelating agent in mild or moderate lead poisoning.
17
18 (Choice H) N-Acetylcysteine is used in cases of acetaminophen toxicity.
19
20 Educational Objective:
21 Iron poisoning disrupts basic cell processes, causing systemic manifestations including abdominal pain,
22 hematemesis, shock and metabolic acidosis. It commonly occurs in children of pregnant women taking pre-natal
23
vitamins because children often confuse brightly colored iron pills for candy. Treatment of iron poisoning involves
24
25
deferoxamine, which binds ferric iron, allowing urinary excretion.


26 Copyright© UWo�d. All rigllts reserved.
27 •
28
Block Time Remaining: 00:46:39
TIMEDTUTOR � Feedback
©
Suspend End Block

..-....
29

r§1 EN . 00: ,j, .,,11 I"•


01:54 �
� T• l/vl l/l •
1M
1 •
=: ltem 5of32
_ ? 61 � � , � I)
Question Id: 4837
:.
- ?
Mark <J
Previous Next
[:> Tutorial Lab Values Notes Calculator Reverse Color Text Zoom

G]
4 A 1-year-old boy is brought to the office for a routine visit. The patient recent ly transitioned from breast milk to

whole milk and enjoys various table foods, including fruits, vegetables, and grains. The family is vegetarian and
6
adds protein to his diet through beans and nut butters. He says 2 words, recently started walking, and often chews
7
8 on his toys and books. The family lives in a house built in the 1940s that has no chipping paint and has recently
9 been renovated, except for the basement and bedrooms. Height, weight, and head circumference are at the 50th
10 percentile for age. Physical examination is normal. Capillary blood test results are as follows:
11
12 Hemoglobin 12.5 g/dL
13
14
Lead 12 µg/dL (normal <5 µg/dL)

Which of the following is the most appropriate next step in management of this patient?

6
17
18
O A. Apply wallpaper to lead-based-paint walls
19 O B. Initiate calcium disodium edetate (EDTA)
20
21 0 C. Initiate dimercaprol (British anti-Lewisite)
22
23
0 D. Initiate dimercaptosuccinic acid (succimer)
24 0 E. Measure venous lead level
25
0 F. Perform abdominal x-ray


26
27
G. Prescribe oral ferrous sulfate
28
29
Block Time Remaining: 00:46:38
TIMEDTUTOR � Feedback
©
Suspend End Block

.... r§1
...- �
EN . 00: ,j, .,,ii 1•• 01:55 �
T• l/vl l/1 •
1M
1 •
=: ltem_5of32
? 61 � � , � I)
Question Id: 4837
:.
- ?Mark <J Previous Next
[:> Tutorial Lab Values Notes Calculator Reverse Color Text Zoom

G]
4
on his toys and books. The family lives in a house built in the 1940s that has no chipping paint and has recently •
been renovated, except for the basement and bedrooms. Height, weight, and head circumference are at the 50th
percentile for age. Physical examination is normal. Capillary blood test results are as follows:
6
7
Hemoglobin 12.5 g/dL
8
9 Lead 12 µg/dL (normal <5 µg/d�
10
11 Which of the following is the most appropriate next step in management of this patient?
12
13 A. Apply wallpaper to lead-based-paint walls [5%>]
14
B. Initiate calcium disodium edetate (EDTA) [15%]


6 ( C. Initiate dimercaprol (British anti-Lewi site) [9%]
17
18 X@' D. Initiate dimercaptosuccinic acid (succimer) [19%]
19
-, E. Measure venous lead level [44%]
20
21 F. Perform abdominal x-ray [3%]
22
23 G. Prescribe oral ferrous sulfate [2%>]
24
25
26
Incorrect
(1,, 44% I• • I:
L!!!. Answered correctl
27
II • •I
Block Time Remaining: 00:46:33
28
29 TIMEDTUTOR

EN • @: ,j > .,,II 1•• 01:55 �


T• l/vl l/1 •
1M
1 •
=: ltem 5of32
_ :. ? 61 � � , � I)
Question Id: 4837 - ? Mark <J
Previous Next
[:> Tutorial Lab Values Notes Calculator Reverse Color Text Zoom

G]
4 Lead poisoning

• Home built before 1978


6
• Pica or mouthing behaviors (eg, infants, developmental delay)
7
8 Risk factors • Sibling with lead poisoning
9 • Low socioeconomic status
10 • Immigrant or international adoptee
11
12 • May be asymptomatic
13 • Abdominal pain/constipation
14 Clinical features
• Cognitive impairment/behavioral problems
• Encephalopathy

6
17 • Obtain venous sample (if screening performed by capillary sample}
18
• Environmental surveillance (identify & remove lead sources)
19
20
Management • Notify public health department
21 • Nutritional counseling
22 • Chelation therapy if lead level :::45 ug/dL
23
24 Children are typically exposed to lead through inhalation or ingestion of lead particles from their environment.
25 They are at increased risk of lead poisoning compared with adults due to an incomplete blood-brain barrier,


26

• -- • • • .. • • ...�. ...�..= - .
increased mouthing of objects in their environment, and increased gastrointestinal absorption of lead. A risk factor
I ! - •-
27
• - - - - • - - ••.,.,,..,·c;-11..--.■Lo- •• a - I I II - I 1 - I -
28
29
Block Time Remaining: 00:46:33
TIMEDTUTOR � Feedback
©
Suspend End Block

,j, .,,ii
01:55 �
EN • 00: 1•• T• l/vl l/1 •
1M
1 •
=: ltem 5of32
_ ? 61 � � , � I)
Question Id: 4837
:.
- ?
Mark <J
Previous Next
[:> Tutorial Lab Values Notes Calculator Reverse Color Text Zoom

G]
4
Children are typically exposed to lead through inhalation or ingestion of lead particles from their environment. •
They are at increased risk of lead poisoning compared with adults due to an incomplete blood-brain barrier,
increased mouthing of objects in their environment, and increased gastrointestinal absorption of lead. A risk factor
6
7
for elevated lead levels is living in a home built before 1978, especially if there is peeling paint or dust released
8 during renovation. Other risk factors include lead piping, having a parent who works with batteries or pottery, or
9 having a playmate or sibling with a history of lead poisoning.
10
11
Patients who are symptomatic from lead toxicity can have anemia, abdominal pain, and encephalopathy. Targeted
12 screening of high-risk populations regardless of symptoms is important as most children with lead toxicity are
13 initially asymptomatic but can have cognitive and behavioral problems that become apparent after school entry.
14
Capillary (fingerstick) blood specimens are widely used in screening for lead poisoning, but false-positive
results are common due to environmental contamination and improper collection. Confirmatory venous lead

6
17 measurement is required to confirm the diagnosis of elevated lead level if a screening capillary lead level is �5
18 µg/dL.
19
20 (Choice A) Wallpapering over walls painted with lead-based paint is not an appropriate treatment measure, as the
21 paint can still loosen underneath the paper and release lead dust. Lead paint should be encapsulated or removed
22
by a professional to avoid exposure.
23
24 (Choices B, C, and D) This patient does not meet the threshold for chelation therapy. Although he is still at risk of
25 cognitive impairment, chelation therapy is not routinely administered for lead levels <45 µg/dL due to lack of


26
evidence for improved neurologic outcomes compared with removal from the lead-containing environment.
27
. .. -- -�- - -
- . .
.
28
29
Block Time Remaining: 00:46:33
TIMEDTUTOR � Feedback
©
Suspend End Block

,j, .,,ii
01:55 �
EN • 00: 1•• T• l/vl l/1 •
1M
1 �
=: ltem 5of32
_ :. ? 61 � � , � I)
Question Id: 4837 - ? Mark <J
Previous Next
[:> Tutorial Lab Values Notes Calculator Reverse Color Text Zoom

G]
4
•y p • p

(Choices B, C, and D) This patient does not meet the threshold for chelation therapy. Although he is still at risk of
6 cogn itive impairment, chelation therapy is not routinely administered for lead levels <45 µg/dL due to lack of
7 evidence for improved neurologic outcomes compared with removal from the lead-containing environment.
8 Dimercaptosuccinic acid (succimer) is typically used when lead levels are 45-69 µg/dL. Dimercaprol (British anti­
9
Lewisite) plus calcium disodium edetate (EDTA) should be administered on an emergency basis for levels �70
10
11
µg/dL or acute encephalopathy.
12 (Choice F) Abdominal x-rays can detect radiopaque lead flecks, paint chips, or lead-paint-covered foreign
13
bodies. Imaging is indicated for patients with elevated levels and gastrointestinal symptoms (constipation,
14
abdominal pain, vomiting) or suspicion of foreign body ingestion.


6 (Choice G) Patients with elevated blood lead levels should be screened for iron deficiency and prescribed oral
17
ferrous sulfate if deficiency exists. Comorbid iron deficiency can increase gastrointestinal absorption of lead.
18
19 Educational objective:
20 Targeted screening for elevated blood lead levels should be performed in children with risk factors (eg, home built
21
before 1978). Due to the potential of falsely elevated results with capillary testing, repeat testing by venous blood
22
23
draw should be performed to verify a high blood lead level.
24
25 References


26
27 • An update on childhood lead poisoning. •
28
29
Block Time Remaining: 00:46:33
TIMEDTUTOR � Feedback
©
Suspend End Block

.... r§1
...- �
EN � 00 ,j, .,,ii 1•• 01:55 �
T• l/vl l/1 •
1M
1 •
=: ltem 6of32
_ ? 61 � � , � I)
Question Id: 2662

- ?
Mark <J
Previous Next
C:::-j Tutorial Lab Values Notes Calculator Reverse Color Text Zoom

G]
4 A 40-year-old man is brought to the emergency department after his wife found him alone in a confused state. He

5
says he overdosed in a suicide attempt but does not specify which drug he took. En route to the hospital, he was
drowsy and ataxic with blurry vision. The pati ent has a history of asthma, insomnia, depression, and substance
7
8 abuse. His temperature is 37.8 C (100 F), blood pressure is 130/80 mm Hg, pulse is 100/min, and respirations are
9 22/min. Examination shows dry mucous membranes and skin. Pupils are 8 mm bilaterally. There is no neck
10 stiffness. Lungs are clear to auscultation bilaterally. Heart sounds are within normal limits. Abdominal
11
examination shows decreased bowel sounds without tenderness. A Foley catheter is placed, and 600 ml of urine
12
are collected immediately. Which of the following is the most likely diagnosis?
13
14
O A. Cocaine intoxication

6 O B. Diphenhydramine poisoning
17
18 O C. Phencyclidine poisoning
19
20
O D. Salicylate intoxication
21
0 E. Serotonin syndrome
22
23 0 F. Theophylline toxicity
24
25


26 Submit
27 •
28
29
Block Time Remaining: 00:45:37
TIMEDTUTOR � Feedback
©
Suspend End Block

.... r§1
...- �
EN . 00: ,j, .,,ii 1•• 01:56 �
T• l/vl l/1 •
1M
1 •
=: ltem_6of32
? 61 � � , � I)
Question Id: 2662
:.
- ?Mark <J
Previous Next
[:> Tutorial Lab Values Notes Calculator Reverse Color Text Zoom

G]
4 A 40-year-old man is brought to the emergency department after his wife found him alone in a confused state. He

5
says he overdosed in a suicide attempt but does not specify which drug he took. En route to the hospital, he was
drowsy and ataxic with blurry vision. The pati ent has a history of asthma, insomnia, depression, and substance
7
8 abuse. His temperature is 37.8 C (100 F), blood pressure is 130/80 mm Hg, pulse is 100/min, and respirations are
9 22/min. Examination shows dry mucous membranes and skin. Pupils are 8 mm bilaterally. There is no neck
10 stiffness. Lungs are clear to auscultation bilaterally. Heart sounds are within normal limits. Abdominal
11
examination shows decreased bowel sounds without tenderness. A Foley catheter is placed, and 600 ml of urine
12
are collected immediately. Which of the following is the most likely diagnosis?
13
14
A. Cocaine intoxication [4%]


6 "'@- B. Diphenhydramine poisoning [74%]
17
18 C. Phencyclidine poisoning [3%)
19
D. Salicylate intoxication [2%]
20
21 E. Serotonin syndrome [4%]
22
23 F. Theophylline toxicity [10%>]
24
25
26
I: t I:
.:..1 •-:..,
27
11 · ••• II • ;.,.1
Block Time Remaining: 00:45:30
28
29 TIMEDTUTOR

EN • @: ,j > .,,II 1•• 01:56 �


T• l/vl l/1 •
1M
1 •
=: ltem 6of32
_ :. ? 61 � � , � I)
Question Id: 2662 - ?
Mark <J
Previous Next
[:> Tutorial Lab Values Notes Calculator Reverse Color Text Zoom

G]
4
Diphenhydramine (used for allergic rhinitis, hives, insect b ites, and motion sickness) is an antihistamine with •

anticholinergic properties. Excessive amounts can cause significant antihistaminic (eg, confusion, drowsiness)
5
and anticholinergic effects. Findings suggestive of anticholinergic excess in this patient include:
7 • Dry mouth/dry skin ("dry as a bone")
8
• Blurry vision/mydriasis ("blind as a bat")
9
10
• Hyperthermia from impaired heat dissipation ("hot as a hare")
11 • Urinary retention ("full as a flask")
12 • Decreased bowel sounds
13
14 Patients with anticholinergic toxicity can also have cutaneous vasodilation ("red as a beet") and delirium or
hallucinations ("mad as a hatter"). Management of significant anticholinergic overdose can include administration

6 of physostigmine, a cholinesterase inhibitor.
17
18
(Choice A) Cocaine intoxication typically presents with elevated blood pressure and heart rate, feelings of
19 euphoria, and increased alertness. More life-threatening consequences include cardiac ischemia and intracranial
20 hemorrhage.
21
22 (Choice C) Phencyclidine poisoning can present with agitation, tachycardia, miosis, nystagmus (horizontal,
23 vertical, or rotary), and often psychotic and/or dissociative behavior.
24
(Choice D) Salicylate intoxication causes tinnitus, nausea/vomiting, and fever. It can also cause altered mental
25
status and acid-base abnormalities (mixed metabolic acidosis and respiratory alkalosis). Treatment includes


26
27 alkalinization of the urine with sodium bicarbonate. •
28
Block Time Remaining: 00:45:30
TIMEDTUTOR � Feedback
©
Suspend End Block

..-.
29

r§1 EN . 00: ,j, .,,ii 1•• 01:56 �


� T• l/vl l/1 •
1M
1 •
=: ltem 6of32
_ :. ? 61 � � , � I)
Question Id: 2662 - ?
Mark <J
Previous Next
[:> Tutorial Lab Values Notes Calculator Reverse Color Text Zoom

G]
4 (Choice D) Salicylate intoxication causes tinnitus, nausea/vomiting, and fever. It can also cause altered mental

5 status and acid-base abnormalities (mixed metabolic acidosis and respiratory alkalosis). Treatment includes
alkalinization of the urine with sodium bicarbonate.
7
8 (Choice E) Serotonin syndrome tends to occur after intentional overdose or drug interactions, classically when
9 monoamine oxidase inhibitors (which inhibit serotonin metabolism) are mixed with other drugs that increase
10
serotonergic activity (eg, serotonin reuptake inhibitors). Patients can develop autonomic hyperactivity (eg,
11
tachycardia, hyperthermia, increased bowel sounds, hypertension), mental status changes, dilated pupils, and
12
13 neuromuscular findings (eg, clonus, hyperreflexia).
14
(Choice F) Theophylline has a low therapeutic index, and intoxication can cause seizures, hyperthermia, cardiac
arrhythmias, tachycardia, and hypotension.

6
17 Educational objective:
18
Diphenhydramine overdose leads to antihistaminic (eg, drowsiness, confusion) and anticholinergic (eg, dry mouth,
19
dilated pupils, blurred vision, reduced bowel sounds, urinary retention) effects. Physostigmine, a cholinesterase
20
21 inhibitor, can counteract these anticholinergic effects.
22
23
References
24
25 • Poisonings with diphenhydramine--a survey of 68 clinical and 55 death cases.


26
Copyright© UWo�d. All rigllts reserved.
27
28
Block Time Remaining: 00:45:30
TIMEDTUTOR � Feedback
©
Suspend End Block

..-.
29

r§1 EN . 00: , j, .,>Ii 1•• 01:56 �


� T• l/vl l/l •
1M
1 •
=: ltem_7of32
? 61 � � , � I)
Question Id: 4509
:.
- ?Mark <J
Previous Next
[:> Tutorial Lab Values Notes Calculator Reverse Color Text Zoom

G]
4 A 32-year-old woman comes to the emergency department due to swelling of the left arm over the last 24 hours.

5
The patient drinks alcohol 2-3 times a week and uses intravenous heroin daily. She has no other chronic medical
6
problems and takes no medicati ons. Temperature is 37.8 C (100 F), blood pressure is 105/62 mm Hg, pulse is
8 92/min, and respirations are 16/min. Chest auscultation reveals clear lung fields and normal first and second heart
9 sounds. The abdomen is soft and nontender. Examination of the extremities shows several needle marks. The
10 left arm is erythematous and swollen as well as warm and tender to the touch. There is no joint swelling, and
11
range of motion is normal in all joints. The patient is started on intravenous clindamycin. The next morning the
12
swelling and pain are improved, but the patient reports "feeling miserable." She has nasal congestion, nausea,
13
14 and abdominal cramps in addition to multiple episodes of vomiting and loose stools. On repeat examination, the
patient is restless and reports aching muscles and joints. Blood pressure is 132/88 mm Hg, pulse is 102/min and

6 regular, and respirations are 16/min. Laboratory results from time of admission (0 hours) and 24 hours later are as
17
follows:
18
19 Complete blood count 0 hours 24 hours
20
21 Hemoglobin 12 g/dL 11.8 g/dL
22
Platelets 280,000/mm3 270,000/mm3
23
24 Leukocytes 13,800/mm3 8,500/mm3
25
26 Neutrophils 80% 62%
27
II• I •

Block Time Remaining: 00:45:26


28
29 TIMEDTUTOR

01:56 �
EN • 00: · •) .,>II 1•• T• l/vl l/1 •
1M
1 •
=: ltem 7of32
_ :. ? 61 � � , � I)
Question Id: 4509 - ?
Mark <J
Previous Next
[:> Tutorial Lab Values Notes Calculator Reverse Color Text Zoom

G]4
follows: •

5 Complete blood count 0 hours 24 hours


6
Hemoglobin 12 g/dL 11.8 g/dL
8
Platelets 280,000/mm3 270,000/mm3
9
10 Leukocytes 13,800/mm3 8,500/mm3
11
12 Neutrophils 80% 62%
13
Lymphocytes 15% 24%
14
Which of the following is the most appropriate next step in management of this patient?

6
17 0 A. Administer intravenous morphine
18
19 0 B. Discontinue clindamycin
20
21
0 C. Initiate intravenous naloxone
22 0 D. Initiate treatment for opioid withdrawal
23
24 0 E. Start flumazenil
25
0 F. Test for Clostridium difficile toxin


26
27 •
28
Block Time Remaining: 00:45:23
TIMEDTUTOR � Feedback
©
Suspend End Block

..-....
29

r§1 EN . 00: o j> .,>II 1•• 01:56 �


� T• l/vl l/1 •
1M
1 •
=: ltem_7of32
? 61 � � , � I)
Question Id: 4509
:.
- ?Mark <JPrevious Next
[:> Tutorial Lab Values Notes Calculator Reverse Color Text Zoom

G]
4 Hemoglobin 12 g/dL 11.8 g/dL

5
Platelets 280,000/mm3 270,000/mm3
6

Leukocytes 13,800/mm3 8,500/mm3


8
9 Neutrophils 80% 62%
10
11
Lymphocytes 15% 24%
12
Which of the following is the most appropriate next step in management of this patient?
13
14
A. Administer intravenous morphine [1%]


6 B. Discontinue clindamycin [5%]
17
18 C. Initiate intravenous naloxone [5%]
19
� D. Initiate treatment for opioid withdrawal [81%]
20
21 X E. Start flumazenil [1%>]
22
23 F. Test for Clostridium difficile toxin [5%>]
24
25
26
lnc�rect
(1,, 81%
L!!!. Answered correctl
27
II • •I
Block Time Remaining: 00:45:18
28
29 TIMEDTUTOR

01:56 �
EN •
00: · •) .,>II 1•• T• l/vl l/1 •
1M
1 •
=: ltem 7of32
_ :. ? 61 � � , � I)
Question Id: 4509 - ?Mark <J
Previous Next
[:> Tutorial Lab Values Notes Calculator Reverse Color Text Zoom

G]
4
Clinical features of opioid withdrawal

5 • Acute opioid cessation/dose reduction after prolonged use


6 • Gastrointestinal: nausea, vomiting, diarrhea, cramping, j bowel sounds
Clinical
• Cardiac:j pulse, j blood pressure, diaphoresis
8 presentation
9 • Psychological: insomnia, yawning, dysphoric mood
10 • Other: myalgias, arthralgias, lacrimation, rhinorrhea, piloerection, mydriasis
11
12 • Opioid agonist: methadone (preferred) or buprenorphine
13 Management • Nonopioid: clonidine or adjunctive medications (antiemetics, antidiarrheals,
14 benzodiazepines)

This patient with a history of intravenous drug abuse presents initially with upper extremity cellulitis, which appears

6
17 to respond appropriately to clindamycin, as noted by improved pain and resolution of leukocytosis. However, her
18 new symptoms are suggestive of acute opioid withdrawal. Patients with opioid dependence typically develop
19
withdrawal symptoms within 6-12 hours of the last dose of a short-acting opioid, with a peak 24-48 hours after
20
21
symptom onset. Manifestations frequently include nausea, vomiting, cramps, diarrhea, restlessness, rhinorrhea,
22 lacrimation, myalgias, and arthralgias. Examination can show hypertension, tachycardia, mydriasis,
23 piloerection, and hyperactive bowel sounds.
24
25 Although symptoms caused by natural opioid cessation (ie, not triggered by administration of an antagonist) are


26 not life-threatening, they can be quite uncomfortable. Low-dose methadone, a long-acting opioid agonist, is often
27 used in combination with adjunctive medications for symptom relief such as loperamide (diarrhea), ibuprofen •
28
Block Ti..... ""'"...........". .......45:18
TIMEDTUTOR � Feedback
©
Suspend End Block

..-••.
29

r§1 EN . 00: o j> .,>II 1•• 01:57 �


� T• l/vl l/1 •
1M
1 •
=: ltem 7of32
_ ? 61 � � , � I)
Question Id: 4509
:.
- ?
Mark <J
Previous Next
[:> Tutorial Lab Values Notes Calculator Reverse Color Text Zoom

G]
4
used in combination with adjunctive medications for symptom relief such as loperamide (diarrhea), ibuprofen •
(myalgias), baclofen (muscle cramps), and clonidine (anxiety, restlessness, hypertension).
5
6 Under federal law, emergency management of opioid withdrawal with methadone is permitted for up to 3 days if the
treatment is inpatient and the primary disease is medical (eg, cellulitis). This should not be confused with
8
methadone replacement therapy for chronic outpatient addiction management, where regulations are d ifferent.
9
10 (Choice A) Intravenous morphine should not be used in the management of opioid withdrawal. It is a potent,
11 highly addictive, and short-acting opioid, and its use could be detrimental to the long-term goal of overcoming
12
addiction.
13
14 (Choice B) Clindamycin can be associated with diarrhea, nausea, vomiting, and anorexia. However, the patient's
restlessness, arthralgias, and history of substance abuse are more suggestive of opioid withdrawal.

6
17 (Choice C) Intravenous naloxone is a potent opioid antagonist used for patients with opioid intoxication. It can
18 induce rapid withdrawal and should not be used in those who have withdrawal symptoms as it may cause acute
19 worsening.
20
21 (Choice E) Flumazenil is a benzodiazepine receptor antagonist and can be used in instances of benzodiazepine
22 intoxication (which typically causes depressed mental status with slurred speech), but it has no use in the
23 treatment of opioid withdrawal.
24
25 (Choice F) Clostridium difficile infection typically occurs in hospitalized patients with antibiotic exposure and can


26 cause abdominal cramps, nausea, and diarrhea. However, it is typically associated with leukocytosis (resolved in
27 this develo s within 5-10 da s of antibiotic use, and is unlike! •
••• •••
28
29
Block 11111.: n.t:llldllllllg. uu.45:18
TIMEDTUTOR � Feedback
©
Suspend End Block

01:57 �
EN • 00: o j> .,>II 1•• T• l/vl l/1 •
1M
1 �
=: ltem 7of32
_ :. ? 61 � � , � I)
Question Id: 4509 - ?
Mark <J
Previous Next
[:> Tutorial Lab Values Notes Calculator Reverse Color Text Zoom

G] g • ry • • gg p • •
4
(Choice C) Intravenous naloxone is a potent opioid antagonist used for patients with opioid intoxication. It can
5
6 induce rapid withdrawal and should not be used in those who have withdrawal symptoms as it may cause acute
worsening.
8
9
(Choice E) Flumazenil is a benzodiazepine receptor antagonist and can be used in instances of benzodiazepine
10 intoxication (which typically causes depressed mental status with slurred speech), but it has no use in the
11 treatment of opioid withdrawal.
12
13
(Choice F) Clostridium difficile infection typically occurs in hospitalized patients with antibiotic exposure and can
14 cause abdominal cramps, nausea, and diarrhea. However, it is typically associated with leukocytosis (resolved in
this patient), usual ly develops within 5-10 days of antibiotic use, and is unlikely to cause arthralgias.

6
17
Educational objective:
18 Common symptoms of opioid withdrawal include nausea, cramps, diarrhea, restl essness, rhinorrhea, lacrimation,
19 myalgias, and arthralgias. Examination can show hypertension, tachycardia, mydriasis, piloerecti on, and
20 hyperactive bowel sounds. Low-dose methadone with adjunctive medications is the treatment of choice.
21
22
23 References
24
• Treatment of Opioid-Use Disorders.
25


26 Copyright© UWo�d. All rigllts reserved.
27 •
28
Block Time Remaining: 00:45:18
TIMEDTUTOR � Feedback
©
Suspend End Block

..-.
29

r§1 �
EN � 00: ,j, .,,11 I"•
01:57 �
T• l/vl l/l •
1M
1 �
=: ltem 8of32
_ ? 61 � � , � I)
Question Id: 3690
:.
- ?
Mark <J
Previous Next
[:> Tutorial Lab Values Notes Calculator Reverse Color Text Zoom

G]
4 A 24-year-old recruit collapsed during military training on a bright, sunny, and humid day. He was disoriented at
5
the scene. He has no medical history and takes no medication. He does not use tobacco, alcohol, or illicit drugs.
6
Fami ly history is insignificant. On arrival in the emergency department, his temperature is 41°C (105.8°F), blood
7
pressure is 92/63 mm Hg, pulse is 140/min, and respirations are 22/min. Mucous membranes are dry, and the skin
9 is moist. Neck is supple. Auscultation of the chest is unremarkable. Abdomen is soft and nontender. Muscle tone
10 and reflexes are within normal limits. Laboratory testing shows a hematocrit of 54%>. Chest x-ray is within normal
11
limits. Urinalysis shows large blood on dipstick but no red blood cells on microscopy. Which of the following is the
12
most likely underlying pathophysiology of his current condition?
13
14
O A. Systemic cytokine activation

6 O B. Inadequate/failure of thermoregulation
17
18 O C. Calcium accumulation in muscle cells
19
20
O D. Inadequate fluid and salt replacement
21
0 E. Cardiac outlet obstruction
22
23
24
Submit
25


26
27
28
Block Time Remaining: 00:45:17
TIMEDTUTOR � Feedback
©
Suspend End Block

..-.
29

r§1 �
EN � 00: ,j, .,,II I"•
01:57 �
T• l/vl l/1 •
1M
1 •
=: ltem 8of32
_ ? 61 � � , � I)
Question Id: 3690
:.
- ?
Mark <J
Previous Next
[:> Tutorial Lab Values Notes Calculator Reverse Color Text Zoom

G]
4 A 24-year-old recruit collapsed during military training on a bright, sunny, and humid day. He was disoriented at

5
the scene. He has no medical history and takes no medication. He does not use tobacco, alcohol, or illicit drugs.
6
Fami ly history is insignificant. On arrival in the emergency department, his temperature is 41°C (105.8°F), blood
7
pressure is 92/63 mm Hg, pulse is 140/min, and respirations are 22/min. Mucous membranes are dry, and the skin
9 is moist. Neck is supple. Auscultation of the chest is unremarkable. Abdomen is soft and nontender. Muscle tone
10 and reflexes are within normal limits. Laboratory testing shows a hematocrit of 54%>. Chest x-ray is within normal
11
limits. Urinalysis shows large blood on dipstick but no red blood cells on microscopy. Which of the following is the
12
most likely underlying pathophysiology of his current condition?
13
14
A. Systemic cytokine activation [2%]


6 B. Inadequate/failure of thermoregulation [65%]
17
18 C. Calcium accumulation in muscle cells [3%]
19
20
X( D. Inadequate fluid and salt replacement [29%>]

1
21 E. Cardiac outlet obstruction [0%>]
22
23
24
1 ncorrect I
25 (1,, 65% fl\ 4 Seconds i==I 08/09/2018
Correct answer L!!!. Answered correctly '.::; TimeSpent l.!::.I Last Updated


B
26
27 •
28
29
Block Time Remaining: 00:45:14
TIMEDTUTOR � Feedback
©
Suspend End Block

.... r§1
...- �
EN . 00: ,j, .,,II I"•
01:57 �
T• l/vl l/1 •
1M
1 •
=: ltem 8of32
_ ? 61 � � , � I)
Question Id: 3690
:.
- ?
Mark <J
Previous Next
[:> Tutorial Lab Values Notes Calculator Reverse Color Text Zoom

G]
4
Heat stroke is categorized as exertional or nonexertional. Nonexertional heat stroke occurs in patients with chronic

medical conditions, often because they cannot remove themselves from the inciting stimulus and/or have impaired
5
6 thermoregulation due to medications and underlying illness. Exerti onal heat stroke occurs in otherwi se healthy
7 individuals undergoing conditioning in extreme heat and humidity. The body loses its ability to dissipate heat when
the humidity is over 75% and the temperature is elevated.
9
10 This patient, who collapsed while running for military training in hot, humid weather, provides a typical history for
11 exertional heat stroke. Signs and symptoms of heat stroke include temperature > 40°C (105°F), altered mental
12 status, hypotension, tachycardia, and tachypnea. Patients may have moist or dry skin and often are not volume­
13
depleted, depending on underlying medical conditions, original hydration status, and rapidity of onset.
14
A body temperature of 41°C (10S.8° F) may cause rhabdomyolysis and organ system damage. Large blood on

6 dipstick with no red blood cells by microscopy on urinalysis suggests rhabdomyolysis, the result of muscle
17
breakdown. Under normal conditions, body temperature is typical ly maintained at 36-37.5°C (96.8-99.5°F) by
18
dissipation of heat, primarily in the form of sweat. In heat stroke, the thermoregulatory center fails to dissipate heat
19
20 at the rate necessary to maintain a euthermic state. Although inadequate fl uid and salt replacement may
21 contribute to heat stroke, it does not cause it.
22
23
(Choice A) Cytokine activation during inflammation is the underlying pathophysiology of fever. Temperatures are
24 usually < 40° C (104° F). Fever results from the thermoregulatory center' s temporary upregulation of body
25 temperature (regulated at the level of the hypothalamus), not from a failure to dissipate heat.


26
(Choice C) Uncontrolled efflux of calcium from the sarcoplasmic reticulum is the underlying pathophysiology of
27 •
28
29
Block Time Remaining: 00:45:14
TIMEDTUTOR � Feedback
©
Suspend End Block

.... r§1
...- �
EN . 00: ,j, .,,ii 1•• 01:57 �
T• l/vl l/1 •
1M
1 •
=: ltem_8of32
? 61 � � , � I)
Question Id: 3690
:.
- ?Mark <J
Previous Next
[:> Tutorial Lab Values Notes Calculator Reverse Color Text Zoom

G]
4
temperature (regulated at the level of the hypothalamus), not from a failure to dissipate heat.

5 (Choice C) Uncontrolled efflux of calcium from the sarcoplasmic reticulum is the underlying pathophysiology of
6 malignant hyperthermia, a rare autosomal dominant disorder. It occurs in genetically susceptible patients after
7
administration of the anesthetic drugs halothane and succinylcholine and may be associated with severe
9 hyperthermia (up to 45° C [113°F]).
10
(Choice 0) Inadequate sodium and water replacement during physical activity can lead to heat exhaustion
11
because of the body's inability to maintain adequate cardiac output. Core body temperature is usual ly < 40° C
12
13 (104° F). Significant CNS dysfunction (eg, seizure, delirium) is not present.
14
(Choice E) Hypertrophic cardiomyopathy causes cardiac outlet obstruction. Like exertional heat stroke, it often
occurs in young, physical ly active individuals. Presenting symptoms include dyspnea, palpitations, syncope, and

6
17 sudden cardiac death.
18
Educational objective:
19
Exertional heat stroke occurs in otherwi se healthy individuals undergoing conditi oning in extreme heat and
20
21 humidity due to thermoregulation failure. Heat exhaustion is due to inadequate fluid and salt replacement. CNS
22 dysfunction (eg, altered mental status) is not present in heat exhaustion.
23
24
References
25
26 • The pathopysiology of heat stroke: an integrative view of the final common pathway
27

Block Time Remaining: 00:45:14


28
29 TIMEDTUTOR

EN • @: ,j > .,,11 1•• 01:57 �


T• l/vl l/1 •
1M
1 •
=: ltem_9of32
? 61 � � , � I)
Question Id: 12503
:.
- ?Mark <J
Previous Next
[:> Tutorial Lab Values Notes Calculator Reverse Color Text Zoom

G]
4
A 26-year-old man comes to the emergency department with a 4-day history of fever and chills. He has no chronic •

medical problems and takes no prescription medications. He drinks alcohol dai ly. Temperature is 38.3 C (101 F),
5
6
blood pressure is 110/70 mm Hg, pulse is 95/min, and respirations are 14/min. Examination reveals clear lung
7 fields and a pansystolic murmur at the lower sternal border that increases with inspiration. Multiple needle marks
8 are present on both upper extremities. Laboratory evaluation shows leukocytosis and an elevated erythrocyte
sedimentation rate. Chest x-ray reveals no airspace disease. Blood cultures are drawn and empiric antibiotics and
10
supportive treatment are begun. An echocardiogram is ordered. A day later, the nurse reports an acute change in
11
12 mentation. The patient had been conversing normally with a visitor shortly before the onset of somnolence. He is
13 responsive only to noxious stimuli. Blood pressure is 95/65 mm Hg, pulse is 70/min, and respirations are 10/min.
14 Pupils are equal and 1 mm bilaterally. The neck is supple and no jugular venous distension is seen. He can move
all extremities. Which of the following is the most likely etiology for the acute clinical change in this patient?

6
17 Q A. Cerebral abscess
18
19 0 8. Embolic stroke
0 C. Hypocalcemia
20
21
22
23
0 D. Hyponatremia
24 0 E. Opioid intoxication
0 F. Subarachnoid hemorrhage
25
26
27 - . ... ··-
Block Time Remaining: 00:45:10
28
29 TIMEDTUTOR

EN • @: ,j > .,,11 1•• 01:57 �


T• l/vl l/1 •
1M
1 •
=: ltem 9of32
_ ? 61 � � , � I)
Question Id: 12503
:.
- ?
Mark <J
Previous Next
[:> Tutorial Lab Values Notes Calculator Reverse Color Text Zoom

G]
4
are present on both upper extremities. Laboratory evaluation shows leukocytosis and an elevated erythrocyte

5 sedimentation rate. Chest x-ray reveals no airspace disease. Blood cultures are drawn and empiric antibiotics and
6 supportive treatment are begun. An echocardiogram is ordered. A day later, the nurse reports an acute change in
7 mentation. The patient had been conversing normally with a visitor shortly before the onset of somnolence. He is
8
responsive only to noxious stimuli. Blood pressure is 95/65 mm Hg, pulse is 70/min, and respirations are 10/min.
Pupils are equal and 1 mm bilaterally. The neck is supple and no jugular venous distension is seen. He can move
10
11 all extremities. Which of the following is the most likely etiology for the acute clinical change in this patient?
12
13 A. Cerebral abscess [4%)
14
8. Embolic stroke [34%)


6 ( C. Hypocalcemia [0%)
17
18 D. Hyponatremia [3%)
19
20
� 'i E. Opioid intoxication [50%)

21 F. Subarachnoid hemorrhage [1 %)
22
23 G. Wernicke encephalopathy [5%]
24
25


26
Corre� 11 .. 50% fl\ 12 Seconds i::==I 09/22/2018
27 L!!!. Answered correctly "-.::.; TimeSpent l.!!!.I Last Updated •
28
29
Block Time Remaining: 00:45:02
TIMEDTUTOR � Feedback
©
Suspend End Block

.... r§1
...- �
EN . @: ,j, .,,ii 1•• 01:57 �
T• l/vl l/1 •
1M
1 •
=: ltem 9of32
_ ? 61 � � , � I)
Question Id: 12503
:.
- ?
Mark <J
Previous Next
[:> Tutorial Lab Values Notes Calculator Reverse Color Text Zoom

G]
4 Clinical features of acute opioid intoxication

5
• Substance abuse
6
• Chronic opioid use
7 Risk factors
8 • Hospitalized patients (especially post-op)
• Hepatic or renal insufficiency
10
11
• Somnolence, AMS
12 • Pinpoint pupils (miosis)
13 Clinical • Shallow breathing & ! respiratory rate
14
findings • Bradycardia, hypothermia, ! bowel
sounds

6
17
• Respiratory acidosis on ABG
18
• Naloxone (may need repeated dosings)
19
• Airvvay management & ventilation
20 Management
21 • Exclude other AMS causes (eg,
22 hypoglycemia)
23
ABG = arterial blood gas; AMS = altered mental status.
24
25
This patient with fever, a pansystolic murmur at the lower sternal border, and evi dence of intravenous drug use (eg,


26
27 needle marks) likely has infective tricuspid valve endocarditis. His subsequent acute change in mental status,
28
Block Time Remaining: 00:45:02
TIMEDTUTOR � Feedback
©
Suspend End Block

..-••.
29

r§1 EN . @: ,j, .,,ii 1•• 01:57 �


� T• l/vl l/1 •
1M
1 •
=: ltem_9of32
? 61 � � , � I)
Question Id: 12503
:.
- ?Mark <J
Previous Next
[:> Tutorial Lab Values Notes Calculator Reverse Color Text Zoom

G]
4
This patient with fever, a pansystolic murmur at the lower sternal border, and evi dence of intravenous drug use (eg, •

needle marks) likely has infective tricuspid valve endocarditis. His subsequent acute change in mental status,
5
respiratory depression, and miosis suggest opioid intoxication (01). The most reliable and predictive sign of 01
6
7 is decreased respiratory rate. Other evidence includes hypotension and bradycardia. These findings in the
8 setting of an acute change after a vi sitor suggest occult opioid use during the hospitalization. Other common
findings include hypothermia and hypoactive bowel sounds. Neurologic examination is typically nonfocal.
10
11 Evaluation should include measurement of blood glucose, tests for other illicit substances, and consideration of
12 arterial blood gas analysis and ECG. Management focuses on protecting the airway and improving ventilation.
13 The prompt administration of naloxone can result in rapid improvements. This drug should be titrated to improve
14
the respiratory rate but not to achieve normal mental status.


6 (Choices A and B) Cerebral abscess and embolic cerebral vascular accidents are important considerations in
17 patients with infective endocarditis and abnormal neurologic findings. However, patients typically have focal
18
findings on examination, and neither condition is commonly associated with miosis or respiratory depression.
19
20 (Choices C and D) Acute hypocalcemia commonly presents with symptoms of tetany such as paresthesias,
21 muscle spasm, and diaphoresis, whereas acute hyponatremia presents with nausea and malaise. Seizures can
22
occur due to hypocalcemia, but the patient lacks other associated findings such as convulsions or loss of bowel or
23
24
bladder function. Patients who have hyponatremia can develop obtundation, but it does not usually present until a
25 markedly lower level of sodium occurs (ie, <120 mEq/L). Neither hypocalcemia nor hyponatremia would explain
26 this patient's respiratory depression or miosis.
27

Block Time Remaining: 00:45:02


28
29 TIMEDTUTOR

EN • @: ,j > .,,11 1•• 01:57 �


T• l/vl l/1 •
1M
1 •
=: ltem 9of32
_ ? 61 � � , � I)
Question Id: 12503
:.
- ?
Mark <J
Previous Next
[:> Tutorial Lab Values Notes Calculator Reverse Color Text Zoom

G]
4
(Choices C and D) Acute hypocalcemia commonly presents with symptoms of tetany such as paresthesias,

muscle spasm, and diaphoresis, whereas acute hyponatremia presents with nausea and malaise. Seizures can
5
6 occur due to hypocalcemia, but the patient lacks other associated findings such as convulsions or loss of bowel or
7 bladder function. Patients who have hyponatremia can develop obtundation, but it does not usually present until a
8 markedly lower level of sodium occurs (ie, <120 mEq/L). Neither hypocalcemia nor hyponatremia would explain
this patient's respiratory depression or miosis.
10
11 (Choice F) Subarachnoid hemorrhage almost universally presents with a sudden, severe headache, which this
12 patient lacks. Patients may also demonstrate signs of meningismus. Loss of consciousness, if present, is typical ly
13
bri ef.
14
(Choice G) Wernicke encephalopathy is characterized by disorientation, inattentiveness, and nystagmus (not

6 miosis) on examination. Acute obtundation and respiratory depression are not common.
17
18 Educational objective:
19 Opioid intoxication is commonly characterized by mental status changes, respiratory depression, and miosis.
20 Hypotension, bradycardia, hypothermia, and decreased bowel sounds may also be present.
21
22
23 References
24
• Illicit opioid intoxication: diagnosis and treatment.
25


26 Copyright© UWo�d. All rigllts reserved.
27 •
28
Block Time Remaining: 00:45:02
TIMEDTUTOR � Feedback
©
Suspend End Block

..-.
29

r§1 EN . @: ,j, .,,11 I"•


01:57 �
� T• l/vl l/l •
1M
1 �
=: ltem 10of32
_ :. ? 61 � � , � 0
Question Id: 23TT - ?
Mark <J
Previous Next
[:> Tutorial Lab Values Notes Calculator Reverse Color Text Zoom

G]
4 A 40-year-old man is brought to the emergency department for retrosternal and epigastric pain after ingesting an
5
unknown amount of sodium hydroxide-based drain cleaner 45 minutes ago. He has a history of major depression
6
and a prior suicide attempt. The patient has difficulty swallowing his saliva and is drooling. His temperature is 36.8
7
8 C (98.2 F), blood pressure is 120/70 mm Hg, pulse is 110/min, and respirations are 20/min. Examination shows
9 oropharyngeal erythema and mild edema. Lungs are clear to auscultation. Abdominal examination shows
tenderness at the epigastrium without rebound or guarding. Chest x-ray shows no abnormalities. Intravenous
11
normal saline infusion is initiated. Which of the following is the most appropri ate next step in management of this
12
patient?
13

0 A. Activated charcoal in water


14


6 O B. Gastric decontamination with ipecac
17
18 0 C. Intravenous methylprednisolone
19
20
O D. Neutralization of alkali with dilute acetic acid solution
21
0 E. Upper gastrointestinal endoscopy
22
23
24
Submit
25


26
27
28
Block Time Remaining: 00:45:01
TIMEDTUTOR � Feedback
©
Suspend End Block

..-.
29

r§1 �
EN � @: ,j, .,,ii 1•• 01:57 �
T• l/vl l/1 •
1M
1 •
=: ltem 10of32
_ :. ? 61 � � , � 0
Question Id: 23TT - ?
Mark <J
Previous Next
[:> Tutorial Lab Values Notes Calculator Reverse Color Text Zoom

G]
4 A 40-year-old man is brought to the emergency department for retrosternal and epigastric pain after ingesting an

5
unknown amount of sodium hydroxide-based drain cleaner 45 minutes ago. He has a history of major depression
6
and a prior suicide attempt. The patient has difficulty swallowing his saliva and is drooling. His temperature is 36.8
7
8 C (98. 2 F), blood pressure is 120/70 mm Hg, pulse is 110/min, and respirations are 20/min. Examination shows
9 oropharyngeal erythema and mild edema. Lungs are clear to auscultation. Abdominal examination shows
tenderness at the epigastrium without rebound or guarding. Chest x-ray shows no abnormalities. Intravenous
11
normal saline infusion is initiated. Which of the following is the most appropri ate next step in management of this
12
patient?
13
14
A. Activated charcoal in water [25%]


6 B. Gastric decontamination with ipecac [7%]
17
18 C. Intravenous methylprednisolone [5%>]
19
X r D. Neutralization of alkali with dilute acetic acid solution [10%]
20
21 �

I
E. Upper gastrointestinal endoscopy [50%]
22
23
24


Incorrect fl\ 4 Seconds 09/12/2018
25 (1,, 50% i==I
Correct answer L!!!. Answered correctly '.::; TimeSpent l.!::.I Last Updated
E
26
27 •
28
29
Block Time Remaining: 00:44:58
TIMEDTUTOR � Feedback
©
Suspend End Block

.... r§1
...- �
EN . @: ,j, .,,II I"•
01:57 �
T• l/vl l/1 •
1M
1 •
=: Item 10 of 32
_ ? 61 l:°f" � , � 0
Question Id: 23TT -
:. ?
Mark <J
Previous Next
[:> Tutorial Lab Values Notes Calculator Reverse Color Text om Zo

G]
4 Caustic ingestion

5
6 Chemical burn or liquefaction necrosis resulting in:
7
Clinical features • Laryngeal damage: Hoarseness, stridor
8
• Esophageal damage: Dysphagia, odynophagia
9
• Gastric damage: Epigastric pain, bleeding
11
12 • Secure airway, breathing, circulation
13 • Decontamination: Remove contaminated clothing
14 Management & visible chemicals; irrigate exposed skin
• Chest x-ray if respiratory symptoms

6
• Endoscopy within 24 hours
17
18
19
• Upper airway compromise
20
• Perforation
21
Complications • Strictures/stenosis (2-3 weeks)
22
• Ulcers
23
• Cancer
24 © USMLEWorld, LLC

25
This patient's presentation is consistent with a likely suicide attempt by ingestion of sodium hydroxide (lye), a


26
27 strongly caustic alkaline solution. The severity of caustic esophageal injury depends on the ingested substance's
28
29
Block Time Remaining: 00:44:58
TIMEDTUTOR � Feedback
©
Suspend End Block

...-
.... r§1 �
EN . @: ,j, .,,II I"•
01:57 �
T• l/vl l/1 •
1M
1 •
=: ltem 10of32
_ :. ? 61 � � , � 0
Question Id: 23TT - ?
Mark <J
Previous Next
[:> Tutorial Lab Values Notes Calculator Reverse Color Text Zoom

G]
4
This patient's presentation is consistent with a likely suicide attempt by ingestion of sodium hydroxide (lye), a

5 strongly caustic alkaline solution. The severity of caustic esophageal injury depends on the ingested substance's
6 corrosive properties, amount and concentration, and duration of contact with the mucosa. Patients typically
7 develop vomiting with occasional hematemesis, dysphagia/odynophagia, retrosternal or epigastric pain, and
8
hypersalivation. Absence of oral injury does not rule out significant esophageal damage.
9
Alkali ingestion can cause immediate chemical burn or liquefaction necrosis in the esophagus that spreads within
11 seconds to minutes through the esophageal wall toward the mediastinum. Possible complications include
12
perforation and mediastinitis. This process can last for 3-4 days with slow mucosal healing over the next 1-3
13
14
months. Patients should initially undergo decontamination of skin and clothing and receive intravenous hydration.
Serial chest and abdominal x-rays should be obtained to identify any perforation. Upper gastrointestinal x-ray

6 study with water-soluble contrast should be performed in patients with suspected perforation.
17
18
In the absence of perforation or severe respiratory distress, early endoscopic evaluation within the first 12-24 hours
19 is recommended in hemodynamically stable patients to assess the severity of esophageal damage. Patients with
20 no or mild esophageal injury may be managed with simple supportive measures. However, more severe injury
21 may require tube feedings and possible surgery (esophagectomy).
22
23 (Choice A) Activated charcoal can decrease systemic absorption of poisons and is used for many types of toxic
24 ingestions. However, lye causes immediate local damage on contact with the esophagus without significant
25 systemic absorption. In addition, charcoal is contraindicated as it will also obstruct the view during endoscopy.


26
27 (Choice B) Lye is unlikely to harm the stomach as the gastric lining is resistant to pH effects, and hydrochloric acid •
28
29
Block Time Remaining: 00:44:58
TIMEDTUTOR � Feedback
©
Suspend End Block

.... r§1
...- �
EN . @: ,j, .,,ii 1•• 01:58 �
T• l/vl l/1 •
1M
1 •
=: ltem 10of32
_ :. ? 61 � � , � 0
Question Id: 23TT - ?
Mark <J
Previous Next
[:> Tutorial Lab Values Notes Calculator Reverse Color Text Zoom

G]
4
(Choice B) Lye is unlikely to harm the stomach as the gastric lining is resistant to pH effects, and hydrochloric acid

5 wi ll eventually neutralize the ingested lye. However, inducing vomiting with ipecac will re-expose the esophagus to
6 the lye and potentially cause further injury.
7
(Choice C) Corticosteroid therapy following caustic ingestion has been advocated to reduce the risk of
8
9 esophageal stricture. However, studies have not shown that corticosteroids prevent stricture development. In
addition, they may increase the risk of secondary infections and are not generally recommended.
11
12
(Choice 0) Acidic solutions should not be given orally to patients with alkali ingestions as such reactions usually
13 generate excess heat and cause further mucosal damage. In addition, most of the damage from alkali occurs
14 instantly after ingestion and would not benefit from weak acid administration.

Educational objective:

6
17 Caustic ingestion with sodium or potassium hydroxide (lye) causes immediate esophageal injury with liquefaction
18 necrosis and potential perforation. Endoscopy is recommended within the first 12-24 hours to assess the severity
19 of damage and guide further therapy. Activated charcoal, corticosteroids, emetics, and acid neutralization are not
20
recommended.
21
22
23 References
24
• Corrosive injury to the GI tract in adults: a practical approach.
25


26 • Caustic ingestion in adu lts: the role of endoscopic classification in predicting outcome.
27 •
28
Block Time Remaining: 00:44:58
TIMEDTUTOR � Feedback
©
Suspend End Block

..-.
29

r§1 EN . @: ,j, .,,ii 1•• 01:58 �


� T• l/vl l/1 •
1M
1 •
=: ltem H of32
:. ? 61 � � , � 0
Question Id: 12397 - ?
Mark <J
Previous Next
[:> Tutorial Lab Values Notes Calculator Reverse Color Text Zoom

G]
4 A 40-year-old man comes to the office due to a month of progressively worsening burning and tingling sensations

5
in the hands and feet. He restores antique furniture as a hobby but recently has had difficulty handling heavy
6
tools. The patient has no medical issues and takes no medications. He does not use tobacco or alcohol. Blood
7
8 pressure is 100/70 mm Hg, and pulse is 70/min. The skin over the neck has patchy areas of hyperpigmentation
9 and hypopigmentation. Hyperkeratoses and scaling are present on the palms and soles. There is increased
10 sensitivity to pinprick and light touch over the fingers and toes. Plantarflexion and dorsiflexion are weak at the
ankle, and there is weakness of the interossei and wrist flexors and extensors. Upper and lower limb deep tendon
12
reflexes are 1+. Laboratory results are as follows:
13
14
Complete blood count


6 Hemoglobin 10.4 g/dL
17
Leukocytes 4,100/mm3
18
19 130,000/mm3
Platelets
20
21 Liver function studies
22
23
Aspartate aminotransferase 50 U/L
24 Alanine aminotransferase 62 U/L
25


26 Which of the following is the most likely diagnosis for this patient?
27
28
29
Block Time Remaining: 00:44:55
TIMEDTUTOR � Feedback
©
Suspend End Block

.... r§1
...- �
EN . @: o j> .,>II 1•• 01:58 �
T• l/vl l/1 •
1M
1 •
=: ltem H of32
:. ? 61 � � , � 0
Question Id: 12397 - ?
Mark <J
Previous Next
[:> Tutorial Lab Values Notes Calculator Reverse Color Text Zoom

G]4 Leukocytes 4,100/mm3


5
6 Platelets 130,000/mm3
7
Liver function studies
8
9 Aspartate aminotransferase 50 U/L
10
Alanine aminotransferase 62 U/L
12
Which of the following is the most likely diagnosis for this patient?
13
14
0 A. Arsenic poisoning

6 0 B. Guillain-Barre syndrome
17
18 0 C. Intermittent porphyria
19
20
0 D. Lead poisoning
21 0 E. Vitamin A deficiency
22
23 0 F. Vitamin 8 deficiency
12

24
25


26 Submit
27 •
28
Block Time Remaining: 00:44:52
TIMEDTUTOR � Feedback
©
Suspend End Block

..-....
29

r§1 EN . @: o j> .,>II 1•• 01:58 �


� T• l/vl l/1 •
1M
1 •
=
-
Item 11 of 32
Question Id: 12397
_
� ?
Mark <J
Previous
C>
Next
?
Tutorial
61
Lab Values

N�tes

Calculator
,
Reverse Color
� 0
Text z oom

G]
4 Leukocytes 4,100/mm3

5
6 Platelets 130,000/mm3
7
Liver function studies
8
9 Aspartate aminotransferase 50 U/L
10
Alanine aminotransferase 62 U/L
12
Which of the following is the most likely diagnosis for this patient?
13
14
A. Arsenic poisoning [30%]

� B. Guillain-Barre syndrome [0%]


6
17
18 C. Intermittent porphyria [14%]
19
D. Lead poisoning [40%]
20
21 E. Vitamin A deficiency [5%]
22
23 X r- F. Vitamin 812 deficiency [7%>]
24
25
26
lnc�ect
: I:
27
-- ..- II • •I
Block Time Remaining: 00:44:49
28
29 TIMEDTUTOR

01:58 �
EN • @: · •) .,,II 1•• T• l/vl l/1 •
1M
1 •
=: Item H of32 ? 61 l:°f" � , � 0
Question Id: 12397 :.
- ?Mark <J
Previous Next
[:> Tutorial Lab Values Notes Calculator Reverse Color Text om Zo

G]
4 Arsenic poisoning

5
6 • Binds to sulfhydryl groups
7
Mechanism • Disrupts cellular respiration & gluconeogenesis
8
9 • Pesticides/insecticides
Sources • Contaminated water (often from wells)
10
• Pressure-treated wood
12
13 • Acute: Garlic breath, vomiting, watery diarrhea,
QTc prolongation
14 Manifestations
• Chronic: Hypo/hyperpigmentation, hyper1<eratosis,
stocking-glove neuropathy

6
17
• Dimercaprol (British anti-Lewisite)
18 Treatment
• DMSA (meso-2,3-dimercaptosuccinic acid, succimer)
19
©UWor1d
20
21 This patient, who is routinely exposed to antique wood, has polyneuropathy, pancytopenia, mild transaminase
22
elevation, and a variety of skin lesions characteristic of chronic arsenic toxicity.
23
24 Arsenic is a metalloid element that exists in numerous forms, including both natural and manmade compounds.
25 Although humans are often exposed to low levels through diet and water, toxic exposure is more likely through
26
mining, pesticide manufacturing, metalworking, and working with antique pressure-treated wood preserved with
27
- . . . . . . . . .. . . . - • • · - · - ·· · . - • - . .. - - .. . . - . . -
Block Time Remaining: 00:44:49
28
29 TIMEDTUTOR

01:58 �
EN • @: · •) .,>II 1•• T• l/vl l/1 •
1M
1 •
=: ltem H of32
:. ? 61 � � , � 0
Question Id: 12397 - ?
Mark <J
Previous Next
[:> Tutorial Lab Values Notes Calculator Reverse Color Text Zoom

G]
4
Although humans are often exposed to low levels through diet and water, toxic exposure is more likely through

mining, pesticide manufacturing, metalworking, and working with antique pressure-treated wood preserved with
5
6 arsenic. Arsenic binding to sulfhydryl groups and interfering with various enzymes is likely responsible for the
7 clinical manifestations of acute and chronic arsenic toxicity.
8
9
Patients chronically exposed to arsenic develop a sensorimotor neuropathy in a stocking-glove distribution with
10 burning, painful hypersensitivity, distal weakness, and hyporeflexia. Hypo- and hyperpigmentation of the skin
occurs early in arsenic toxicity, and hyperkeratosis and scaling of the soles and palms is often seen later. Mees
12 lines (horizontal striation of fingernails) are characteristic. Acute, severe arsenic toxicity can lead to profound
13
pancytopenia and hepatitis, and chronic exposure leads to these manifestations to a smaller degree. Diagnosis
14
is made through history, physical examination, and measurement of urine arsenic levels. Chelation (eg,
dimercaprol, dimercaptosuccinic acid) is the treatment of choice for severe toxicity.

6
17
(Choice B) Guillain-Barre syndrome presents with progressive motor neuropathy and decreased/absent deep­
18
19
tendon reflexes. However, it is not typically associated with hypersensitivity to tactile sensation, skin changes, or
20 liver enzyme abnormalities.
21
(Choice C) Acute interm ittent porphyria is caused by a partial deficiency of porphobilinogen deaminase, an
22
23
enzyme involved in heme synthesis. Acute attacks are triggered by a number of factors and are associated with
24 patchy sensorimotor neuropathy and autonomic dysfunction. However, severe abdominal pain is typical, and skin
25 involvement or cytopenias are rare.


26
(Choice D) Chronic lead poisoning can lead to sensorimotor neuropathy in a stocking-glove distribution along with
27 •
28
29
Block Time Remaining: 00:44:49
TIMEDTUTOR � Feedback
©
Suspend End Block

.... r§1
...- �
EN . @: o j> .,>II 1•• 01:58 �
T• l/vl l/1 •
1M
1 •
=: ltem H of32
:. ? 61 � � , � 0
Question Id: 12397 - ?
Mark <J
Previous Next
[:> Tutorial Lab Values Notes Calculator Reverse Color Text Zoom

G]
4 (Choice D) Chronic lead poisoning can lead to sensorimotor neuropathy in a stocking-glove distribution along with

5 microcytic anemia. Although the neuropathy is similar to arsenic toxicity, gastrointestinal complaints are typical and
6 skin changes are not usually seen.
7
8 (Choice E) Vitamin A deficiency is associated with xerophthalmia (pathologic eye dryness), night blindness,
9 follicular hyperkeratosis (rather than on palms and soles), and immunity impairment. It is rare in developed
10 countries and not generally associated with polyneuropathy.

12 (Choice F) Vitamin B,2 deficiency can lead to macrocytic anemia, loss of position and vibration sensation, ataxia,
13 spasticity, and incontinence. It is not associated with liver enzyme abnormalities or hyporeflexia. The skin
14 changes seen in this patient are also more suggestive of arsenic toxicity.

Educational objective:

6
17 Arsenic toxicity should be suspected in a patient with possible environmental exposure (eg, pressure-treated wood,
18 pesticides) who has painful sensorimotor polyneuropathy, skin lesions (hypo- and hyperpigmented, hyperkeratotic),
19
pancytopenia, and mild transaminase elevation. Diagnosis is confirmed with elevated urine arsenic levels.
20
21
22 References
23
• The broad scope of health effects from chronic arsenic exposure: update on a worldwide public health
24
25 problem.


26
Copyright© UWo�d. All rigllts reserved.
27 •
28
Block Time Remaining: 00:44:49
TIMEDTUTOR � Feedback
©
Suspend End Block

..-.
29

r§1 EN . @: o j> .,>II 1•• 01:58 �


� T• l/vl l/1 •
1M
1 �
=: ltem 12of32
_ :. ? 61 � � , � 0
Question Id: 3223 - ?
Mark <J
Previous Next
[:> Tutorial Lab Values Notes Calculator Reverse Color Text Zoom

G]
4 A 35-year-old woman is brought to the emergency department after being rescued from inside a burning building
5
by firefighters. She had a brief tonic-clonic seizure en route to the hospital. Her past medical history is unknown.
6
She is confused and mildly agitated. Her temperature is 37 C (98.6 F), blood pressure is 100/60 mm Hg, pulse is
7
8 115/min, and respirations are 24/min. Her oxygen saturation is 96%> on room air as measured by standard pulse
9 oximetry. Physical examination shows no burns, and her skin color is normal. There are symmetric breath sounds
10 bilaterally with scattered end-expiratory wheezes. Neurologic examination shows no abnormalities apart from
11
confusion. The abdomen is soft and nontender. Which of the following is the best initial treatment for this patient?

13
14
QA. 50% dextrose with thiamine
0 B. 100%> oxygen with facemask

6
17 0 C. Intravenous lorazepam
18
19
0 D. Intravenous naloxone

20 0 E. Intravenous phenytoin
21
22
23 Submit
24
25


26
27
28
Block Time Remaining: 00:44:48
TIMEDTUTOR � Feedback
©
Suspend End Block

..-.
29

r§1 �
EN � @: o j> .,>II 1•• 01:58 �
T• l/vl l/1 •
1M
1 •
=: ltem 12of32
_ :. ? 61 � � , � 0
Question Id: 3223 - ?
Mark <J
Previous Next
[:> Tutorial Lab Values Notes Calculator Reverse Color Text Zoom

G]
4 A 35-year-old woman is brought to the emergency department after being rescued from inside a burning building

5
by firefighters. She had a brief tonic-clonic seizure en route to the hospital. Her past medical history is unknown.
6
She is confused and mildly agitated. Her temperature is 37 C (98.6 F), blood pressure is 100/60 mm Hg, pulse is
7
8 115/min, and respirations are 24/min. Her oxygen saturation is 96%> on room air as measured by standard pulse
9 oximetry. Physical examination shows no burns, and her skin color is normal. There are symmetric breath sounds
10 bilaterally with scattered end-expiratory wheezes. Neurologic examination shows no abnormalities apart from
11
confusion. The abdomen is soft and nontender. Which of the following is the best initial treatment for this patient?

13
A. 50% dextrose with thiamine [3%)
14
1' � B. 100%> oxygen with facemask [88%]
6

17
C. Intravenous lorazepam [5%]
18
D. Intravenous naloxone [0%>]
19

I
20 E. Intravenous phenytoin [2%>]
21
22
23
24 (1, , 88% II\ 6 Seconds I==! 06/11/2018
L!!!. Answere<I correctly I..::; TimeSpent l.!!!.I Last Update<!
Correo�


25
26
27
28
29
Block Time Remaining: 00:44:43
TIMEDTUTOR � Feedback
©
Suspend End Block

.... r§1
...- �
EN . @: o j> .,>II 1•• 01:58 �
T• l/vl l/1 •
1M
1 •
=:
ltem 12of32
_
Question Id: 3223
:.
- ? Mark <J
Previous Next
[:>
?
Tutorial
61
Lab Values

Notes

Calculator
,
Reverse Color
� 0
Text Zoom

G]
4 Carbon monoxide poisoning

5
• Smoke inhalation
6
7 Epidemiology • Defective heati ng systems
8 • Motors operating in poorly ventilated areas
9
10 Mild-moderate:
11 • Headache, confusion
• Malaise, dizziness, nausea
13 Manifestations
Severe:
14
• Seizure, syncope, coma
• Myocardial ischemia, arrhythmias

6
17
• ABG - carboxyhemoglobin level
18
Diagnosis • ECG
19
20 • Cardiac enzymes (if ischemia or CAD)
21
• High-flow 100% oxygen
22 Treatment
23
• lntubation/hyperbaric oxygen therapy (severe)
24
ABG = arterial blood gas; CAD = coronary artery disease.
25
26 This patient presents with confusion, wheezing, and a seizure following smoke inhalation; this clinical picture
• •L=-i,.--ii-"!� � I I II I I I .. I -
27
• � I I I I �
Block Time Remaining: 00:44:43
28
29 TIMEDTUTOR

EN • @: · •) .,>II 1•• 01:58 �


T• l/vl l/1 •
1M
1 •
=: ltem 12of32
_ :. ? 61 � � , � 0
Question Id: 3223 - ?
Mark <J
Previous Next
[:> Tutorial Lab Values Notes Calculator Reverse Color Text Zoom

G]
4
This patient presents with confusion, wheezing, and a seizure following smoke inhalation; this clinical picture •

suggests carbon monoxide (CO) poisoning. CO is a tasteless, colorless, and odorless gas produced by
5
incomplete combustion of carbon-containing compounds. CO poisoning should be considered in all patients
6
7 exposed to smoke in a closed space. The affinity of CO for binding hemoglobin (Hb) is >200 times that of oxygen;
8 once bound to Hb, CO forms carboxyhemoglobin, which impairs oxygen delivery to tissue by shifting the Hb­
9 oxygen dissociation curve to the left.
10
11 Manifestations of mild to moderate CO toxicity include headache, nausea, dyspnea, malaise, altered mentation,
and dizziness. Severe CO poisoning can present with seizures, coma, syncope, heart failure, or arrhythmias.
13 Bright cherry lips can be a sign of CO poisoning (not spec ific). The diagnosis is confirmed clinically and by
14
documenting an elevated carboxyhemoglobin level (eg, >3% in nonsmokers, >10% in smokers). A standard pulse
oxymetry is unreliable and may appear normal because it cannot differentiate carboxyhemoglobin from

6
17 oxyhemoglobin (as seen in this patient).
18
The treatment of CO poisoning involves administration of 100% oxygen via nonrebreather facemask to compete
19
with CO binding to Hb and to decrease the half-life of CO (from ~5 hours on room air to 1-2 hours on 100%
20
21 oxygen). Patients should then be mon itored (for >4 hours) and hospitalized if their condition has not improved.
22 Hyperbaric oxygen is sometimes used in severe cases that are unresponsive to facemask-administered oxygen.
23
24
(Choice A) A bolus of 50% dextrose can be given for suspected hypoglycemia in an unconscious patient.
25 Intravenous thiamine can be given for suspected Wernicke encephalopathy (delirium, oculomotor abnormalities,


26 ataxia). This patient' s presentation after being inside a burning building is more concerning for CO poisoning.
27
-. - -
28
29
Block Time Remaining: 00:44:43
TIMEDTUTOR � Feedback
©
Suspend End Block

EN . @: o j> .,>II 1•• 01:58 �


T• l/vl l/1 •
1M
1 �
=: ltem 12of32
_ :. ? 61 � � , � 0
Question Id: 3223 - ? Mark <J
Previous Next
[:> Tutorial Lab Values Notes Calculator Reverse Color Text Zoom

G]
4
•• • •• • • • I II

oxygen). Patients should then be monitored (for >4 hours) and hospitalized if their condition has not improved.
5
Hyperbaric oxygen is sometimes used in severe cases that are unresponsive to facemask-administered oxygen.
6
7 (Choice A) A bolus of 50% dextrose can be given for suspected hypoglycemia in an unconscious patient.
8 Intravenous thiamine can be given for suspected Wernicke encephalopathy (delirium, oculomotor abnormalities,
9
ataxia). This patient' s presentation after being inside a burning building is more concerning for CO poisoning.
10
11 (Choices C and E) Intravenous lorazepam and phenytoin are treatment options if the patient continues to have
seizures or develops status epilepticus.
13
14 (Choice 0) Intravenous naloxone is indicated to reverse (suspected) opioid toxicity with respiratory depression,
but this patient's respiratory rate of 24/min makes this less likely.

6
17
Educational objective:
18 All patients with smoke inhalation should be suspected to have acute carbon monoxide (CO) poisoning and treated
19 with 100% oxygen via a nonrebreather facemask. Early symptoms of CO poisoning are typically neurological and
20 include agitation, confusion, and somnolence.
21
22
23 References
24
• Carbon monoxide poisoning
25


26 Copyright© UWo�d. All rigllts reserved.
27 •
28
29
Block Time Remaining: 00:44:43
TIMEDTUTOR � Feedback
©
Suspend End Block

.... r§1
...- �
EN � @: o j> .,>II 1•• 01:58 �
T• l/vl l/l •
1M
1 �
=: ltem 13of32
_ :. ? 61 � � , � 0
Question Id: 4084 - ?
Mark <J
Previous Next
[:> Tutorial Lab Values Notes Calculator Reverse Color Text Zoom

G]
4 A group of teenagers attend an indoor barbecue on a cold winter night. They all ate servings of potato salad and
5
barbecue chicken. A few hours later, they present to the local emergency room with headache, nausea, vomiting,
6
vague abdominal discomfort and confusion. A quick physical examination of one patient reveals tachycardia,
7
8 tachypnea and a pinkish-skin hue. Which of the following is the most likely diagnosis?
9
10 0 A. Viral illness
11
12
0 B. Carbon monoxide poisoning
0 C. Acute gastroenteritis
14
0 D. Cyanide poisoning

6
17
0 E. Methemoglobinemia
18
19
Submit
20
21
22
23
24
25


26
27
28
Block Time Remaining: 00:44:42
TIMEDTUTOR � Feedback
©
Suspend End Block

..-.
29

r§1 �
EN � @: o j> .,>II 1•• 01:58 �
T• l/vl l/1 •
1M
1 •
=: ltem 1 3 of32
_ ? 61 � � , � 0
Question Id: 4084
:.
- ?Mark <J Previous Next
[:> Tutorial Lab Values Notes Calculator Reverse Color Text Zoom

G]
4 A group of teenagers attend an indoor barbecue on a cold winter night. They all ate servings of potato salad and

5
barbecue chicken. A few hours later, they present to the local emergency room with headache, nausea, vomiting,
6
vague abdominal discomfort and confusion. A quick physical examination of one patient reveals tachycardia,
7
8 tachypnea and a pinkish-skin hue. Which of the following is the most likely diagnosis?
9
10 A. Viral illness [0%]
11
�@ B. Carbon monoxide poisoning [86%]
12
C. Acute gastroenteriti s [5%]
14
D. Cyanide poisoning [3%>]


6

I
E. Methemoglobinemia [3%]
17
18
19 �
"
20 "
Iii, 86% II\ 4 Seconds ¢=l 06/18/2018
21 Co L!!!. Answere<l correctly TimeSpent
'-:::J l.!!!.I Last Update<l
22
23
24 Explanation
25
26
Carbon monoxide is a colorless, odorless gas emitted by automobiles, furnaces and charcoal grills. When inhaled,
27

Block Time Remaining: 00:44:39


28
29 TIMEDTUTOR

EN • @: · •) .,>II 1•• 01:59 �


T• l/vl l/1 •
1M
1 •
=: ltem 13of32
_ ? 61 � � , � 0
Question Id: 4084
:.
- ?
Mark <J
Previous Next
[:> Tutorial Lab Values Notes Calculator Reverse Color Text Zoom

G]
4
Carbon monoxide is a colorless, odorless gas emitted by automobiles, furnaces and charcoal grills. When inhaled,

it prevents the body's tissues from utilizing oxygen effectively. Acute exposure can present with headaches,
5
6 nausea, vomiting, abdominal discomfort, confusion and coma. Patients often present with a pinkish-red skin hue.
7 Diagnosis is established by obtaining carboxyhemoglobin levels. Treatment consists of hyperbaric oxygen
8 administration.
9
10 (Choices A and C) Headache, nausea and vomiting common ly occur in viral illness and gastroenteritis. However,
11 a fire with improper ventilation and pinkish skin hue suggest carbon monoxide poisoning.
12
(Choice D) Burning of rubber or plastic, not wood, can result in cyanide inhalation. The symptoms can be similar
14 to those of carbon monoxide exposure, so the history is very important. Bitter almond breath is characteristic of
inhaled cyanide.

6
(Choice E) Methemoglobinemia presents with many similarities to carbon monoxide poisoning. It can be induced
17
18 by certain drugs and environmental exposures. Unlike carbon monoxide and cyanide poisoning, it presents with
19 cyanosis and bluish discoloration of skin and mucous membranes.
20
Educational objective:
21
22 The most important factor when diagnosing carbon monoxide poisoning is the history, because physical symptoms
23 can be vague. Suspect carbon monoxide poisoning in cases of smoke inhalation and when multiple people from
24 the same confined quarters present with headache, nausea and abdominal discomfort. Pinkish-red skin hue is
25
noted on examination and diagnosis is confirmed by carboxyhemoglobin level.


26
27 Copyright© UWo�d. All rigllts reserved. •
28
Block Time Remaining: 00:44:39
TIMEDTUTOR � Feedback
©
Suspend End Block

..-.
29

r§1 EN . @: , j, .,>Ii 1•• 01:59 �


� T• l/vl l/l •
1M
1 �
=: ltem 14of32
_ :. ? 61 � � , � 0
Question Id: 3138 - ?
Mark <J
Previous Next
[:> Tutorial Lab Values Notes Calculator Reverse Color Text Zoom

G]
4 A 35-year-old woman is brought to the emergency department after an apparent suicide attempt. Her current
5
prescription of imipramine was found at her bedside along with a suicide note. En route to the hospital, she
6
suffered a seizure. The patient has been treated with many different antidepressants over the past several years
7
8 without improvement. Her temperature is 38.3 C (101 F), blood pressure is 90/50 mm Hg, pulse is 120/min, and
9 respirations are 24/min. Electrocardiogram shows QRS widening (140 msec). She is given sodium bicarbonate
10 along with supportive treatment. Which of the following best explains the beneficial effects of sodium bicarbonate
11
in this patient?
12
13
Q A. Alkalinization of urine will promote diuresis of the antidepressant

15 Q B. Alkalinization of serum wi ll prevent life-threatening hyperkalemia


16
17
Q C. Alkalinization of serum wi ll improve tissue oxygen delivery
18 Q D. Sodium load will alleviate depressant action on myocardial potassium channels
19
20 Q E. Sodium load will alleviate depressant action on myocardial sodium channels
21
22
23 Submit
24
25


26
27
28
Block Time Remaining: 00:44:38
TIMEDTUTOR � Feedback
©
Suspend End Block

..-.
29

r§1 �
EN � @: o j> .,>II 1•• 01:59 �
T• l/vl l/1 •
1M
1 •
=: ltem 14of32
_ :. ? 61 � � , � 0
Question Id: 3138 - ?
Mark <J
Previous Next
[:> Tutorial Lab Values Notes Calculator Reverse Color Text Zoom

G]
4 A 35-year-old woman is brought to the emergency department after an apparent suicide attempt. Her current

5
prescription of imipramine was found at her bedside along with a suicide note. En route to the hospital, she
6
suffered a seizure. The patient has been treated with many different antidepressants over the past several years
7
8 without improvement. Her temperature is 38.3 C (101 F), blood pressure is 90/50 mm Hg, pulse is 120/min, and
9 respirations are 24/min. Electrocardiogram shows QRS widening (140 msec). She is given sodium bicarbonate
10 along with supportive treatment. Which of the following best explains the beneficial effects of sodium bicarbonate
11
in this patient?
12
13
A. Alkalinization of urine will promote diuresis of the antidepressant [35%]

15 X@' B. Alkalinization of serum will prevent life-threatening hyperkalemia [19%]


16
17
C. Alkalinization of serum wi ll improve tissue oxygen delivery [2%]
18
D. Sodium load will alleviate depressant action on myocardial potassium channels [14%]
19

1
20 E. Sodium load will alleviate depressant action on myocardial sodium channels [28%]
21
22
23
1 ncorrecl:s
l1o. 28% II\ 5 Seconds i==l 10/31/2018
Corr�t answer
24
L!!!. Answere<I correctly TimeSpent
'-:::.J l.!!!.I Last Update<!
25 E


26
27
28
29
Block Time Remaining: 00:44:34
TIMEDTUTOR � Feedback
©
Suspend End Block

.... r§1
...- �
EN . @: o j> .,>II 1•• 01:59 �
T• l/vl l/1 •
1M
1 •
=: Item _1 4 of 32
? 61 l:°f" � , � 0
Question Id: 3138 :.
- ?
Mark <J
Previous Next
[:> Tutorial Lab Values Notes Calculator Reverse Color Text Zoom

G]
4
Clinical features & management of tricyclic antidepressant overdose

5
6 Central • Mental status changes (eg, drowsiness,
nervous delirium, coma)
7
system • Seizures, respiratory depression
8
9
10
• Sinus tachycardia, hypotension
Clinical
11 presentation Cardiovascular • Prolonged PR/QRS/QT intervals
12 • Arrhythmias (eg, ventricular tachycardia,
13 fibrillation)

15 • Dry mouth, blurred vision, dilated pupils


Anticholinergic
16 • Urinary retention, flushing, hyperthermia
17
18 • Supplemental oxygen, intubation
19 • Intravenous fluids
20 Supportive • Activated charcoal for patients within 2
Management
21 care & therapy hours of ingestion (unless ileus present)
22 • Intravenous sodium bicarbonate for QRS
23 widening or ventricular arrhythmia
24 ©uwo�d.com
25
Patients with tricyclic antidepressant (TCA) overdose can have central nervous system toxicity (eg, sedation,


26
27 seizures, coma), anticholinergic toxicity (eg, hyperthermia, dilated pupils, intestinal ileus, urinary retention), and •
28
29
Block Time Remaining: 00:44:34
TIMEDTUTOR � Feedback
©
Suspend End Block

...-
••. r§1 �
EN . @: • •> .,IIJ 1■a,
01:59 �
T• l/vl l/1 •
1M
1 �
=: ltem 14of32
_ :. ? 61 � � , � 0
Question Id: 3138 - ?
Mark <J
Previous Next
[:> Tutorial Lab Values Notes Calculator Reverse Color Text Zoom

G]
4
• I -' I •

seizures, coma), anticholinergic toxicity (eg, hyperthermia, dilated pupils, intestinal ileus, urinary retention), and
5
cardiac toxicity. Cardiac toxicity causes most deaths in TCA overdose. TCAs inhibit fast sodium channels in the
6
His-Purkinje system and myocardium. This decreases conducti on velocity, increases duration of repolarization,
7
8 and prolongs absolute refractory periods. The result is hypotension, QRS prolongation, and ventricular
9 arrhythmias (eg, ventricular tachycardia, ventri cular fibrillation).
10
QRS interval >100 msec in the setting of TCA overdose is an indication for sodium bicarbonate therapy. Sodium
11
12 bicarbonate increases serum pH and extracellular sodium. Increased pH (goal 7.50-7.55) decreases drug avidity
13 for sodium channels. Elevated extracellular sodium concentration increases the electrochemical gradient across
cardiac cells and affects the ability of TCAs to bind to fast sodium channels.
15
16 (Choice A) U rine alkalinization is the mechanism by which sodium bicarbonate increases salicylate excretion in
17 aspirin overdose. However, it is not the mechanism responsible for alleviating QRS prolongation in TCA overdose.
18
(Choice B) Sodium bicarbonate is an adjunctive therapy for treating significant hyperkalemia or rapidly increasing
19
20 potassium with electrocardiogram changes (eg, peaked T waves, short QT, increased QRS interval). However,
21 TCA overdose is not associated with severe hyperkalemia.
22
23
(Choice C) Sodium bicarbonate has no effect on tissue oxygen delivery. In fact, some studies have shown
24 increased mortality in patients with sepsis and lactic acidosis who were treated with sodium bicarbonate.
25
(Choice D) TCAs have a depressant action on the cardiac fast sodium channels but not on potassium channels.


26
27 Educational objective: •
28
Block Time Remaining: 00:44:34
TIMEDTUTOR � Feedback
©
Suspend End Block

..-.
29

r§1 �
EN � @: o j> .,>II 1•• 01:59 �
T• l/vl l/1 •
1M
1 •
=: ltem 14of32
_ ? 61 � � , � 0
Question Id: 3138
:.
- ?
Mark <J
Previous Next
[:> Tutorial Lab Values Notes Calculator Reverse Color Text Zoom

G]
4
bicarbonate increases serum pH and extracellular sodium. Increased pH (goal 7.50-7.55) decreases drug avidity

5
for sodium channels. Elevated extracellular sodium concentration increases the electrochemical gradient across
6 cardiac cells and affects the ability of TCAs to bind to fast sodium channels.
7
(Choice A) Urine alkalinization is the mechanism by which sodium bicarbonate increases salicylate excretion in
8
9 aspirin overdose. However, it is not the mechanism responsible for alleviating QRS prolongation in TCA overdose.
10
(Choice B) Sodium bicarbonate is an adjunctive therapy for treating significant hyperkalemia or rapidly increasing
11
potassium with electrocardiogram changes (eg, peaked T waves, short QT, increased ORS interval}. However,
12
13 TCA overdose is not associated with severe hyperkalemia.

(Choice C) Sodium bicarbonate has no effect on tissue oxygen delivery. In fact, some studies have shown
15
increased mortality in patients with sepsis and lactic acidosis who were treated with sodium bicarbonate.
16
17 (Choice D) TCAs have a depressant action on the cardiac fast sodium channels but not on potassium channels.
18
19 Educational objective:
20 Tricyclic antidepressant overdose can present with central nervous system, cardiac, and anticholinergic findings.
21 Sodium bicarbonate is used to treat cardiac toxicity, which is characterized by prolonged QRS duration (>100
22
msec) and ventricular arrhythmias. Sodium bicarbonate increases serum pH and extracellular sodium, thereby
23
alleviating the cardio-depressant action on sodium channels.
24
25


26 References
27
28
29
Block Time Remaining: 00:44:34
TIMEDTUTOR � Feedback
©
Suspend End Block

.... r§1
...- �
EN . @: o j> .,>II 1•• 01:59 �
T• l/vl l/1 •
1M
1 •
=: ltem 15of32
_ ? 61 � � , � 0
Question Id: 10146
:.
- ?
Mark <J
Previous Next
[:> Tutorial Lab Values Notes Calculator Reverse Color Text Zoom

G]
4

5 The following vignette applies to the next 2 items


6
A 27-year-old woman is brought to the emergency department by local paramedics. The patient was found
7
8 unconscious at the scene of a house fire. On examination, she does not appear to have any burns. Black soot is
9 present near the patient' s nares and mouth. Her capillary refill time is 4 seconds. Blood pressure is 132/90 mm
10 Hg, heart rate is 122/min and regular, and respirations are 24/min. Supplemental oxygen by a non-rebreather
11
mask is administered. Laboratory results are as follows:
12

..
13 Arterial blood gases
14
pH 7.15

114 mm
17
18
Hg
19 PaC02 33 mm Hg
20
21
Blood, plasma, and serum
22
23 Bicarbonate 12 mEq/L
24
25 Lactic acid,
20 mg/dL


26 venous
27 •
28
Block Time Remaining: 00:44:31
TIMEDTUTOR � Feedback
©
Suspend End Block

..-.
29

r§1 EN . @: o j> .,>II 1•• 01:59 �


� T• l/vl l/1 •
1M
1 •
= Item 15 of 32
- Question Id: 10146
_
� ?
Mark <J
Previous
C>
Next
?
Tutorial
61
Lab Values

N�tes

Calculator
,
Reverse Color
� 0
Text zoom

G]4 33 mm Hg

5
6
Blood, plasma, and serum
7
8 Bicarbonate 12 mEq/L
9
10
Lactic acid,
20 mg/dl
11 venous
12
Item 1 of 2

..
13
14
Which of the following should be empirical ly treated in this patient?

17 0 A. Cyanide poisoning
18
19
0 B. Methanol exposure
20 0 C. Methemoglobinemia
21
22 0 D. Salicylate poisoning
23
24
0 E. Status epilepticus
25


26
Submit
27 •
28
Block Time Remaining: 00:44:28
TIMEDTUTOR � Feedback
©
Suspend End Block

..-....
29

r§1 EN . @: o j> .,111 1•• 01:59 �


� T• l/vl l/1 •
1M
1 •
= Item 15 of 32
- Question Id: 10146
_
� ?
Mark <J
Previous
C>
Next
?
Tutorial
61
Lab Values

N�tes

Calculator
,
Reverse Color
� 0
Text zoom

G]
4
33 mm Hg

5
6 Blood, plasma, and serum
7
Bicarbonate 12 mEq/L
8
9 Lactic acid,
10
20 mg/dl
venous
11
12
Item 1 of 2

..
13
14 Which of the following should be empirical ly treated in this patient?

A. Cyanide poisoning [36%]


17
18 B. Methanol exposure [2%]
19
X r C. Methemoglobinemia [55%>]
20
21 D. Salicylate po�oning [3%]
22
23 E. Status epilepticus [1 %]
24
25
26
Incorrect
27
-- ..- II • •I
Block Time Remaining: 00:44:24
28
29 TIMEDTUTOR

01:59 �
EN • @: · •) .,,II 1•• T• l/vl l/1 •
1M
1 •
=: Item 1 5 of32
_ :. ? 61 l:°f" � , � 0
Question Id: 10146 - ?Mark <J
Previous Next
[:> Tutorial Lab Values Notes Calculator Reverse Color Text om Zo

G]
4 Treatment overview for suspected cyanide poisoning

5
6 Dermal exposure
7 • Removal of clothing
8 • Skin decontamination
9 Ingestion
10 • Activated charcoal
11 Decontamination All exposures
12 • Antidote
o Hydroxocobalamin preferred

..
13
14 o Sodium thiosulphate as alternate therapy
• Antidote not available
o Nitrites to induce methemoglobinemia

17
18 • No mouth-to-mouth resuscitation
Respiratory
• Supplemental oxygen
19 support
• Airway protection (intubation)
20
21
Cardiovascular
22
support • Intravenous fluids for hypotension
23
24 ©UWOl1d

25 This patient is the victim of a house fire and most likely has a smoke inhalation injury; such injuries are estimated
26
to cause 60%-80% of deaths during fire incidents. Smoke injury resu lts in glottic edema from heat and airway
27

Block Time Remaining: 00:44:24


28
29 TIMEDTUTOR

01:59 �
EN • @: · •) .,>II 1•• T• l/vl l/1 •
1M
1 •
=: ltem 15of32
_ ? 61 � � , � 0
Question Id: 10146
:.
- ?
Mark <J
Previous Next
[:> Tutorial Lab Values Notes Calculator Reverse Color Text Zoom

G]
4
This patient is the victim of a house fire and most likely has a smoke inhalation injury; such injuries are estimated

5
to cause 60%-80% of deaths during fire incidents. Smoke injury results in glottic edema from heat and airway
6 irritation due to particulate matter found in smoke. Smoke also contains multiple toxins that are absorbed
7 systemically. Hydrogen cyanide (HCN) and carbon monoxide (CO) are the 2 major products of combustion in
8 closed spaces. HCN is produced from the combustion of nitrogen-containing synthetic polymers (eg, foam, cotton,
9
paint, silk).
10
11 HCN is a potent and fast-acting poison, and blood levels cannot be measured rapidly to confirm diagnosis prior to
12 treatment. Toxic exposure can be inhalational, dermal, or intestinal. Exposure to moderate to high concentrations

..
13
causes symptoms to develop within seconds to minutes. Early acute toxicity causes neurologic and
14
cardiorespiratory stimulation, and patients develop headache, verti go, dizziness, hyperventilation, tachycardia,
nausea, and vomiting. Neurologic, respiratory, and cardiovascular depression eventual ly occurs and manifests as
17 coma, seizures, bradycardia, hypotension, and cardiorespiratory arrest. HCN can also cause anoxic brain injury
18 leading to permanent neurologic deficits.
19
20 Cyanide toxicity can be treated with an anti dote such as hydroxocobalamin or sodium thiosulfate, which direct ly
21 binds cyanide molecules. An alternate treatment is induction of methemoglobinemia with nitrites to increase ferric
22 iron (Fe3+) in circulating hemoglobin. Cyanide binds avidly to Fe3+, and so methemoglobinemia provides an
23
alternate binding site.
24
25 (Choice B) Methanol is found in antifreeze solutions (eg, windshield wiper fluid, fuels, solvents). Poisoning occurs


26
after ingestion of such compounds.
27 •
28
29
Block Time Remaining: 00:44:24
TIMEDTUTOR � Feedback
©
Suspend End Block

.... r§1
...- �
EN . @: o j> .,>II 1•• 01:59 �
T• l/vl l/1 •
1M
1 �
=: ltem 15of32
_ ? 61 � � , � 0
Question Id: 10146
:.
- ?
Mark <J
Previous Next
[:> Tutorial Lab Values Notes Calculator Reverse Color Text Zoom

G] = J. • • • • •

4 after ingestion of such compounds.


5
6 (Choice C) Methemoglobin is formed by the oxidation of ferrous (Fe2+) to ferric (Fe3+) iron in hemoglobin. Fe3+
7 is unable to bind oxygen, and it also increases the affinity of Fe2+ to oxygen, causing a left shift in the oxygen
8 dissociation curve. The result is functional anemia due to reduced oxygen delivery to tissues.
9
Methemoglobinemia occurs after exposure to oxidizing agents (eg, dapsone, nitrates, topical/local
10
anesthetics).
11
12 (Choice 0) Salicylate toxicity usually results from intentional or accidental ingestion of 10-30 g of salicylic acid in

..
13
adults (3 g in children). Because levels can be readily measured, there is no need to treat salicylate toxicity
14
empirically in this patient, who is more likely to have suffered HCN and CO poisoning.

(Choice E) Antiepileptics may be needed if seizures or status epilepticus develops as a complication of cyanide
17
toxicity. This patient has no symptoms of ongoing seizure activity. Early decontamination with an antidote is
18
crucial in the prevention and treatment of this complication.
19
20 Educational objective:
21
To prevent cardiorespiratory arrest and permanent neurologic disability, victims of smoke inhalation injury should
22
23
be treated empirically for cyanide toxicity with an antidote, such as hydroxocobalamin or sodium thiosulfate, or with
24 nitrites to induce methemoglobinemia.
25


26
References
27 •
28
29
Block Time Remaining: 00:44:24
TIMEDTUTOR � Feedback
©
Suspend End Block

.... r§1
...- �
EN � @: o j> .,>II 1•• 01:59 �
T• l/vl l/1 •
1M
1 �
=: ltem 16of32
_ :. ? 61 � � , � 0
Question Id: 10145 - ?
Mark <J
Previous Next
[:> Tutorial Lab Values Notes Calculator Reverse Color Text Zoom

G]4 Item 2 of 2
5
6 Which of the following is the primary cause of this patient' s acid-base disturbance?
7
8 0 A. Impaired excretion of lactic acid
10
9
0 B. Increased gut absorption of lactic acid
11 0 C. Increased ketoacid production
12
0 D. Increased metabolic rate

-
13
14
0 E. Reduced oxygen utilization by tissues
17
Submit
18
19
20
21
22
23
24
25


26
27
28
Block Time Remaining: 00:44:23
TIMEDTUTOR � Feedback
©
Suspend End Block

..-....
29

r§1 �
EN � @: o j> .,>II 1•• 01:59 �
T• l/vl l/1 •
1M
1 •
=: ltem 16of32
_ :. ? 61 � � , � 0
Question Id: 10145 - ?
Mark <J
Previous Next
[:> Tutorial Lab Values Notes Calculator Reverse Color Text Zoom

G]4 Item 2 of 2

5
6 Which of the following is the primary cause of this patient' s acid-base disturbance?
7
8 A. Impaired excretion of lactic acid [7%]
9
B. Increased gut absorption of lactic acid [0%]
10
11 C. Increased ketoacid production [3%]
12
D. Increased metabolic rate [5%]

-
13
14
"@ E. Reduced oxygen utilization by tissues [82%>]

17
18
(1,, 82% II\ 4 Seconds ¢=l 05/29/2018
19 L!!!. Answere<l correctly TimeSpent
'-:::J I.!!!.! Last Update<l
20
21
22 Explanation
23
24
Cyanide is a potent inhibitor of cytochrome oxidase a3 in the mitochondrial electron transport chain. It binds to
25
ferric iron (Fe3+), inhibiting its reduction to ferrous iron (Fe2+) and blocking production of ATP from oxidative


26
27 phosphorylation. Cells then switch to anaerobic metabolism, leading to lactic acid formation and causing metabolic •
28
Block Time Remaining: 00:44:20
TIMEDTUTOR � Feedback
©
Suspend End Block

..-....
29

r§1 EN . @: o j> .,>II 1•• 01:59 �


� T• l/vl l/1 •
1M
1 •
=: ltem 16of32
_ :. ? 61 � � , � 0
Question Id: 10145 - ?
Mark <J
Previous Next
[:> Tutorial Lab Values Notes Calculator Reverse Color Text Zoom

G]
4
Cyanide is a potent inhibitor of cytochrome oxidase a3 in the mitochondrial electron transport chain. It binds to

5 ferric iron (Fe3+), inhibiting its reduction to ferrous iron (Fe2+) and blocking production of ATP from oxidative
6 phosphorylation. Cells then switch to anaerobic metabolism, leading to lactic acid formation and causing metabolic
7 acidosis. Serum bicarbonate levels fall in an attempt to buffer excess acid. Metabolic acidosis also triggers central
8
and peripheral chemoreceptors, increasing alveolar ventilation and then presenting as tachypnea. This leads to a
9
fall in arterial PC02 (PaC02).
10
11 (Choice A) Impaired excretion of lactic acid and other serum acids occurs in renal failure and results in anion gap
12
metabolic acidosis.

-
13
14 (Choice B) Increased gut absorption of lactic acid may occur in blind loop syndrome, in which excessive
production and absorption of D-lactate causes hypochloremic metabolic acidosis.

17 (Choice C) Increased ketogenesis occurs during starvation and with lack of insulin in insulin dependent diabetics.
18
(Choice D) An increased metabolic rate may cause lactic acidosis if insufficient oxygen is present in tissues and
19
20 anaerobic metabolism ensues. It is common in post-seizure lactic acidosis.
21
Educational objective:
22
Cyanide binds to ferric iron in cytochrome oxidase a3 in the mitochondrial electron transport chain. This activity
23
24 blocks oxidative phosphorylation and promotes anaerobic metabolism, causing lactic acidosis.
25


26 References
27 •
28
29
Block Time Remaining: 00:44:20
TIMEDTUTOR � Feedback
©
Suspend End Block

.... r§1
...- �
EN . @: o j> .,>II 1•• 01:59 �
T• l/vl l/1 •
1M
1 •
=: ltem 17of32
_ :. ? 61 � � , � 0
Question Id: 4510 - ?
Mark <J
Previous Next
[:> Tutorial Lab Values Notes Calculator Reverse Color Text Zoom

G]
4 A 38-year-old homeless man is found obtunded on the sidewalk and brought to the emergency department. No

5
medical history is available. Review of the medical records shows no prior encounters. Temperature is 34 C (93.2
6
F), blood pressure is 91/63 mm Hg, pulse is 95/min, respirations are 5/min, and oxygen saturation is 86% on room
7
8 air . The patient looks disheveled and his breath smells of alcohol. He is unresponsive to verbal and noxious
9 stimuli. Examination demonstrates dry oral mucosa and no jugular venous distension. Lung fields are clear, first
10 and second heart sounds are normal, and bowel sounds are decreased. The abdomen is soft and nontender.
11
Extremities are cool to the touch and peripheral pulses are diminished. The pupils are 3 mm (normal, 2-4 mm in
12
bright light) bilaterally. Muscular tone is decreased in all extremities. Deep tendon reflexes are 2+ throughout. A
13
14 flexor plantar response is present. Supplemental oxygen is given via facemask, and preparation is made for
endotracheal intubation. A large-bore intravenous catheter is placed. In addition to intravenous fluids, thiamine,
6 and glucose, which of the following is the most appropriate immediate next step in management of this patient?

18
QA. Buprenorphine
19
20 0 B. CT scan of the head
21
22
0 C. Epinephrine
23 0 D. External rewarming
24
25 0 E. Glucagon


26
27
0 F. Lorazepam •
28
Block Time Remaining: 00:44:14
TIMEDTUTOR � Feedback
©
Suspend End Block

..-.
29

r§1 EN . @: o j> .,>II 1•• 02: 00 �


� T• l/vl l/1 •
1M
1 •
=: ltem 17of32
_ ? 61 � � , � 0
Question Id: 4510
:.
- ?
Mark <J
Previous Next
[:> Tutorial Lab Values Notes Calculator Reverse Color Text Zoom

G]
4
and second heart sounds are normal, and bowel sounds are decreased. The abdomen is soft and nontender. •
Extremities are cool to the touch and peripheral pulses are diminished. The pupils are 3 mm (normal, 2-4 mm in
5
bright light) bilaterally. Muscular tone is decreased in all extremities. Deep tendon reflexes are 2+ throughout. A
6
7 flexor plantar response is present. Supplemental oxygen is given via facemask, and preparation is made for
8 endotracheal intubation. A large-bore intravenous catheter is placed. In addition to intravenous fluids, thiamine,
9 and glucose, which of the following is the most appropriate immediate next step in management of this patient?
10
11
QA. Buprenorphine
12
13 0 B. CT scan of the head
14
0 C. Epinephrine

6 0 D. External rewarming
18 0 E. Glucagon
19
20
0 F. Lorazepam
21 0 G. Naloxone
22
23 0 H. Urine toxicology
24
25


26 Submit
27 •
28
29
Block Time Remaining: 00:44:12
TIMEDTUTOR � Feedback
©
Suspend End Block

.... r§1
...- �
EN . @: o j> .,>II 1•• 02: 00 �
T• l/vl l/1 •
1M
1 •
=: ltem _1 7 of32
:. ? 61 � � , � 0
Question Id: 4510 - ?Mark <J Previous Next
[:> Tutorial Lab Values Notes Calculator Reverse Color Text Zoom

G]
4
and second heart sounds are normal, and bowel sounds are decreased. The abdomen is soft and nontender. •
Extremities are cool to the touch and peripheral pulses are diminished. The pupils are 3 mm (normal, 2-4 mm in
5
bright light) bilaterally. Muscular tone is decreased in all extremities. Deep tendon reflexes are 2+ throughout. A
6
7 flexor plantar response is present. Supplemental oxygen is given via facemask, and preparation is made for
8 endotracheal intubation. A large-bore intravenous catheter is placed. In addition to intravenous fluids, thiamine,
9 and glucose, which of the following is the most appropriate immediate next step in management of this patient?
10
11 A. Buprenorphine [0%>]
12
13 B. CT scan of the head [7%]
14
C. Epinephrine [1%>]

6 ( D. External rewarming [57%]


18 E. Glucagon [0%]
19
F. Lorazepam [3%]
20
21 � 'i G. Naloxone [24%)
22
23 H. Urine toxicology [3%]
24
25
26
(1,, 24% fl\ 10 Seconds i::==I 08/03/2018
L!!!. Answered correctly "-.::.; TimeSpent l.!!!.I Last Updated •
27

Block Time Remaining: 00:44:10


28
29 TIMEDTUTOR

02: 00 �
EN • @: · •) .,>II 1•• T• l/vl l/1 •
1M
1 •
=: ltem _1 7 of32
:. ? 61 � � , � 0
Question Id: 4510 - ?Mark <J Previous Next
[:> Tutorial Lab Values Notes Calculator Reverse Color Text Zoom

G]
4
Clinical features of acute opioid intoxication

5 • Decreased mental status


6
• Decreased respiratory rate & shallow breaths
7
8 • Miosis (normal/enlarged if co-ingestions)
Signs & symptoms
9 • Bradycardia
10 • Decreased bowel sounds
11
• Hypothermia (or normothermia)
12
13 • Arterial blood gas (respiratory acidosis)
14 • Fingerstick blood glucose (hypoglycemia)
15 Workup
• Evaluate for presence of other drugs (eg, acetaminophen)
16
• ECG for prolonged QTc with methadone overdose
18
• Naloxone, may need repeated doses
19
Treatment • Airway management & ventilation
20
21
• Consider continuous cardiac mon itoring (if QTc >500 msec)
22
This patient with altered mental status, hypothermia, bradypnea, and hypoxia most likely has opioid intoxication
23
24 (01). The most reliable and predictive sign of 01 is a decreased respiratory rate. Other evidence includes
25 decreased bowel sounds and hypotension. Hypothermia results from environmental exposure and impaired
26 thermogenesis, and can occur even at room temperature in severely intoxicated patients. In addition, although
27
11,1 •- ·1 1 • "J!■l!;:s.. !I - I - � • -
1 , - II ••HJ,,. I - � "". • I ••HI-■
Block Time Remaining: 00:44:10
28
29 TIMEDTUTOR

02: 00 �
EN • @: · •) . ,>II 1•• T• l/vl l/1 •
1M
1 �
=: ltem 17of32
_ ? 61 � � , � 0
Question Id: 4510
:.
- ? Mark <J
Previous Next
[:> Tutorial Lab Values Notes Calculator Reverse Color Text Zoom

G]
4
I I I I • YP yp p • p •

thermogenesis, and can occur even at room temperature in severely intoxicated patients. In addition, although
5
many patients with 01 have miosis, its absence does not exclude the diagnosis. Normal or even enlarged pupils
6
7
may be seen in patients who have co-exposures to other agents that can counteract miosis (eg,
8 methamphetamine).
9
Management of patients with 01 should focus on airway protection and improving ventilation. The prompt
10
11
administration of naloxone can result in rapid improvements in respiration in both apneic and bradypneic patients,
12 thereby reducing the need for more invasive interventions. Naloxone should be titrated to achieve a respiratory
13 rate >12/min but not to achieve normal mental status.
14
(Choice A) Buprenorphine is an alternate opioid agonist used in long-term management of opioid addiction, but it
6 is not useful in acute intoxication.

(Choice B) A CT scan would be most appropriate for evaluating head trauma, intracranial bleeding, or stroke.
18
19
This patient's presentation is more consistent with 01 than any of these conditions. Due to his altered mental
20 status and significant hypoxia and bradypnea, the patient should not be transported out of the care area until
21 further stabilized (eg, airway secured).
22
23 (Choice C) Epinephrine may be used in cardiac arrest, septic shock, and anaphylaxis, but it has no role in the
24 treatment of 01.
25
(Choice D) External rewarming should be considered if the patient fails to respond to naloxone, but it otherwise


26
27 would not be required as hypothermia normal ly improves spontaneously if 01 is reversed.
28
29
Block Time Remaining: 00:44:10
TIMEDTUTOR � Feedback
©
Suspend End Block

.... r§1
...- �
EN � @: o j> .,>II 1•• 02: 00 �
T• l/vl l/1 •
1M
1 •
=: ltem 17of32
_ ? 61 � � , � 0
Question Id: 4510
:.
- ?
Mark <J
Previous Next
[:> Tutorial Lab Values Notes Calculator Reverse Color Text Zoom

G]
4 (Choice C) Epinephrine may be used in cardiac arrest, septic shock, and anaphylaxis, but it has no role in the

5 treatment of 01.
6
7 (Choice 0) External rewarming should be considered if the patient fails to respond to naloxone, but it otherwise
8 would not be required as hypothermia normal ly improves spontaneously if 01 is reversed.
9
10 (Choice E) Glucagon is administered in cases of beta blocker intoxication, which presents with bradycardia and
11 hypotension but not respiratory depression.
12
(Choice F) Benzodiazepines may contribute to respiratory depression, especially when ingested with other
13
14 substances (eg, alcohol), but should not be administered as a treatment for altered mental status or bradypnea.
15
(Choice H) Although a urine toxicology screen should be performed to evaluate for other substance exposures,
16
treatment of the patient' s respiratory depression should not be delayed to send this test or await its results.
18 Educational objective:
19
Decreased respiratory rate is the most reliable and predictive sign of opioid intoxication, which also frequently
20
presents with altered mental status, hypothermia, and miosis (which may be absent due to co-ingestions).
21
22 Management should focus on airway protection, improving ventilation, and prompt administration of naloxone.
23
24 References
25


26 • Illicit opioid intoxication: diagnosis and treatment.
27 •
28
Block Time Remaining: 00:44:10
TIMEDTUTOR � Feedback
©
Suspend End Block

..-••.
29

r§1 EN . @: o j> .,>II 1•• 02: 00 �


� T• l/vl l/1 •
1M
1 •
=: ltem 1 8 of32
_ ? 61 � � , � 0
Question Id: 3827
:.
- ?Mark <J
Previous Next
[:> Tutorial Lab Values Notes Calculator Reverse Color Text Zoom

G]
4 A 4-year-old boy is brought to the emergency department with nausea, vomiting, diarrhea, and abdominal pain

5
over the past hour. The child's grandmother found pills scattered on the kitchen floor next to an open, unlabeled
6
bottle several hours before the onset of these symptoms. His temperature is 37.2° C (99° F), blood pressure is
7
8 80/40 mm Hg, pulse is 140/min, and respirations are 50/min. During examination, the boy develops hematemesis.
9 Abdominal x-ray reveals several small opacities in the stomach and duodenum. Laboratory results are as follows:
10
11 Complete blood count
12
Hemoglobin 13.5 g/dL
13
14 Hematocrit 42%

Platelets 300,000/µL

6
17 Leukocytes 15,800/µL

19
Serum chemistry
20
21 Sodium 135 mEq/L
22
23 Potassium 3.5 mEq/L
24
Chloride 102 mEq/L
25
26 Bicarbonate 1 6 mEq/L
27
= ••• . ..-. � ... .
Block Time Remaining: 00:44:07
28
29 TIMEDTUTOR

02: 00 �
EN • @: · •) .,>II 1•• T• l/vl l/1 •
1M
1 •
=
-
Item 1 8 of32
Question Id: 3827
_
� ?
Mark <J
Previous
C>
Next
? 61 � � , � 0
- - -- · - - -
Tutorial Lab Values N�tes Calculator Reverse Color Text zoom

G] •
4 Chloride 102 mEq/L
5
6 Bicarbonate 16 mEq/L
7
Blood urea nitrogen 14 mg/dl
8
9 Creatinine 0.3 mg/dl
10
11
Glucose 118 mg/dl
12
13 Arterial blood gases
14
pH 7.3

� 90 mm Hg
6
17
29 mm Hg
19
20 Which of the following was most likely ingested?
21
22 0 A. Acetaminophen
23
24
O B. Aspirin
25 O C. Iron


26
27 Q D. Lead •
28
Block Time Remaining: 00:43:41
TIMEDTUTOR � Feedback
©
Suspend End Block

..-....
29

r§1 EN . @: ,j, .,,ii 1•• 02: 00 �


� T• l/vl l/1 •
1M
1 •
=
-
Item 1 8 of32
Question Id: 3827
_
� ?
Mark <J
Previous
C>
Next
?
Tutorial
61
Lab Values

N�tes

Calculator
,
Reverse Color
� 0
Text zoom

G]4
7.3

5 90 mm Hg
6
7 29 mm Hg
8
9 Which of the following was most likely ingested?
10
11 0 A. Acetaminophen
12
13 0 B. Aspirin
14
0 C. Iron

6 0 D. Lead
17
0 E. Vitamin A
19
20
0 F. Vitamin 82
21 0 G. Vitamin D
22
23 0 H. Vitamin K
24
25


26 Submit
27 •
28
Block Time Remaining: 00:43:38
TIMEDTUTOR � Feedback
©
Suspend End Block

..-....
29

r§1 EN . @: ,j, .,,ii 1•• 02: 00 �


� T• l/vl l/1 •
1M
1 •
=
-
Item 1 8 of32
Question Id: 3827
_
� ?
Mark <J
Previous
C>
Next
?
Tutorial
61
Lab Values

N�tes

Calculator
,
Reverse Color
� 0
Text zoom

G]
4
7. 3

5 90 mm Hg
6
7 29 mm Hg
8
9 Which of the following was most likely ingested?
10
11 A. Acetaminophen [6%]
12
13 B. Aspirin [37%>]
14
1' C. Iron [54%>]


6 X C D. Lead [1%>]
17
E. Vitamin A [0%>]
19
F. Vitamin 82 [0%]
20
21 G. Vitamin D [0%>]
22
23 H. Vitamin K [0%>]
24
25
26
Incorrecf[
27
-- ..- II • •I
Block Time Remaining: 00:43:34
28
29 TIMEDTUTOR

EN • @: ,j > .,,11 1••


1M
1 •
=: Item 18 of 32
_ ? 61 l:°f" � , � 0
Question Id: 3827 -
:. ?Mark <J
Previous Next
[:> Tutorial Lab Values Notes Calculator Reverse Color Text om Zo

G]
4

Iron Poisoning
5
6
• Within 30 minutes to 4 days:
7
• Abdominal pain
8
9 • Vomiting (eg, hematemesis)
10 Clinical • Diarrhea (eg, melena)
11 features • Hypotensive shock
12 • Metabolic acidosis
13 • Within 2 days: hepatic necrosis
14 • Within 2-8 weeks: pyloric stenosis


6 • Anion gap metabolic acidosis
Diagnostic
17 findings • Radiopaque pills

19
• Whole bowel irrigation
20
Treatment • Deferoxamine
21
• Supportive care for circulation, airway and breathing
22
23 ©USMLEWorld, LlC

24
This patient's symptoms and x-ray findings are highly suggestive of iron poisoning, one of the most common
25
causes of death by poisoning. When ingested in large amounts, elemental iron is corrosive to the gastrointestinal


26
27 mucosa, causing abdominal pain, nausea, vomiting, diarrhea, and hematemesis wi thin 30 minutes to 6 hours of •
28
29
Block Time Remaining: 00:43:34
TIMEDTLITOR � Feedback
©
Suspend End Block

...-
.... r§1 �
EN . @: ,j, .,,ii 1•• 02: 01 �
T• l/vl l/1 •
1M
1 •
=: ltem 1 8 of32
_ ? 61 � � , � 0
Question Id: 3827
:.
- ?Mark <J
Previous Next
[:> Tutorial Lab Values Notes Calculator Reverse Color Text Zoom

G]
4
causes of death by poisoning. When ingested in large amounts, elemental iron is corrosive to the gastrointestinal •

mucosa, causing abdominal pain, nausea, vomiting, diarrhea, and hematemesis within 30 minutes to 6 hours of
5
6
ingestion. Iron is also a potent vasodilator and toxic to cellular processes. Severely affected patients develop
7 hypotensive shock and anion-gap metabolic acidosis from poor perfusion and accumulation of lactic acid.
8 These patients may become tachypneic and develop respiratory alkalosis to compensate for the acidosis. Other
9 dangerous complications include liver necrosis, coagulopathy, seizures, and death. Patients who survive acute
10
toxicity are at risk of gastric scarring and pyloric stenosis within weeks of ingestion.
11
12 The diagnosis is confirmed by measuring serum iron levels. Iron is radiopaque, and visualization of gastric tablets
13 on abdominal x-ray further supports the diagnosis. Treatment depends on the severity of the poisoning. Whole­
14
bowel irrigation is sometimes instituted, but other methods of decontamination (eg, activated charcoal, syrup of
ipecac, gastric lavage) are not routinely recommended. Chelation therapy with intravenous deferoxamine is used

6
17 in moderate-to-severe cases and would be indicated in this patient.

(Choice A) Acute acetaminophen overdose can cause nausea and vomiting, but many patients remain
19
20
asymptomatic within 24 hours of ingestion. Acetaminophen also is not associated with acute hematemesis,
21 making this diagnosis unlikely.
22
(Choice B) Acute aspirin poisoning is characterized by tinnitus, fever, hyperpnea (respiratory alkalosis), and
23
24 metabolic acidosis. However, aspirin is not radiopaque and would not appear on abdominal x-ray, making this
25 diagnosis less likely.
26
(Choice D) Lead poisoning in children is typically a chronic process identified by routine laboratory screening, as
27

Block Time Remaining: 00:43:34


28
29 TIMEDTUTOR

EN • @: · •) .,>II 1••
1M
1 •
=: ltem 1 8 of32
_ ? 61 � � , � 0
Question Id: 3827
:.
- ?Mark <J
Previous Next
[:> Tutorial Lab Values Notes Calculator Reverse Color Text Zoom

G]
4
(Choice D) Lead poisoning in children is typically a chronic process identified by routine laboratory screening, as •
most children are asymptomatic. Children who become symptomatic may experience abdominal pain, vomiting,
5
anemia, and behavioral problems. Like iron, lead is radiopaque on x-ray. However, it is not corrosive to gastric
6
7 mucosa, making this diagnosis unlikely.
8
(Choice E) Acute overdose of vitamin A causes nausea, vomiting, and blurry vision. Chronic toxicity can result in
9
increased intracranial pressure (pseudotumor cerebri). It is not associated with hematemesis.
10
11 (Choice F) Vitamin 82 (riboflavin) is a water-soluble vitamin. Excessive amounts are usually not absorbed and
12
therefore are unlikely to cause toxicity.
13
14 (Choice G) Symptoms of acute vitamin D overdose are related to hypercalcemia and include nausea, vomiting,
confusion, polyuria, and polydipsia. Vitamin D is not known to erode the gastric mucosa and cause hematemesis,

6
making this diagnosis unlikely.
17
(Choice H) Vitamin K toxicity is very rare but can cause hemolytic anemia and hyperbilirubinemia in infants.
19
20
Educational objective:
21 Acute iron poisoning presents as abdominal pain, hematemesis, and metabolic acidosis. Iron is radiopaque, and
22 ingested tablets can be seen on x-ray. Chelation therapy with deferoxamine should be provided for moderately to
23 severely ill patients.
24
25
26 References
27 • - • •"' O,_,• •
:u,u..i., • .., • • =-•:J.OJL:
0-ai:,Jll•
= •.,
•....,u.:,

Block Time Remaining: 00:43:34


28
29 TIMEDTUTOR

EN • @: · •) .,>II 1••
1M
1 �
=: ltem 1 9 of32
_ ? 61 � � , � 0
Question Id: 2378
:.
- ?Mark <J
Previous Next
[:> Tutorial Lab Values Notes Calculator Reverse Color Text Zoom

G]
4 An 18-month-old boy is brought to the emergency department an hour after drinking liquid oven cleaner from an
5
unlocked kitchen cabinet. His parents tried to give him water and milk, but he has difficulty swallowing. The boy
6
also has blood-tinged oral secretions. His vital signs are stable. Examination shows an anxious child who is crying
7
8 and drooling. His lips and chin are swollen and erythematous. He has no strider and his breathing pattern
9 appears normal. Lungs are clear to auscultation. His shirt is covered in oven cleaner. Which of the following is
10 the best next step in management of this patient?
11
12 Q A. Barium swallow study
13
14 0 B. Clothing removal
0 C. Intravenous corticosteroids

6
17 0 D. Nasogastric feeding tube
18
0 E. Nasogastric lavage
20
21
0 F. Neutralization with vinegar
22 0 G. Upper gastrointestinal endoscopy
23
24
25 Submit
26
27

Block Time Remaining: 00:43:33


28
29 TIMEDTUTOR

EN � @: · •) .,>II 1••
1M
1 •
=: ltem 1 9 of32
_ ? 61 � � , � 0
Question Id: 2378
:.
- ?Mark <JPrevious Next
[:> Tutorial Lab Values Notes Calculator Reverse Color Text Zoom

G]
4 An 18-month-old boy is brought to the emergency department an hour after drinking liquid oven cleaner from an

5
unlocked kitchen cabinet. His parents tried to give him water and milk, but he has difficulty swallowing. The boy
6
also has blood-tinged oral secretions. His vital signs are stable. Examination shows an anxious child who is crying
7
8 and drooling. His lips and chin are swollen and erythematous. He has no strider and his breathing pattern
9 appears normal. Lungs are clear to auscultation. His shirt is covered in oven cleaner. Which of the following is
10 the best next step in management of this patient?
11
12 X@ A. Barium swallow study [0%]
13
14 � B. Clothing removal [68%]

( C. Intravenous corticosteroids [1%]



6
17 D. Nasogastric feeding tube [1°/o]
18
E. Nasogastric lavage [15%]
20
F. Neutralization with vinegar [1 %]
21
22 G. Upper gastrointestinal endoscopy [11 %]
23
24
25
lnconc_.ct II\ 4 Seconds 05/14/2018
26 (1,, 68% I==!
Corre<:t answer L!!!. Answered correctly 1..::.J TimeSpent l.!!!.I Last Updated
27

Block Time Remaining: 00:43:30


28
29 TIMEDTUTOR

EN • @: · •) .,>II 1••
1M
1 •
=: Item 19 of 32
_ ? 61 l:°f" � , � 0
Question Id: 2378 -
:. ?
Mark <J
Previous Next
[:> Tutorial Lab Values Notes Calculator Reverse Color Text om Zo

G] •
4 Caustic ingestion
5
6
Chemical burn or liquefaction necrosis resulting in:
7
8 Clinical features • Laryngeal damage: Hoarseness, stridor
9 • Esophageal damage: Dysphagia, odynophagia
10 • Gastric damage: Epigastric pain, bleeding
11
12 • Secure airway, breathing, circulation
13 • Decontamination: Remove contaminated clothing
14 & visible chemicals; irrigate exposed skin
Management
• Chest x-ray if respiratory symptoms

6
17 • Endoscopy within 24 hours
18
• Upper airway compromise
20 • Perforation
21 Complications • Strictures/stenosis (2-3 weeks)
22 • Ulcers
23 • Cancer
© USMLEWorld, LLC
24
25
Infants and toddlers are at high risk for foreign body ingestion as much of their normal behavior involves putting


26
27 objects in their mouths. Parents should be vigilant about locking up hazardous substances (eg, cleaning supplies) •
28
Block Time Remaining: 00:43:30
TIMEDTUTOR � Feedback
©
Suspend End Block

..-....
29

r§1 EN . @: • •> .,IIJ 1■a,


02: 01 �
� T• l/vl l/1 •
1M
1 •
=: ltem 1 9 of32
_ ? 61 � � , � 0
Question Id: 2378
:.
- ?Mark <J
Previous Next
[:> Tutorial Lab Values Notes Calculator Reverse Color Text Zoom

G]
4
Infants and toddlers are at high risk for foreign body ingestion as much of their normal behavior involves putting •
objects in their mouths. Parents should be vigilant about locking up hazardous substances (eg, cleaning supplies)
5
as most accidental ingestions occur at home.
6
7 This patient is experiencing orofacial inflammation and dysphagia after ingesting a caustic alkaline substance.
8
After assessing his airvvay, breathing, and circulation, the best next step is to remove contaminated clothing, as
9
persistent exposure to the caustic agent can cause ongoing damage. Patients should be hospitalized under close
10
11 supervision for any developing airvvay compromise. Upper gastrointestinal endoscopy (Choice G) is
12 recommended within 24 hours to assess the extent of the injury. The extent of the injury may not be apparent if
13 performed immediately, and delayed endoscopy increases perforation risk.
14
(Choice A) Barium swallow studies are usually not helpful in determining the extent of injury in the acute setting.
All patients with persistent dysphagia or significant esophageal burns on endoscopy should undergo barium

6
17 contrast studies 2-3 weeks after ingestion to assess for esophageal strictures or pyloric stenosis.
18
(Choice C) Steroids are not recommended given the lack of proven efficacy in preventing strictures and potential
20 increased chance of perforation.
21
(Choices D, E, and F) Any intervention that could provoke vomiting should be avoided. This includes
22
23 administration of milk, water, activated charcoal, vinegar, or nasogastric lavage, as vomiting can increase the
24 extent of injury. In addition, vinegar combined with an alkaline substance can cause an exothermic reacti on and
25 burn the mucosa, exacerbating the existing injury. During endoscopy, a nasogastric feeding tube can be placed
26
under direct visual ization; it should not be placed blindly due to the risk of perforation.
27 •
Block Time Remaining: 00:43:30
28
29 TIMEDTUTOR

EN • @: · •) .,>II 1••
1M
1 •
=: ltem 1 9 of32
_ ? 61 � � , � 0
:.
?Mark <J [:>
. . .. .
. "" .. . . .- . . . ., . " .
Question Id: 2378 - Previous Next Tutorial Lab Values Notes Calculator Reverse Color Text Zoom
-
G]
4
.. •
(Choice C) Steroids are not recommended given the lack of proven efficacy in preventing strictures and potential
5
increased chance of perforation.
6
7 (Choices D, E, and F) Any intervention that could provoke vomiting should be avoided. This includes
8
administration of milk, water, activated charcoal, vinegar, or nasogastric lavage, as vomiting can increase the
9
extent of injury. In addition, vinegar combined with an alkaline substance can cause an exothermic reacti on and
10
11 burn the mucosa, exacerbating the existing injury. During endoscopy, a nasogastric feeding tube can be placed
12 under direct visual ization; it should not be placed blindly due to the risk of perforation.
13
Educational objective:
14
The first step in managing caustic ingestions is assessing airway, breathing, and circulation. Contaminated
clothing should be removed prompt ly. Upper gastrointestinal endoscopy is the diagnostic study of choice to

6
17 evaluate the extent of injury. Attempting to neutralize the alkali with vinegar or lavage is dangerous as these
18
interventions may trigger vomiting, which may cause further mucosa! damage.

20
21 References
22
• Caustic injury of the upper gastrointestinal tract: a comprehensive review.
23
24 • Caustic ingestion in children.
25
26 • Updates in pediatric gastrointestinal foreign bodies.
27
i!a
I I.!.

Block Time Remaining: 00:43:30


28
29 TIMEDTUTOR

EN • @: · •) .,>II 1••
1M
1 �
=: ltem _20of32
:. ? 61 � � , � 0
Question Id: 3134 - ?Mark <J
Previous Next
[:> Tutorial Lab Values Notes Calculator Reverse Color Text Zoom

G]
4 A 34-year-old farmer is brought to the hospital after attempting suicide. His body and clothes are soiled with
5
vomitus. He is short of breath and appears agitated. His family reports no previous medical history or regular use
6
of prescri ption medications. His temperature is 36.6 ° C (98° F), blood pressure is 110/60 mm Hg, pulse is 50/min,
7
8 and respirations are 22/min. His oxygen saturation is 86% on room air. Examination shows watering of the eyes
9 and 1 mm pupils, bilaterally. Lung examination shows widespread rhonchi with prolonged expiration. Abdominal
10 examination shows increased bowel sounds. Neurologic examination shows muscle fasciculations. What would
11
be the most appropriate next step in management of this patient?
12
13
Q A. Obtain EKG for ORS duration
14
Q B. Remove all the clothing and wash the body

6
17
Q C. Obtain a chest x-ray
18 Q D. Obtain a head CT scan
19
Q E. Give physostigmine
21
22
23 Submit
24
25
26
27

Block Time Remaining: 00:43:29


28
29 TIMEDTUTOR

EN � @: · •) .,>II 1••
1M
1 •
=: ltem _20of32
:. ? 61 � � , � 0
Question Id: 3134 - ?Mark <JPrevious Next
[:> Tutorial Lab Values Notes Calculator Reverse Color Text Zoom

G]
4 A 34-year-old farmer is brought to the hospital after attempting suicide. His body and clothes are soiled with

5
vomitus. He is short of breath and appears agitated. His family reports no previous medical history or regular use
6
of prescri ption medications. His temperature is 36.6 ° C (98° F), blood pressure is 110/60 mm Hg, pulse is 50/min,
7
8 and respirations are 22/min. His oxygen saturation is 86% on room air. Examination shows watering of the eyes
9 and 1 mm pupils, bilaterally. Lung examination shows widespread rhonchi with prolonged expiration. Abdominal
10 examination shows increased bowel sounds. Neurologic examination shows muscle fasciculations. What would
11
be the most appropriate next step in management of this patient?
12
13
X@ A. Obtain EKG for QRS duration [11%]
14
-, B. Remove all the clothing and wash the body [52%]

6
17 C. Obtain a chest x-ray [2%]
18
D. Obtain a head CT scan [1 %]
19

1
E. Give physostigmine [32%]
21
22
23
1 ncorrec{,
l1o. 52% II\ 5 Seconds i==l 08/09/2018
orr�t an�er
24
� L!!!. Answere<I correctly TimeSpent
'-:::.J l.!!!.I Last Update<!
25
26
27

Block Time Remaining: 00:43:25


28
29 TIMEDTUTOR

EN • @: · •) .,>II 1••
1M
1 •
=: ltem _20of32
:. ? 61 � � , � 0
Question Id: 3134 - ?Mark <J
Previous Next
[:> Tutorial Lab Values Notes Calculator Reverse Color Text Zoom

G]
4
This farmer is suffering from organophosphate poisoning. In organophosphate poisoning, acetylcholinesterase is

5 rendered non-functional, leading to symptoms of cholinergic excess. The common presentation includes
6 bradycardia, miosis, bronchorrhea, muscle fasciculations, salivation, lacrimation, diarrhea and urination. To
7 counteract the effects of organophosphate poisoning, atropine should be administered immediately. It acts by
8
competing with acetylcholine at the muscarinic receptors, and would therefore reverse the patient's symptoms. Of
9
equal importance is the immediate removal of the patient' s clothing (since it is soaked with vomitus) to prevent
10
11 continued absorption of organophosphates through the skin (Choice B).
12
(Choice A) Obtaining an emergent ECG and analyzing the ORS complex is critical in a TCA overdose, as it has
13
prognostic and therapeutic significance.
14
(Choice C) This patient's widespread rhonchi are an effect of organophosphate poisoning. This symptom should

6
begin to reverse after initiation of proper treatment. At this stage, obtaining a chest x-ray before removing the
17
patient' s clothes would cause unnecessary further drug exposure.
18
19 (Choice D) There is no indication in the patient's history of any head trauma. The neurologic findings are
secondary to organophosphate poisoning. Obtaining a head CT would cost valuable time and will not provide any
21
useful information.
22
23 (Choice E) Physostigmine (acetylcholinesterase inhibitor) would worsen his condition.
24
25 Educational objective:
26 Organophosphate poisoning inhibits acetylcholinesterase, leading to symptoms of cholinergic excess including
27 brad cardia, miosis, rhonchi, muscle fasciculations, salivation, lacrimation, urination and defecation.
28
Block T....... ""'"...........". .......-.25
29 TIMEDTUTOR

EN • @: · •) .,>II 1••
1M
1 •
=: ltem _20of32
:. ? 61 � � , � 0
Question Id: 3134 - ?Mark <J
Previous Next
[:> Tutorial Lab Values Notes Calculator Reverse Color Text Zoom

G]
4
competing with acetylcholine at the muscarinic receptors, and would therefore reverse the patient's symptoms. Of

equal importance is the immediate removal of the patient' s clothing (since it is soaked with vomitus) to prevent
5
6 continued absorption of organophosphates through the skin (Choice 8).
7
(Choice A) Obtaining an emergent ECG and analyzing the ORS complex is critical in a TCA overdose, as it has
8
prognostic and therapeutic significance.
9
10 (Choice C) This patient's widespread rhonchi are an effect of organophosphate poisoning. This symptom should
11
begin to reverse after initiation of proper treatment. At this stage, obtaining a chest x-ray before removing the
12
patient' s clothes would cause unnecessary further drug exposure.
13
14 (Choice 0) There is no indication in the patient's history of any head trauma. The neurologic findings are
secondary to organophosphate poisoning. Obtaining a head CT would cost valuable time and will not provide any

6
useful information.
17
18 (Choice E) Physostigmine (acetylcholinesterase inhibitor) would worsen his condition.
19
Educational objective:
21 Organophosphate poisoning inhibits acetylcholinesterase, leading to symptoms of cholinergic excess including
22 bradycardia, miosis, rhonchi, muscle fasciculations, salivation, lacrimation, urination and defecation. Atropine
23
administration can reverse these effects. Of equal importance is the removal of any clothes (which may be
24
25
contaminated with pesticides) and washing of the skin to prevent further transcutaneous absorption.
26 Copyright© UWo�d. All rigllts reserved.
27 •
Block Time Remaining: 00:43:25
28
29 TIMEDTUTOR

EN • @: · •) .,>II 1••
1M
1 •
=: ltem _21 of32
:. ? 61 � � , � 0
Question Id: 4862 - ?Mark <J
Previous Next
[:> Tutorial Lab Values Notes Calculator Reverse Color Text Zoom

G]
4 A 67-year-old man is brought to the office by his wife due to a month of increasing forgetfulness. Over the last 2

5
weeks, she has noticed that his gait has been unsteady. The patient reports increased fatigue. He has
6
hypertension and hyperlipidemia, and is compliant with all medications. Over the last month, the patient has taken
7
8 over-the-counter stool softeners for constipation. He worked as an automobile mechanic for 30 years before
9 retiring last year. The patient smokes a pack of cigarettes daily and usually has a beer after dinner, but recently
10 started drinking home-distilled whiskey. Blood pressure is 120/80 mm Hg, and pulse is 76/min. Examination
11
shows clear lung fields and normal first and second heart sounds. The abdomen is soft and nontender without
12
organomegaly. There is weakness on dorsiflexion of bilateral wrists and feet. Pinprick sensation is reduced over
13
14 the hands and feet. Upper and lower limb deep tendon reflexes are 1+. Laboratory results are as follows:

Complete blood count



6
17 Hemoglobin 9.0 g/dL
18
19 Hematocrit 28%
20
Mean corpuscular volume 74 fl

22
23
Serum chemistry
24 Blood urea nitrogen 14 mg/dL
25
26 Creatinine 1.1 mg/dL
27

Block Time Remaining: 00:43:22


28
29 TIMEDTUTOR

EN • @: · •) .,>II 1••
1M
1 •
=: ltem _21 of32
:. ? 61 � � , � 0
Question Id: 4862 - ?Mark <J
Previous Next
[:> Tutorial Lab Values Notes Calculator Reverse Color Text Zoom

G]
4
Mean corpuscular volume 74 fl •

5
Serum chemistry
6
7 Blood urea nitrogen 14 mg/dl
8
9 Creatinine 1.1 mg/dl
10
Uric acid, serum 11 mg/dl
11
12 Which of the following underlying mechanisms is most likely associated with this patient' s clinical presentation?
13
14
0 A. Cerebral amyloid deposition

6 0 B. Degeneration of substantia nigra
17
18
0 C. Impaired amino acid metabolism
19 O D. Impaired demethylation of tetrahydrofolate
20
O E. Impaired heme synthesis
22
23
O F. Increased glycosylation of end-products
24 O G. Lacunar basal ganglia infarctions
25
26
27 Submit
Block Time Remaining: 00:43:18
28
29 TIMEDTUTOR

EN • @: · •) .,>II 1••
1M
1 •
=: ltem 21of32
_ :. ? 61 � � , � 0
Question Id: 4862 - ?
Mark <J
Previous Next
[:> Tutorial Lab Values Notes Calculator Reverse Color Text Zoom

G]4
Mean corpuscular volume 74 fl •

5
Serum chemistry
6
7 Blood urea nitrogen 14 mg/dl
8
9 Creatinine 1.1 mg/dl
10
Uric acid, serum 11 mg/dl
11
12 Which of the following underlying mechanisms is most likely associated with this patient' s clinical presentation?
13
14 A. Cerebral amyloid deposition [4%]

X� B. Degeneration of substantia nigra [3%]



6
17
C. Impaired amino acid metabolism [16%]
18
19 D. Impaired demethylation of tetrahydrofolate [20%]
20
E. Impaired heme synthesis [45%]
22
F. Increased glycosylation of end-products [6%]
23
24 G. Lacunar basal ganglia infarcti ons [3%]
25


26
27
28
Block Time Remaining: 00:43:15
TIMEDTUTOR � Feedback
©
Suspend End Block

..-....
29

r§1 EN . @: o j> .,>II 1•• 02: 02 �


� T• l/vl l/1 •
1M
1 •
=: Item _21of 32
? 61 l:°f" � , � 0
Question Id: 4862 :.
- ?
Mark <J
Previous Next
[:> Tutorial Lab Values Notes Calculator Reverse Color Text Zoom

G]
4 Lead poisoning in adults

5
6 Risk Occupational exposure (eg, lead paint,
7 factors batteries, ammunition, construction)
8
9 • Gastrointestinal (abdominal pain,
10 constipation, anorexia)
Clinical
11
features • Neurologic (cognitive deficits, peripheral
12 neuropathy)
13 • Hematologic (anemia)
14
• Anemia
Laboratory • Elevated venous lead level

6
17 findings • Elevated serum zinc protoporphyrin level
18
• Basophilic stippling on peripheral smear
19 ©UWorl d
20
This patient's presentation suggests acute lead toxicity, which can include:
22 • Gastrointestinal manifestations (eg, abdominal pain, constipation)
23
• Neuropsychiatric manifestations (eg, forgetfulness), motor and sensory neuropathies (eg, extensor
24
weakness, stocking-glove distribution)
25
• Hematologic manifestations (eg, microcytic anemia)


26
27
-. • • • -- .- •• •
28
29
Block Time Remaining: 00:43:15
TIMEDTUTOR � Feedback
©
Suspend End Block

EN . @: • •> .,IIJ 1■a,


02: 02 �
T• l/vl l/1 •
1M
1 •
=: ltem _21 of32
:. ? 61 � � , � 0
Question Id: 4862 - ?Mark <J
Previous Next
[:> Tutorial Lab Values Notes Calculator Reverse Color Text Zoom

G]
4
Nonspecific symptoms (eg, fatigue, irritability, insomnia) are common. Chronic lead exposure can also cause

hypertension and nephropathy. Diagnosis hinges on eliciting a history of exposure through occupation (eg, battery
5
6 manufacturing, plumbing, home restoration) or other means (eg, distillation of alcohol through parts with lead
7 soldering).
8
9
Lead is primarily absorbed via the respiratory tract in adults but may otherwise be absorbed through the skin or
10 gastrointestinal tract. It is 99% bound to erythrocytes and can disrupt heme synthesis to cause microcytic
11 anemia. Diagnosis is made via blood lead levels, complete blood count with blood smear, and x-ray fluorescence
12 to measure bone lead concentration. Peripheral blood smear can show basophilic stippling. Hyperuricemia (due
13
to impaired purine metabolism) is also typical.
14
(Choices A and B) Cerebral amyloid deposition is the pathogenic mechanism of Al zheimer disease. It is

6 associated with impairment of memory and executive function and insidious neuropsychiatric disturbances but with
17
an otherwise normal gait and neurological examination, especially in the early stages. Neuronal degeneration in
18
19
the substantia nigra is responsible for Parkinson disease, characterized by bradykinesia, resting "pill-rolling"
20 tremor, rigidity, shuffling gait, and postural instability. Neither is usually associated with peripheral neuropathy,
hyperuricemia, or microcytic anemia.
22
23
(Choices C and D) Thiamine (vitamin 8,) deficiency is seen with alcohol abuse or white-rice/cereal-based diets
24 and manifests as Wernicke-Korsakoff syndrome (eg, nystagmus, ophthalmoplegia, ataxia, confabulation) or
25 beriberi (eg, distal and symmetric peripheral neuropathy, cardiomyopathy). Cobalamin (vitamin 812) deficiency
26
(from pernicious anemia, gastrectomy, or intestinal malabsorption) leads to macrocytic anemia and subacute
27
� - ·- --
Block Time Remaining: 00:43:15
28
29 TIMEDTUTOR

02: 02 �
EN • @: · •) .,>II 1•• T• l/vl l/1 •
1M
1 •
=: ltem 21of32
_ :. ? 61 � � , � 0
Question Id: 4862 - ?
Mark <J
Previous Next
[:> Tutorial Lab Values Notes Calculator Reverse Color Text Zoom

G]4
tremor, rigidity, shuffling gait, and postural instability. Neither is usually associated with peripheral neuropathy, •

hyperuricemia, or microcytic anemia.


5
6 (Choices C and D) Thiamine (vitamin B,) deficiency is seen with alcohol abuse or white-rice/cereal-based diets
7 and manifests as Wernicke-Korsakoff syndrome (eg, nystagmus, ophthalmoplegia, ataxia, confabulation) or
8
beriberi (eg, distal and symmetric peripheral neuropathy, cardiomyopathy). Cobalamin (vitamin 812) deficiency
9
10 (from pernicious anemia, gastrectomy, or intestinal malabsorption) leads to macrocytic anemia and subacute
11 combined degeneration (eg, spasticity, incontinence, paraplegia). Neither deficiency is usually associated with
12 microcytic anemia or hyperuricemia.
13
14 (Choice F) Diabetes leads to glycosylation of many end-products, increasing the risk of cardiovascular disease,
nephropathy, retinopathy, and neuropathy (eg, peripheral neuropathy, gastroparesis). It typically does not cause

6 prominent forgetfulness, gait unsteadiness, microcytic anemia, or hyperuricemia.
17
18 (Choice G) Recurrent lacunar basal ganglia infarcts lead to subcortical vascular dementia, which can cause gait
19 disturbance, unsteadiness, and mild memory impairment. However, it is often characterized by a stepwi se decline
20 with focal motor deficits and sometimes personality changes.

22 Educational objective:
23 Lead toxicity is usually due to lead exposure at work or home. Patients may present with nonspecific (eg, fatigue,
24 irritability, insomnia), neuropsychiatric (eg, neurocogn itive deficits, peripheral neuropathy), or gastrointestinal
25
manifestations.


26
27 •
28
Block Time Remaining: 00:43:15
TIMEDTUTOR � Feedback
©
Suspend End Block

..-....
29

r§1 EN . @: o j> .,>II 1•• 02: 02 �


� T• l/vl l/1 •
1M

= Item 21 of32
1
- Question Id: 4862
_
� ? Mark <J
Previous
C>
Next
?
Tutorial
61
Lab Values
i:;?,,
N�tes

Calculator
,
Reverse Color

Text zoom
0
G]
4
• - • • .. . • . .. . - ' - • • • • -- · -t- • .. • - - • • • • •• f.:t . • •• • • -.. - . . , •• -
.. - -

5
6
7
8
9
10
11
12
13
14


6
17
18
19
20

22
23
24
25 IR::ic:nnhilir c:tinnlinaI
26 Zoom In Zoom Out Reset Add To Flash Card
27
• - .. • a,c �
- ��- • • •• • •• - •• •• •• • .. •• - • - • j.-
-
■I -a a .. ..,-,,�,••�- 0 0 - .. - --
Block Time Remaining: 00:43:15
28
29 TIMEDTUTOR

02: 02 �
EN � @: · •) .,>II 1•• T• l/vl l/1 •
1M
1 •
=: ltem 22of32
_ :. ? 61 � � , � 0
Question Id: 4487 - ?
Mark <J
Previous Next
[:> Tutorial Lab Values Notes Calculator Reverse Color Text Zoom

G]
4 A 45-year-old woman is brought to the emergency department (ED) by ambulance after a neighbor found her to be

5
in a somewhat obtunded state with an empty bottle of medications next to her. While en route to the ED, she had a
6
tonic-clonic seizure. Her medical history is significant for major depression and chronic back pain. Temperature is
7
8 38.3 C (100.9 F), blood pressure is 90/70 mm Hg, pulse is 120/min, and respirations are 16/min. She is able to
9 respond to painful stimuli. Pupils are 8 mm in size bilateral ly. Skin is warm and flushed. Lungs are clear to
10 auscultation. Abdomen is soft and nontender. Bowel sounds are decreased. There is no neck stiffness. ECG
11
shows QRS duration of 130 msec, a change from her previous ECG a month ago. Which of the following is the
12
most appropriate next step in management?
13

0 A. Lumbar puncture
14


6
0 B. Echocardiogram
17
18 0 C. Thyroid function tests
0 D. Calcium gluconate
19
20
21
0 E. Magnesium sulfate
23 0 F. Sodium bicarbonate
0 G. Atropine and pralidoxime
24
25

0 H. Naloxone


26
27
28
Block Time Remaining: 00:43:14
TIMEDTUTOR � Feedback
©
Suspend End Block

..-.
29

r§1 EN . @: o j> .,>II 1•• 02: 02 �


� T• l/vl l/1 •
1M
1 •
=: ltem _22of32
:. ? 61 � � , � 0
Question Id: 4487 - ?Mark <J Previous Next
[:> Tutorial Lab Values Notes Calculator Reverse Color Text Zoom

G]
4
tonic-clonic seizure. Her medical history is significant for major depression and chronic back pain. Temperature is •
38.3 C (100.9 F), blood pressure is 90/70 mm Hg, pulse is 120/min, and respirations are 16/min. She is able to
5
respond to painful stimuli. Pupils are 8 mm in size bilateral ly. Skin is warm and fl ushed. Lungs are clear to
6
7 auscultation. Abdomen is soft and nontender. Bowel sounds are decreased. There is no neck stiffness. ECG
8 shows QRS duration of 130 msec, a change from her previous ECG a month ago. Which of the following is the
9 most appropriate next step in management?
10
11 A. Lumbar puncture [0%]
12
13 B. Echocardiogram [0%]
14
X@' C. Thyroid function tests [0%]

� D. Calcium gluconate (13%]


6
17
18 E. Magnesium sulfate (8°/o]
19
F. Sodium bicarbonate [58%]
20
21 G. Atropine and pralidoxime [8%]

23 H. Naloxone (9%]
24
25
26
Incorrect
(1,, 58% I• : I:
L!!!. Answered correctl
27
II • •I
Block Time Remaining: 00:43:10
28
29 TIMEDTUTOR

02: 02 �
EN • @: · •) .,>II 1•• T• l/vl l/1 •
1M
1 •
=: ltem 22of32
_ :. ? 61 � � , � 0
Question Id: 4487 - ?
Mark <J
Previous Next
[:> Tutorial Lab Values Notes Calculator Reverse Color Text Zoom

G]
4
This patient's history of depression, coupled with her presenting symptoms, suggests she has most likely •

overdosed on tricyclic antidepressants (TCAs). A TCA overdose can cause hyperthermia as well as other
5
6
anticholinergic effects, including dilated pupils, skin flushing, and intestinal ileus (eg, decreased bowel sounds).
7 It can also cause seizures. TCA overdose blocks the cardiac fast sodium channels and decreases myocardial
8 conduction velocity, leading to QRS prolongation and risk of ventricular arrhythmia. Other manifestations of
9 cardiac toxicity include TCA-induced hypotension, which can be refractory to management and is the major cause
10
of mortality in TCA overdose.
11
12 The first priority when managing a patient with TCA overdose is to secure the airway, breathing, and circulation. In
13 cases of cardiac toxic ity-as evidenced by hypotension, QRS prolongation (eg, >100 msec), or ventricular
14
arrhythmia-sodium bicarbonate is administered. Sodium bicarbonate improves systolic blood pressure, narrows
the QRS complex, and decreases the incidence of ventricular arrhythmia. The mechanism of action is thought to

6
17 be due in part to effects on pH, as the neutral form of the TCA is less available to bind sodium channels.
18
(Choice A) Lumbar puncture would be an appropriate diagnostic test if meningitis is suspected. Although the
19
20
patient is obtunded, she has no neck stiffness, and her presentation is more consistent with TCA toxicity given the
21 dilated pupils and ORS prolongation.

(Choice B) This patient's cardiac findings are due to TCA overdose and can be reversed with sodium bicarbonate
23
24 administration. An echocardiogram would not be appropriate in this setting and would delay appropriate
25 management.


26
(Choice C) Severe hypothyroidism can cause myxedema coma which, like TCA overdose, has systemic effects,
27
28
29
Block Time Remaining: 00:43:10
TIMEDTUTOR � Feedback
©
Suspend End Block

.... r§1
...- �
EN . @: o j> .,>II 1•• 02: 02 �
T• l/vl l/1 •
1M
1 •
=: ltem 22of32
_ :. ? 61 � � , � 0
Question Id: 4487 - ?
Mark <J
Previous Next
[:> Tutorial Lab Values Notes Calculator Reverse Color Text Zoom

G]
4
(Choice C) Severe hypothyroidism can cause myxedema coma which, like TCA overdose, has systemic effects, •

including central nervous system depression and hypotension. However, unlike TCA overdose, myxedema causes
5
6
hypothermia and bradycardia.
7 (Choice 0) Calcium gluconate is cardioprotective in cases of hyperkalemia. Hyperkalemia presents on ECG as
8
peaked T waves, followed by lengthening of the PR and QRS intervals, eventually resulting in a sine wave.
9
10 (Choice E) Magnesium sulfate is used in patients who have torsades de pointes induced by a prolonged QT
11
interval.
12
13 (Choice G) Atropine and pralidoxime are the proper antidotes for anticholinesterase toxicity, as seen in
14 organophosphate poisoning. Patients present with cholinergic toxicity characterized by bradycardia, miosis, and
salivation.

6
17 (Choice H) Naloxone reverses opioid intoxication, which would cause miosis (rather than mydriasis) and is less
18 likely to cause hyperthermia, skin fl ushing, or ORS prolongation as compared to TCA overdose.
19
20 Educational objective:
21 Tricyclic antidepressant (TCA) overdose is characterized by central nervous system depression, hypotension, and
other anticholinergic effects including dilated pupils, hyperthermia, and intestinal ileus. It can also cause QRS
23
prolongation, increasing the risk of ventricular arrhythmias. Management of patients suspected of TCA overdose
24
should start with securing the airway, breathing, and circulation; in patients with evidence of cardiac toxicity, sodium
25
bicarbonate should then be administered to improve blood pressure, narrow the QRS interval, and prevent


26
27 arrhythmia. •
28
29
Block Time Remaining: 00:43:10
TIMEDTUTOR � Feedback
©
Suspend End Block

.... r§1
...- �
EN . @: o j> .,>II 1•• 02: 02 �
T• l/vl l/1 •
1M
1 �
=: ltem 23of32
_ :. ? 61 � � , � 0
Question Id: 2664 - ?
Mark <J
Previous Next
[:> Tutorial Lab Values Notes Calculator Reverse Color Text Zoom

G]
4 A 32-year-old woman is brought to the emergency department by her husband due to slurred speech and difficulty
5
walking. The patient has also been uncharacteristically drowsy for the past several hours. She has a history of
6
bipolar disorder, insomnia, migraine headaches, seizures, and hypothyroidism. Temperature is 37 C (98.6 F),
7
8 blood pressure is 110/70 mm Hg, pulse is 76/min, and respirations are 12/min. The patient is lethargic and falls
9 asleep during the interview and physical examination. Pupils are 3 mm and reactive to light. The neck is supple
10 and the oropharynx is clear. Chest auscultation is unremarkable. The abdomen is soft and nontender; bowel
11
sounds are normal. Limited neurologic examination shows 2+ deep tendon reflexes in all 4 extremities; there is no
12
Babinski sign and strength is 5 on a scale of 0-5 throughout. No nystagmus or hand tremor is present. Blood
13
14 glucose is 130 mg/dL. Which of the following is the most likely cause of this patient's symptoms?


6 0 A. Benzodiazepine overdose
17
18
0 B. Lithium toxicity
19 0 C. Opioid intoxication
20
21 0 D. Phenytoin toxicity
22
0 E. Serotonin syndrome
24
25
Submit


26
27
28
Block Time Remaining: 00:43:09
TIMEDTUTOR � Feedback
©
Suspend End Block

..-.
29

02: 02 �
r§1 �
EN � @: o j> .,>II 1•• T• l/vl l/1 •
1M
1 •
=: ltem 23of32
_ :. ? 61 � � , � 0
Question Id: 2664 - ?
Mark <J
Previous Next
[:> Tutorial Lab Values Notes Calculator Reverse Color Text Zoom

G]
4 A 32-year-old woman is brought to the emergency department by her husband due to slurred speech and difficulty

5
walking. The patient has also been uncharacteristically drowsy for the past several hours. She has a history of
6
bipolar disorder, insomnia, migraine headaches, seizures, and hypothyroidism. Temperature is 37 C (98.6 F),
7
8 blood pressure is 110/70 mm Hg, pulse is 76/min, and respirations are 12/min. The patient is lethargic and falls
9 asleep during the interview and physical examination. Pupils are 3 mm and reactive to light. The neck is supple
10 and the oropharynx is clear. Chest auscultation is unremarkable. The abdomen is soft and nontender; bowel
11
sounds are normal. Limited neurologic examination shows 2+ deep tendon reflexes in all 4 extremities; there is no
12
Babinski sign and strength is 5 on a scale of 0-5 throughout. No nystagmus or hand tremor is present. Blood
13
14 glucose is 130 mg/dL. Which of the following is the most likely cause of this patient's symptoms?

1' A. Benzodiazepine overdose [71 %]



6
17
B. Lithium toxicity [13%]
18
19 X � C. Opioid intoxication [8%>]
20
21 D. Phenytoin toxicity [5%]
22
E. Serotonin syndrome [0%]
24
25
lncor�ct


(1,, 7 1% II\ 4 Seconds � 09/23/2018
26
Corre<:t answer L!!!. Answere<l correctly '-::J TimeSpent Last Update<l
l.!!!.I
27 •
28
Block Time Remaining: 00:43:06
TIMEDTUTOR � Feedback
©
Suspend End Block

..-.
29

r§1 EN . @: o j> .,>II 1•• 02: 03 �


� T• l/vl l/1 •
1M
1 •
=: ltem _23of32
:. ? 61 � � , � 0
Question Id: 2664 - ?Mark <J
Previous Next
[:> Tutorial Lab Values Notes Calculator Reverse Color Text Zoom

G]
4

5
This patient's slurred speech, unsteady gait, and drowsiness are consistent with a benzodiazepine overdose.
6 Benzodiazepines are used for the treatment of anxiety, insomnia, and seizures and for alcohol withdrawal. In the
7 absence of coingestion, benzodiazepine intoxication typically presents with normal vital signs and pupillary
8 size (2-4 mm in bright light). Respiratory depression may develop when coingestion (eg, opioids, alcohol) has
9
occurred or when benzodiazepines are administered intravenously.
10
11 (Choice B) Lithium toxicity presents with tremor, hyperreflexia, ataxia, seizures, vomiting, and diarrhea.
12
13 (Choice C) In addition to sedation, opioid intoxication presents with respiratory depression and miosis.
14
(Choice D) Phenytoin toxicity presents with horizontal nystagmus, cerebellar ataxia, and confusion.
(Choice E) Serotonin syndrome presents with hypertension, tachycardia, hyperreflexia, clonus, and agitation in

6
17 the setting of serotonergic drug use (eg, coprescription of antidepressant and triptan). This patient's slurred
18 speech and overall presentation is better explained by benzodiazepine overdose.
19
20 Educational objective:
21 Slurred speech, unsteady gait, and drowsiness can be seen in the overdose of multiple drugs. Benzodiazepine
22
overdose can be distinguished from opioid overdose by the lack of severe respiratory depression and the lack of
pupillary constri ction.
24
25
26 References
27

Block Time Remaining: 00:43:06


28
29 TIMEDTUTOR

02: 03 �
EN • @: · •) .,>II 1•• T• l/vl l/1 •
1M
1 �
=: ltem 24of32
_ ? 61 � � , � 0
Question Id: 4035
:.
- ?
Mark <J
Previous Next
[:> Tutorial Lab Values Notes Calculator Reverse Color Text Zoom

G]
4 A 9-year-old boy is rushed to the emergency department after having a seizure while vi siting his grandparents for
5
the weekend. His past medical history is insignificant. When the grandmother is questioned about any
6
medications, she replies, "He isn' t taking any medications, but I take nortri ptyline and my husband takes atenolol
7
8 and enalapril." The boy's blood pressure is 80/40 mm Hg and pulse is 90/min. Physical examination shows dry
9 oral mucosa and dilated pupils. On questioning, his answers are adequate but delayed. Electrocardiogram shows
10 wide ORS complexes. What is the best next step in management of this patient?
11
12
0 A. Observation
13
14 0 B. Phenytoin
0 C. Physostigmine

6
17 0 D. Propranolol
O E. Sodium bicarbonate
18
19
20
21
22 Submit
23

25


26
27
28
Block Time Remaining: 00:43:05
TIMEDTUTOR � Feedback
©
Suspend End Block

..-.
29

02: 03 �
r§1 �
EN � @: o j> .,>II 1•• T• l/vl l/1 •
1M
1 •
=: ltem 24of32
_ ? 61 � � , � 0
Question Id: 4035
:.
- ?Mark <J
Previous Next
[:> Tutorial Lab Values Notes Calculator Reverse Color Text Zoom

G]
4 A 9-year-old boy is rushed to the emergency department after having a seizure while vi siting his grandparents for

5
the weekend. His past medical history is insignificant. When the grandmother is questioned about any
6
medications, she replies, "He isn' t taking any medications, but I take nortri ptyline and my husband takes atenolol
7
8 and enalapril." The boy's blood pressure is 80/40 mm Hg and pulse is 90/min. Physical examination shows dry
9 oral mucosa and dilated pupils. On questioning, his answers are adequate but delayed. Electrocardiogram shows
10 wide ORS complexes. What is the best next step in management of this patient?
11
12 X@ A. Observation [4%>]
13
14 B. Phenytoin [1 %]

C. Physostigmine [15%]

6
17 D. Propranolol [0%>]
18
E. Sodium bicarbonate [77%]

I
19
20
21
22
lncorre� II\ 3 Seconds 10/31/2018
(1,, 77% I==!
23 Corre<:t answer L!!!. Answered correctly "-::.J TimeSpent l.!!!.I Last Updated
E


25
26
Explanation
27
28
Block Time Remaining: 00:43:03
TIMEDTUTOR � Feedback
©
Suspend End Block

..-.
29

r§1 EN . @: , j, .,,11 1•• 02: 03 �


� T• l/vl l/1 •
1M
1 •
=: Item _24of 32
? 61 l:°f" � , � 0
Question Id: 4035 :.
- ?
Mark <J
Previous Next
[:> Tutorial Lab Values Notes Calculator Reverse Color Text Zoom

G]
4 Clinical features & management of tricyclic antidepressant overdose

5
6 Central • Mental status changes (eg, drowsiness,
7 nervous delirium, coma)
8
system • Seizures, respiratory depression
9
10 • Sinus tachycardia, hypotension
Clinical
11
presentation Cardiovascular • Prolonged PR/QRS/QT intervals
12 • Armythmias (eg, ventricular tachycardia,
13 fibrillation)
14
• Dry mouth, blurred vision, dilated pupils
Anticholinergic

6 • Urinary retention, flushing, hyperthermia
17
18 • Supplemental oxygen, intubation
19 • Intravenous fluids
20 Supportive • Activated charcoal for patients within 2
Management
21 care & therapy hours of ingestion (unless ileus present)
22 • Intravenous sodium bicarbonate for QRS
23 widening or ventricular arrhythmia
©uwo�d.com
25
This patient's presentation suggests tricyclic antidepressant (TCA) overdose. Patients with TCA overdose can


26
27 develop central nervous system findings, including mental status changes (eg, drowsiness, delirium, coma), •
28
29
Block Time Remaining: 00:43:03
TIMEDTUTOR � Feedback
©
Suspend End Block

...-
.... r§1 �
EN . @: • •> .,IIJ 1■a,
02: 03 �
T• l/vl l/1 •
1M
1 �
=: ltem 24of32
_ :. ? 61 � � , � 0
Question Id: 4035 - ?
Mark <J
Previous Next
[:> Tutorial Lab Values Notes Calculator Reverse Color Text Zoom

G]
4

.l • • .. •

develop central nervous system findings, including mental status changes (eg, drowsiness, delirium, coma),
5
seizures, and respiratory depression. Anticholinergic effects include dry mouth, blurred vision, dilated pupils,
6
7
fl ushing, hyperthermia, and urinary retention. Cardiotoxic effects (eg, sinus tachycardia, hypotension, ventricular
8 arrhythmias) cause most of the mortality in patients with TCA overdose.
9
The first steps in management of this pediatric emergency are the ABCs (airway, breathing, circulation). Patent
10
11
airways and adequate breathing should be assured, and cardiac monitors, oximetry, and intravenous lines should
12 be established. QRS interval >100 msec in TCA overdose is an indication for sodium bicarbonate therapy.
13 Sodium bicarbonate alleviates the inhibitory action of TCAs on fast sodium channels of the myocardium, improving
14 hypotension and decreasing the risk of fatal ventricular arrhythmias (by shortening the ORS interval).

(Choice A) Observation is not appropriate as the widened ORS interval seen on electrocardiogram puts the

6
17 patient at risk for fatal ventricular arrhythmias.
18
19
(Choice B) Seizures caused by TCA overdose are due to inhibition of GABA receptors. Therefore, they are
20 treated with GABA agonists such as benzodiazepines rather than sodium channel-blocking agents such as
21 phenytoin. Phenytoin is also associated with hypotension and cardiac arrhythmias and should not be used in this
22 patient.
23
(Choice C) Physostigmine is used to reverse life-threatening delirium caused by anticholinergic agents (eg,
25 atropine, diphenhydramine). Despite prominent anticholinergic toxicity seen in TCA overdose, physostigmine is


26
not used in these patients due to the risk of cardiac arrest.
27 •
28
Block Time Remaining: 00:43:03
TIMEDTUTOR � Feedback
©
Suspend End Block

..-.
29

02: 03 �
r§1 �
EN � @: o j> .,>II 1•• T• l/vl l/1 •
1M
1 •
=: ltem 24of32
_ ? 61 � � , � 0
Question Id: 4035
:.
- ?
Mark <J
Previous Next
[:> Tutorial Lab Values Notes Calculator Reverse Color Text Zoom

G]
4
treated with GABA agonists such as benzodiazepines rather than sodium channel-blocking agents such as

phenytoin. Phenytoin is also associated with hypotension and cardiac arrhythmias and should not be used in this
5
6 patient.
7
(Choice C) Physostigmine is used to reverse life-threatening delirium caused by anticholinergic agents (eg,
8
9
atropine, diphenhydramine). Despite prominent anticholinergic toxicity seen in TCA overdose, physostigmine is
10 not used in these patients due to the risk of cardiac arrest.
11
(Choice 0) Beta blockers are not used for TCA overdose as they can worsen hypotension and cause cardiac
12
13
arrest.
14 Educational objective:
Tricyclic antidepressant (TCA) overdose can present with central nervous system, cardiac, and anticholinergic

6
17
findings. Sodium bicarbonate is used to treat cardiac toxicity, which is characterized by prolonged QRS duration
18 (>100 msec) and ventricular arrhythmias (eg, ventricular tachycardia, ventricular fibrillation). Benzodiazepines are
19 used to treat seizures due to TCA overdose.
20
21
22
References
23 • Management of the cardiovascular complications of tricyclic antidepressant poisoning: role of sodium
bicarbonate.
25


26 Copyright© UWo�d. All rigllts reserved.
27 •
28
Block Time Remaining: 00:43:03
TIMEDTUTOR � Feedback
©
Suspend End Block

..-.
29

r§1 EN . @: o j> .,>II 1•• 02: 03 �


� T• l/vl l/l •
1M
1 �
=: ltem 25of32
_ :. ? 61 � � , � 0
Question Id: 3136 - ?
Mark <J
Previous Next
[:> Tutorial Lab Values Notes Calculator Reverse Color Text Zoom

G]
4 A 24-year-old woman brought to the emergency department says that she ingested about twenty 500-mg
5
acetaminophen tablets 2 hours ago after an intense argument with her husband. She currently has no symptoms.
6
The patient does not smoke but drinks alcohol on weekends. She is alert and oriented. Her temperature is 37.2 C
7
8 (99 F), blood pressure is 110/60 mm Hg, pulse is 90/min, and respirations are 16/min. Examination shows no
9 abnormalities. Complete blood count, coagulation studies (prothrombin time/International Normalized Ratio), and
10 serum chemistries, including liver functions tests, are within normal limits. Which of the following is the most
11
appropriate next step in management of this patient?
12
13
Q A. Administer charcoal and obtain serum acetaminophen levels
14
Q B. Discharge home

6
17
Q C. Observe in the hospital for 24 hours
18 Q D. Obtain a psychiatry consultation
19
20 Q E. Obtain an electrocardiogram (ECG)
21
22
Q F. Transfer to liver transplantation facility
23
24
Submit


26
27
28
Block Time Remaining: 00:43:02
TIMEDTUTOR � Feedback
©
Suspend End Block

..-.
29

02: 03 �
r§1 �
EN � @: o j> .,>II 1•• T• l/vl l/1 •
1M
1 •
=: ltem 25of32
_ :. ? 61 � � , � 0
Question Id: 3136 - ?
Mark <J
Previous Next
[:> Tutorial Lab Values Notes Calculator Reverse Color Text Zoom

G]
4 A 24-year-old woman brought to the emergency department says that she ingested about twenty 500-mg

5
acetaminophen tablets 2 hours ago after an intense argument with her husband. She currently has no symptoms.
6
The patient does not smoke but drinks alcohol on weekends. She is alert and oriented. Her temperature is 37.2 C
7
8 (99 F), blood pressure is 110/60 mm Hg, pulse is 90/min, and respirations are 16/min. Examination shows no
9 abnormalities. Complete blood count, coagulation studies (prothrombin time/International Normalized Ratio), and
10 serum chemistries, including liver functions tests, are within normal limits. Which of the following is the most
11
appropriate next step in management of this patient?
12
13
� A. Administer charcoal and obtain serum acetaminophen levels [69%]
14
B. Discharge home [0%]

6
17
X � C. Observe in the hospital for 24 hours [20%]
18
D. Obtain a psychiatry consultation [7%>]
19
20 E. Obtain an electrocardiogram (ECG) [1%]
21
22 F. Transfer to liver transplantation facility [0%>]
23
24

lnco4ct 10/09/2018
(1,, 69% II\ 4 Seconds ¢=l
Correct answer


26 L!!!. Answere<l correctly TimeSpent
'-:::J l.!!!.I Last Update<l
27 A •
28
Block Time Remaining: 00:42:59
TIMEDTUTOR � Feedback
©
Suspend End Block

..-.
29

r§1 EN . @: o j> .,>II 1•• 02: 03 �


� T• l/vl l/1 •
1M
1 •
=: Item _25of 32
? 61 l:°f" � , � 0
Question Id: 3136 -
:. ? Mark <J
Previous Next
[:> Tutorial Lab Values Notes Calculator Reverse Color Text om Zo

G] Acetaminophen intoxication

I I
4
5 Single dose �7.5 g (pediatric ::::150 mg/kg): lNo (or indeterminate)
6
Yes
' '
I I I I
7
8 S4 hours since ingestion Chronic ingestion
9
10
Yes
11 Administer
Yes
12 activated charcoal

13
14
I Check acetaminophen levels I ·

6
• Levels above treatment line in nomogram
17
• >10 µg/ml if timing of ingestion unclear
18 • Any evidence of liver injury

19
20
Administer N -acetylcysteine
21 & monitor for liver injury
22
©UWorld
23
24 This patient was brought to the emergency department 2 hours after ingesting a potentially toxic dose (single dose
of >7.5 g) of acetaminophen. Patients at this early stage of acetaminophen intoxication can be asymptomatic
26
(first 24 hours) or may have on ly nonspecific symptoms such as nausea, vomiting, and anorexia. After 24 hours,
27

Block Time Remaining: 00:42:59


28
29 TIMEDTUTOR

02: 03 �
EN • @: · •) .,>II 1•• T• l/vl l/1 •
1M
1 �
=: ltem _25of32
:. ? 61 � � , � 0
Question Id: 3136 - ?Mark <J
Previous Next
[:> Tutorial Lab Values Notes Calculator Reverse Color Text Zoom

G]
4
p y p J ry
The initial management should be focused on gastric decontamination with activated charcoal if the patient
5

6 presents within 4 hours of ingestion. Acetaminophen levels should be obtained at the same time. The Rumack­
7 Matthew nomogram provides the likelihood of hepatotoxic effects of acetaminophen overdose after a single
8 ingestion based on plasma acetaminophen level and hours since ingestion. This tool is also used in guiding the
9
administration of N-acetylcysteine in patients with dangerous acetaminophen levels. The first data point begins at
10
4 hours. However, patients with an overdose often report incorrect times and doses; although an acetaminophen
11
12 level may not be predictive of hepatotoxicity at this stage, it is prudent to obtain one. If the level is not within the
13 toxic range by the nomogram, it should be repeated in 2 hours based on the patient's history of ingestion.
14
(Choices B and C) It would be inappropriate to discharge the patient or place her in observation only before
obtaining a current and 4-hour acetaminophen level as the reported ingestion places her at risk for hepatotoxicity.

6
17
(Choice D) A psychiatry consultation may be required but is less urgent than medical stabilization.
18
19 (Choice E) Aspirin overdoses are associated with an increased risk of arrhythmias, but acetaminophen overdoses
20 usually do not have primary cardiac manifestations; electrocardiogram (ECG) is less helpful in assessing toxicity.
21
22 (Choice F) Patients at risk for fulminant hepatic failure should be evaluated for potential liver transplantation.
23 However, this patient has no signs of toxicity and was seen soon after ingestion, making the likelihood of need for
24 transplantation relatively low.

26 Educational objective:
27 For patients who present soon after a single, potentially toxic ingestion {> 7.5 g) of acetaminophen, the first step in •
Block Ti1.._ ··-···-······u· --· -
28
-·- -

m
29 TIMEDTUTOR P,2

tr EN � @: · •) .,>II 1•• 02: 03 �


T• l/vl l/1 •
1M
1 •
=: ltem _25of32
:. ? 61 � � , � 0
Question Id: 3136 - ?Mark <J
Previous Next
[:> Tutorial Lab Values Notes Calculator Reverse Color Text Zoom

G]
4
toxic range by the nomogram, it should be repeated in 2 hours based on the patient's history of ingestion.

5 (Choices B and C) It would be inappropriate to discharge the patient or place her in observation only before
6 obtaining a current and 4-hour acetaminophen level as the reported ingestion places her at risk for hepatotoxicity.
7
8 (Choice 0) A psychiatry consultation may be required but is less urgent than medical stabilization.
9
(Choice E) Aspirin overdoses are associated with an increased risk of arrhythmias, but acetaminophen overdoses
10
11
usually do not have primary cardiac manifestations; electrocardiogram (ECG) is less helpful in assessing toxicity.
12
(Choice F) Patients at risk for fulminant hepatic failure should be evaluated for potential liver transplantation.
13
However, this patient has no signs of toxicity and was seen soon after ingestion, making the likelihood of need for
14
transplantation relatively low .


6
Educational objective:
17
For patients who present soon after a single, potentially toxic ingestion {> 7.5 g) of acetaminophen, the first step in
18
19 management is gastric decontamination and measurement of acetaminophen levels. Patients can be
20 asymptomatic during the first 24 hours after ingestion.
21
22
References
23
24 • A review of acetaminophen poisoning.

26
• Acetaminophen poisoning: an update for the intens ivist.
27
.......0 :ii••· • 1.,, 1 -_... .,_,
Block Time Remaining: 00:42:59
28
29 TIMEDTUTOR

02: 03 �
EN • @: · •) .,>II 1•• T• l/vl l/1 •
1M
1 •
=: ltem _26of32
:. ? 61 � � , � 0
Question Id: 3161 - ?Mark <J
Previous Next
[:> Tutorial Lab Values Notes Calculator Reverse Color Text Zoom

G]
4 A 34-year-old homeless man is brought to the emergency room in a confused state. He complains of epigastric

5
pain, vomiting and blurred vision. His temperature is 36.8° C (98.2° F), blood pressure is 110/60 mm Hg, pulse is
6
110/min and respirations are 22/min. Physical examination shows poor oral hygiene and dry mucous membranes.
7
8 Lungs are clear to auscultation. Abdominal exam shows mild epigastric discomfort. Funduscopic examination
9 reveals opti c disc hyperemia. Laboratory studies show:
10
11 Serum sodium 136 mEq/L
12
Serum potassium 3.0 mEq/L
13
14 Chloride 93 mEq/L

6
Bicarbonate 6 mEq/L

17 Blood urea nitrogen (BUN) 30 mg/dL
18
19 Serum creatinine 0.8 mg/dL
20
Blood glucose 80 mg/dL
21
22 Serum ketones negative
23
24 Plasma lactate 2.2 mEq/L
25
Which of the following is the most likely cause of his symptoms?
27
- .!. • -.- •
Block Time Remaining: 00:42:58
28
29 TIMEDTUTOR

02: 03 �
EN • @: · •) .,>II 1•• T• l/vl l/1 •
1M
1 •
=: ltem 26of32
_ :. ? 61 � � , � 0
Question Id: 3161 - ?
Mark <J
Previous Next
[:> Tutorial Lab Values Notes Calculator Reverse Color Text Zoom

G]4 Blood urea nitrogen (BUN) 30 mg/dL


5
6 Serum creatinine 0.8 mg/dL
7
Blood glucose 80 mg/dL
8
9 Serum ketones negative
10
11
Plasma lactate 2.2 mEq/L
12
Which of the following is the most likely cause of his symptoms?
13
14
O A. Ethylene glycol poisoning
6

O B. Mesenteric ischemia
17
18 0 C. Septic shock
19
20
O D. Diabetic ketoacidosis
21 0 E. Methanol poisoning
22
23 0 F. Aspirin overdose
24
25


Submit
27 •
28
Block Time Remaining: 00:42:54
TIMEDTUTOR � Feedback
©
Suspend End Block

..-....
29

r§1 EN . @: o j> .,>II 1•• 02: 03 �


� T• l/vl l/1 •
1M
1 •
= Item 26 of 32
- Question Id: 3161
_
� ?
Mark <J
Previous
C>
Next
?
Tutorial
61
Lab Values

N�tes

Calculator
,
Reverse Color
� 0
Text zoom

G]
4 Blood urea nitrogen (BUN) 30 mg/dL

5
6 Serum creatinine 0.8 mg/dL
7
Blood glucose 80 mg/dL
8
9 Serum ketones negative
10
11
Plasma lactate 2.2 mEq/L
12
Which of the following is the most likely cause of his symptoms?
13
14
X (i' A. Ethylene glycol poisoning [24%]

6
� B. Mesenteric ischemia [1 %]
17
18 C. Septic shock [0%]
19
D. Diabetic ketoacidosis [1%]
20
21 E. Methanol poisoning [68%)
22
23 F. Aspirin overdose [3%]
24
25

Incorrect
27 I:I• I:
II • •I
Block Time Remaining: 00:42:51
28
29 TIMEDTUTOR

02: 03 �
EN • @: · •) .,>II 1•• T• l/vl l/1 •
1M
1 •
=: ltem 26of32
_ :. ? 61 � � , � 0
Question Id: 3161 - ?
Mark <J
Previous Next
[:> Tutorial Lab Values Notes Calculator Reverse Color Text Zoom

G]
4
This homeless man's symptoms are likely the result of ingesting methanol as a subst itute for alcohol. Methanol's

immediate effects are similar to those of alcohol, causing disinhibition. However, within 24 hours of ingestion,
5
6
methanol can lead to headache, nausea, vomiting and epigastric pain. The most severe consequences of
7 methanol intoxication are vision loss and coma. Physical exam in methanol intoxication shows optic disc
8 hyperemia and laboratory studies reveal anion gap metabolic acidosis. An increased osmolar gap is often seen as
9
well. In this patient, the anion gap, calculated by the equation: (Na)-([Cl]+[HCO3]) = (136)-(93+6)=37. This value
10
is well above the normal anion gap of 8-12. Such profound metabolic acidosis and very low bicarbonate [6 mEq/L]
11
12 can be caused by methanol and ethylene glycol poisoning. There are many other causes of anion gap metabolic
13 acidosis but in the setting of blurred vision, epigastric pain, vomiting and hyperemic optic disc, this patient is most
14 likely suffering from methanol poisoning. The patient's hypokalemia is the likely result of alcoholism and vomiting.

6 (Choice A) Ethylene glycol and methanol poisoning share similarities. Both are sometimes ingested as a

17 substitute for ethanol and both can cause anion gap metabolic acidosis and increased osmolar gap. Whereas
18 methanol damages the eyes, ethylene glycol damages the kidneys.
19
20 (Choice B) Mesenteric ischemia is most common in older patients, those with a history of atherosclerotic disease
21 or in those with cardiac arrhythmia. It presents with abdominal pain out of proportion to exam (this patient has on ly
22 mild abdominal discomfort) and is not associated with blurred vision. Elevated lactate would be expected.
23
24 (Choice C) Septic shock presents with significant hypotension (systolic blood pressure <90), hypo or
25 hyperthermia, elevated white blood cell count and positive blood cultures. This patient does not meet any of the


criteria mentioned.
27
28
29
Block Time Remaining: 00:42:51
TIMEDTUTOR � Feedback
©
Suspend End Block

.... r§1
...- �
EN . @: o j> .,>II 1•• 02: 03 �
T• l/vl l/1 •
1M
1 •
=: ltem 26of32
_ :. ? 61 � � , � 0
Question Id: 3161 - ?
Mark <J
Previous Next
[:> Tutorial Lab Values Notes Calculator Reverse Color Text Zoom

G]
4
(Choice A) Ethylene glycol and methanol poisoning share similarities. Both are sometimes ingested as a

substitute for ethanol and both can cause anion gap metabolic acidosis and increased osmolar gap. Whereas
5
6
methanol damages the eyes, ethylene glycol damages the kidneys.
7
(Choice B) Mesenteric ischemia is most common in older patients, those with a history of atherosclerotic disease
8
or in those with cardiac arrhythmia. It presents with abdominal pain out of proportion to exam (this patient has on ly
9
10 mild abdominal discomfort) and is not associated with blurred vision. Elevated lactate would be expected.
11
(Choice C) Septic shock presents with significant hypotension (systolic blood pressure <90), hypo or
12
hyperthermia, elevated white blood cell count and positive blood cultures. This patient does not meet any of the
13
14 criteria mentioned.

(Choice D) Diabetic ketoacidosis causes anion gap metabolic acidosis and abdominal complaints as in this
6
� patient but presents with hyperglycemia and positive serum ketones.
17
18 (Choice F) Aspirin overdose also presents with anion gap metabolic acidosis. Tinnitus, fever and hyperventilation
19
are common.
20
21 Educational Objective:
22 The most severe consequences of methanol intoxication are vision loss and coma. Physical exam in methanol
23
intoxication reveals optic disc hyperemia while laboratory studies reveal anion gap metabolic acidosis. An
24
25
increased osmolar gap is often seen as well.


Copyright© UWo�d. All rigllts reserved.
27 •
28
Block Time Remaining: 00:42:51
TIMEDTUTOR � Feedback
©
Suspend End Block

..-.
29

r§1 EN . @: o j> .,>II 1•• 02: 04 �


� T• l/vl l/l •
1M
1 �
=: ltem _27of32
:. ? 61 � � , � 0
Question Id: 3125 - ?Mark <J
Previous Next
[:> Tutorial Lab Values Notes Calculator Reverse Color Text Zoom

G]
4 A 35-year-old woman calls her fami ly physician after her ?-year old son accidentally splashed an unknown liquid
5
containing acid on his face. Some liquid likely entered his eye, as the child is complaining of severe pain in his
6
r ight eye. She knows a doctor living in her immediate neighborhood but is hesitant about what to do next. Which
7
8 of the following is the most appropriate initial course of action that the mother should take?

O A. Call 911 immediately


9
10
11
12
O B. Go to local emergency room immediately
13 O C. Go to the doctor in the neighborhood immediately
14
O D. Wash the eye with copious amount of water

6
17
O E. Do not try to wash the eye as it can enhance the damage
18
19
Submit
20
21
22
23
24
25
26

28
Block Time Remaining: 00:42:50
29 TIMEDTUTOR

02: 04 �
EN � @: · •) .,>II 1•• T• l/vl l/1 •
1M
1 •
=: ltem 27of32
_ :. ? 61 � � , � 0
Question Id: 3125 - ?
Mark <J
Previous Next
[:> Tutorial Lab Values Notes Calculator Reverse Color Text Zoom

G]
4 A 35-year-old woman calls her fami ly physician after her ?-year old son accidentally splashed an unknown liquid

5
containing acid on his face. Some liquid likely entered his eye, as the child is complaining of severe pain in his
6
r ight eye. She knows a doctor living in her immediate neighborhood but is hesitant about what to do next. Which
7
8 of the following is the most appropriate initial course of action that the mother should take?
9
10 X@ A. Call 911 immediately (5%]
11
B. Go to local emergency room immediately [2%]
12
13 C. Go to the doctor in the neighborhood immediately [0%]
14
D. Wash the eye with copious amount of water (90%>]


6

I
E. Do not try to wash the eye as it can enhance the damage (1%]
17
18
19
lncorr� 90% II\ 5 Seconds F==I 10/05/2018
20
Corr�t answer 11 ..
L!!!. Answered correctly "-::.J TimeSpent l.!::.I Last Updated
21
D
22
23
24 Explanation


25
26
In the case of any chemical contact with the eye, the first priority is to immediately begin flushing the affected eye
28
29
Block Time Remaining: 00:42:46
TIMEDTUTOR � Feedback
©
Suspend End Block

....
...- �
EN . @: ,j, .,,II I"•
02: 04 �
T• l/vl l/1 •
1M
1 •
=: ltem 27of32
_ :. ? 61 � � , � 0
Question Id: 3125 - ?
Mark <J
Previous Next
[:> Tutorial Lab Values Notes Calculator Reverse Color Text Zoom

G] I•
Corre<:t answer
(1,, 90%
L!!!. Answere<l correctly
II\ 5 Seconds
'-::J TimeSpent
� 10/05/2018
Last Update<l
l.!!!.I

D
4
5
6
7 Explanation
8
9
In the case of any chemical contact with the eye, the first pri ority is to immediately begin fl ushing the affected eye
10
with copious amounts of running water. This can best be achieved by holding the eyelid open under a running
11
12 faucet for at least 15 minutes before evaluation. Calling the emergency room, 911, or the neighborhood doctor
13 would all be appropriate actions after initiating flushing of the eye. With exposure of the eye to acid, there is a
14 good chance of full recovery. With alkaline exposure, permanent corneal damage is more likely to result.
Regardless, flushing the eye with water is the appropriate initial course of action for both cases.

6
17 (Choices A , B, and C) In the case of a cut or scratch to the eye, or a foreign body embedded in the eye, seeking
18 immediate medical care is the proper course of action.
19
20 Educational Objective:
21 Chemicals in the eye deserve immediate attention. The best primary course of action is to flush the eye with water
22 • best achieved under a faucet of running water • for at least 15 minutes. Obtaining medical care is also
23
appropriate once this treatment is initiated. For foreign bodies or cuts/scratches of the eye, obtaining medical care
24
25
is the first pri ority.


26 Copyright© UWo�d. All rigllts reserved.


28
29
Block Time Remaining: 00:42:46
TIMEDTUTOR � Feedback
©
Suspend End Block

....
...- �
EN . @: ,j, ..,11 1•• 02: 04 �
T• l/vl l/l •
1M
1 •
=: ltem _28of32
:. ? 61 � � , � 0
Question Id: 3129 - ?Mark <J
Previous Next
[:> Tutorial Lab Values Notes Calculator Reverse Color Text Zoom

G]
4 A 19-year-old woman is brought to the emergency department following a motor vehicle collision. According to the

5
accident report, she hit a temporary guardrail erected to redirect traffic due to construction. On arrival, the patient
6
appears anxious and confused. She has no apparent head, neck, or body injuri es. When her parents arrive, they
7
8 report that she has no other medical problems but that a physician prescribed something for anxiety and insomnia
9 during a stressful period last year. Last month, the patient had her wisdom teeth extracted and experienced a lot of
10 postoperative pain that has since resolved. She occasionally drinks alcohol and smokes half a pack of cigarettes
11
daily. Temperature is 37.2 C (99 F), blood pressure is 128/75 mm Hg, pulse is 110/min, and respirations are
12
22/min. Examination shows a thin woman with dry mouth and conjunctiva! injection; the pupils are equal and
13
14 reactive to light and accommodation. She is oriented to person and place but not time and recalls 1 of 3 objects in
5 minutes. Which of the following substances is most likely contributing to this patient's presentation?

6
17 Q A. Alcohol

0 B. Benzodiazepines
18
19
20
21
0 C. Cocaine
22 0 D. Marijuana
0 E. Opioids
23
24
25
26
0 F. Phencyclidine
27 •
Block Time Remaining: 00:42:45
29

EN • 02: 04 �
@: ,j > .,,II 1•• T• l/vl l/l •
1M
1 �
=: ltem _28of32
:. ? 61 � � , � 0
Question Id: 3129 - ?Mark <J Previous Next
[:> Tutorial Lab Values Notes Calculator Reverse Color Text Zoom
■- • •
G]
4
- •- -•• - - - • • .. ' e l.. • .. - - -• • -• - ., e

appears anxious and confused. She has no apparent head, neck, or body injuries. When her parents arrive, they
I • W .. J.. - •- -

5 report that she has no other medical problems but that a physician prescribed something for anxiety and insomnia
6
during a stressful period last year. Last month, the patient had her wisdom teeth extracted and experienced a lot of
7
8
postoperative pain that has since resolved. She occasionally drinks alcohol and smokes half a pack of cigarettes
9 daily. Temperature is 37.2 C (99 F), blood pressure is 128/75 mm Hg, pulse is 110/min, and respirations are
10 22/min. Examination shows a thin woman with dry mouth and conjunctiva! injection; the pupils are equal and
11 reactive to light and accommodation. She is oriented to person and place but not time and recalls 1 of 3 objects in
12
5 minutes. Which of the following substances is most likely contributing to this patient's presentation?
13
14
A. Alcohol [1 %]


6 X@ B. Benzodiazepines [6%]
17
18 C. Cocaine [2%>]
19
D. Mar ijuana [87%]
20
21 E. Opioids [1 %]
22
23 F. Phencyclidine [1 %]
24
25
26
(1,, 87%
L!!!. Answered correctl
27
II • •I
Block Time Remaining: 00:42:39

EN � 02: 04 �
@: ,j > .,,11 1•• T• l/vl l/l •
1M
1 •
=: ltem 28of32
_ :. ? 61 � � , � 0
Question Id: 3129 - ?
Mark <J
Previous Next
[:> Tutorial Lab Values Notes Calculator Reverse Color Text Zoom

G]
4
This patient's clinical presentation and examination findings are most consistent with acute cannabis (marijuana)

5 intoxication. Signs of marijuana intoxication include tachycardia, tachypnea, dry mouth, conjunctiva!
6 injection, and increased appetite. Marijuana slows reaction time and impairs coordination, attention,
7 concentration, and short-term memory. Impairment of cognition, judgment, and coordination can last much longer
8
than the acute euphoric effect, altering the ability to operate automobiles and increasing the risk of motor vehicle
9
collisions. Patients with cannabis intoxication are managed supportively.
10
11 (Choices A and B) Alcohol and benzodiazepine intoxication can present with unsteady gait, incoordination,
12
disinhibited behavior, drowsiness, and conjunctiva! injection, but dry mouth would not be expected.
13
14 (Choice C) Cocaine intoxication is characterized by sympathetic stimulation, with an increase in heart rate and
blood pressure, diaphoresis, and dilated pupils. This response is accompanied by euphori a, a sense of self­

6
confidence, increased arousal, and improved performance on tasks of alertness and vigilance. In contrast, this
17
patient' s pupils are not dilated, and she had a slowed reaction time while driving.
18
19 (Choice E) Opioid intoxication typically presents with depressed mental status, decreased respiratory rate,
20
bradycardia, and small, constricted pupils (miosis); tachypnea and conjunctiva! injection are not seen.
21
22 (Choice F) Impaired judgment and tachycardia can also be seen in phencyclidine (PCP) intoxication. However,
23 PCP intoxication is more like ly to cause aggressive behavior, ataxia, mild hypertension, and multidirectional
24
nystagmus.
25


26 Educational objective:
27 Clinical features of mari'uana intoxication include increased a .. - • - • • - •
29
Block Time Remaining: 00:42:39
� Feedback
©
Suspend End Block

EN . @: ,j, .,,II I"•


02: 04 �
T• l/vl l/l •
1M
1 �
=: ltem _28of32
:. ? 61 � � , � 0
Question Id: 3129 - ?Mark <J
Previous Next
[:> Tutorial Lab Values Notes Calculator Reverse Color Text Zoom

G] t I = " •• • • •
4 disinhibited behavior, drowsiness, and conjunctiva! injection, but dry mouth would not be expected.
5
6 (Choice C) Cocaine intoxication is characterized by sympathetic stimulation, with an increase in heart rate and
7 blood pressure, diaphoresis, and dilated pupils. This response is accompanied by euphoria, a sense of self­
8 confidence, increased arousal, and improved performance on tasks of alertness and vigilance. In contrast, this
9
patient' s pupils are not dilated, and she had a slowed reaction time while driving.
10
11 (Choice E) Opioid intoxication typically presents with depressed mental status, decreased respiratory rate,
12 bradycardia, and small, constricted pupils (miosis); tachypnea and conjunctiva! injection are not seen.
13
14 (Choice F) Impaired judgment and tachycardia can also be seen in phencyclidine (PCP) intoxication. However,
PCP intoxication is more likely to cause aggressive behavior, ataxia, mild hypertension, and multidirectional

6 nystagmus.
17
18 Educational objective:
19 Clinical features of marijuana intoxication include increased appetite, tachycardia, tachypnea, dry mouth, and
20 conjunctiva! injection. Mar ijuana use slows reaction time; impairs attention, concentration, and coordination; and
21
can increase the risk of motor vehicle collisions.
22
23
24 References
25
• The effects of cannabis intoxication on motor vehicle collision revis ited and revised.
26
27 tion and motor vehicle collision risk: s stematic review of observational studies and
Block Time Remaining: 00:42:39
29

EN � 02: 04 �
@: ,j > .,,II 1•• T• l/vl l/l •
1M
1 •
=: ltem _29of32
:. ? 61 � � , � 0
Question Id: 3876 - ?Mark <J
Previous Next
[:> Tutorial Lab Values Notes Calculator Reverse Color Text Zoom

G]
4 A 28-year-old woman comes to the urgent care center at a ski resort due to frostbite. She dropped one of her

5
gloves while riding the chairlift and was unable to find it while skiing down the slope. By the time the patient
6
returned to the resort, she was experiencing numbness and paresthesias in her left hand. Her medical history is
7
8 unremarkable, and her only regular medication is an oral contraceptive. The patient does not smoke and drinks
9 alcohol only on social occasions. Temperature is 36. 7 C (98 F), blood pressure is 108/65 mm Hg, and pulse is
10 88/min. Examination shows stiff, pale skin at the fourth and fifth digits of the left hand. Vesicles and small bullae
11
are seen at the lateral aspect of the fingerti ps. The affected dig its show absent light-touch sensation and capillary
12
refill. Which of the following is the most appropriate next step in management of this patient's injury?
13

0 A. Debridement of devitalized tissue


14


6
0 B. Infusion of warmed fluids
17
18 0 C. Oral nifedipine
0 D. Rapid rewarming with dry heat
19
20
21
0 E. Rapid rewarming with warm water
22
23 0 F. Thrombolytic therapy
24
25
26 Submit
27 •
Block Time Remaining: 00:42:38

EN • 02: 04 �
@: ,j > .,,II 1•• T• l/vl l/l •
1M
1 •
=: ltem _29of32
:. ? 61 � � , � 0
Question Id: 3876 - ?Mark <J
Previous Next
[:> Tutorial Lab Values Notes Calculator Reverse Color Text Zoom

G]
4 A 28-year-old woman comes to the urgent care center at a ski resort due to frostbite. She dropped one of her

5
gloves while riding the chairlift and was unable to find it while skiing down the slope. By the time the patient
6
returned to the resort, she was experiencing numbness and paresthesias in her left hand. Her medical history is
7
8 unremarkable, and her only regular medication is an oral contraceptive. The patient does not smoke and drinks
9 alcohol only on social occasions. Temperature is 36. 7 C (98 F), blood pressure is 108/65 mm Hg, and pulse is
10 88/min. Examination shows stiff, pale skin at the fourth and fifth digits of the left hand. Vesicles and small bullae
11
are seen at the lateral aspect of the fingerti ps. The affected dig its show absent light-touch sensation and capillary
12
refill. Which of the following is the most appropriate next step in management of this patient's injury?
13
14
A. Debridement of devitalized tissue [5%]


6 X� B. Infusion of warmed fluids [6%]
17
18 C. Oral nifedipine [6%]
19
D. Rapid rewarming with dry heat [14%>]
20
21 E. Rapid rewarming with warm water [66%]
22
23 F. Thrombolytic therapy [0%]
24
25
26
Incorrect
27
.!I ,• :.1 I• u- ,-. II • :.1
Block Time Remaining: 00:42:34

EN • 02: 04 �
@: ,j > .,,II 1•• T• l/vl l/l •
1M
1 •
=: Item _29of 32
? 61 l:°f" � , � 0
Question Id: 3876 :.
- ? Mark <J
Previous Next
[:> Tutorial Lab Values Notes Calculator Reverse Color Text om Zo

G]
4 Frostbite

5
• Superficial pallor & anesthesia
6 Clinical
• Blistering, eschar formation
7 findings
• Deep tissue necrosis & mummification
8
9 • Rapid rewarming in 37-39 C (98.6 - 1 02.2 F)
10 water bath
Management
11
• Analgesia & wound care
• Thrombolysis in severe, limb-threatening cases
12
©UWorld
13
14 Frostbite is characterized by freezing of tissue, leading to disruption of cell membranes, ischemia, vascular
thrombosis, and inflammatory changes. Severity can range from superficial pallor and anesthesia to blistering,

6
eschar formation, or deep-tissue necrosis and mummification. Affected tissues typically have a stiff or waxy
17
texture. Most cases invo lve the face, ears, or distal limbs.
18
19 Initial management of frostbite is based on rapid rewarming in a 37-39 C (98.6-102.2 F) water bath. Aggressive
20
analgesia should be offered due to the potential of severe pain in the rewarming process. Hot air rewarming is not
21
recommended due to the difficulty of maintaining precise temperature control (Choice D). In the field, rewarming
22
23 can be accomplished by holding the affected limbs against the rescuer' s abdomen. However, it should not be
24 attempted if there is a possibility of refreezing before definitive care can be provided as it can worsen tissue
25 damage. After rewarming, careful wound care should be provided to reduce the risk of secondary infection.
26
27 (Choice A) Severe frostbite injuries may require mu ltiple debridement procedures, but initial assessment may •
Block Time Remaining: 00:42:34

EN • 02: 04 �
@: ,j > .,,II 1•• T• l/vl l/l •
1M
1 •
=: ltem _29of32
:. ? 61 � � , � 0
Question Id: 3876 - ?Mark <J
Previous Next
[:> Tutorial Lab Values Notes Calculator Reverse Color Text Zoom

G]
4
damage. After rewarming, careful wound care should be provided to reduce the risk of secondary infection.

5 (Choice A) Severe frostbite injuries may require multiple debridement procedures, but initial assessment may
6
overestimate the extent of tissue damage. Debridement is not indicated until the tissue is rewarmed and an
7
accurate survey of devitalized tissue can be performed.
8
9 (Choice B) Intravenous infusion of warmed fluids is indicated in the management of systemic hypothermia but is
10
not effective in frostbite injury.
11
12 (Choice C) Calcium channel blockers (eg, nifedipine, amlodipine) are useful in the management of Raynaud
13 phenomenon. Neurovascular injury in frostbite can lead to chronic Raynaud symptoms, but calcium channel
14
blockers are not indicated in the acute injury.


6 (Choice F) Thrombolytic therapy (eg, tissue plasminogen activator) can be considered for deep frostbite injury that
17 is expected to require extensive amputation, although the evidence is limited. It is not advised in limited, distal
18
frostbite.
19
20 Educational objective:
21 Initial management of frostbite is based on rapid rewarming of affected tissues in a 37-39 C (98.6-102.2 F) water
22
bath. Hot air rewarming is not recommended, and rewarming should not be attempted if there is a possibil ity of
23
24
refreezing before definitive care can be provided.
25
26 References
27

Block Time Remaining: 00:42:34

EN • 02: 04 �
@: ,j > .,,II 1•• T• l/vl l/l •
1M
1 •
=: ltem _30of32
:. ? 61 � � , � 0
Question Id: 4511 - ?Mark <J
Previous Next
[:> Tutorial Lab Values Notes Calculator Reverse Color Text Zoom

G]
4 A 34-year-old homeless man is brought to the emergency department in a confused state and complaining of flank

5
pain. His past medical history is unknown. The patient's temperature is 36.8 C (98.2 F), blood pressure is 110/70
6
mm Hg, pulse is 110/min, and respirations are 22/min. His appearance is disheveled. Pupils are equal, 4 mm in
7
8 size, and reactive to light. Fundoscopic examination is within normal limits. Lungs have vesicular breath sounds.
9 Mild costovertebral angle tenderness is present. A Foley catheter is placed, and the urine appears red, with
10 calcium oxalate crystals seen on microscopy. Laboratory results are as follows:
11
12 Sodium 136 mEq/L
13
14
Potassium 3.6 mEq/L

Chloride 93 mEq/L

6
17 Bicarbonate 6 mEq/L
18
Blood urea
19 34 mg/dL
20
nitrogen
21 Creatinine 2.8 mg/dL
22
23 Calcium 6.0 mg/dL
24
Glucose 80 mg/dL
25
26 2.3 mEq/L (0.5-2.2
27 Plasma lactate

Block Time Remaining: 00:42:31


28
29 TIMEDTUTOR

EN • 02: 04 �
@: ,j > .,,11 1•• T• l/vl l/1 •
1M
1 •
=: ltem _30of32
:. ? 61 � � , � 0
Question Id: 4511 - ?Mark <J- -
Previous
-
Next
[:>
- - . -
Tutorial Lab Values Notes Calculator Reverse Color Text Zoom

G]
4

Calcium 6.0 mg/dl
5
6 Glucose 80 mg/dl
7
2.3 mEq/L (0.5-2.2
8 Plasma lactate
9
mEq/L)
10 Serum ketones Negative
11
12 Prothrombin time 11 sec (11-15 sec)
13
14 Which of the following is the most appropriate next step in management of this patient?


6 0 A. Broad-spectrum antibiotics
17
18 0 B. Fomepizole
19
20
0 C. Methylene blue
21 0 D. N-acetylcysteine
22
23 0 E. Sodium thiosulfate
24
25
26 Submit
27 •
Block Time Remaining: 00:42:26
28
29 TIMEDTUTOR

EN • 02: 04 �
@: ,j > .,,11 1•• T• l/vl l/1 •
1M
1 •
=: ltem _30of32
:. ? 61 � � , � 0
Question Id: 4511 - ?Mark <J Previous
- --
Next
[:> Tutorial Lab Values Notes Calculator Reverse Color Text Zoom

G]
- - - -

4 Calcium 6.0 mg/dl
5
6 Glucose 80 mg/dl
7
2.3 mEq/L (0.5-2.2
8 Plasma lactate
9
mEq/L)
10 Serum ketones Negative
11
12 Prothrombin time 11 sec (11-15 sec)
13
14 Which of the following is the most appropriate next step in management of this patient?


6 A. Broad-spectrum antibiotics [6%]
17
18 B. Fomepizole [60%]
19
C. Methylene blue [10%]
20
21 D. N-acetylcysteine [9%]
22
23 X r E. Sodium thiosulfate [12%]
24
25
26
Incorrect
(1,, 60% I• I• I:
L!!!. Answered correctl
27
II • •I
Block Time Remaining: 00:42:24
28
29 TIMEDTUTOR

EN • 02: 0 5 �
@: ,j > .,,11 1•• T• l/vl l/1 •
1M
1 •
=:
ltem 30of32
_
Question Id: 4511
:.
- ? Mark <J
Previous Next
[:>
?
Tutorial
61
Lab Values

Notes

Calculator
,
Reverse Color
� 0
Text Zoom

G]
4
Toxic alcohols

5 Toxicity Clinical features Laboratory results


6
7 • High osmolar gap
• Slurred speech
8 Alcohol • Increased anion gap
• Unsteady gait
9 ketoacidosis metabolic acidosis
10 • Altered mentation
(ketosis)
11

12 • Visual blurring, central


13
• High osmolar gap
Methanol scotomata
14 • Increased anion gap
ingestion • Afferent pupillary defect
metabolic acidosis
• Altered mentation

6
17
• High osmolar gap
18 • Flank pain
Ethylene • Increased anion gap
19 • Hematuria, oliguria
20 glycol metabolic acidosis
• Cranial nerve palsies,
21 ingestion • Calcium oxalate
tetany
22 crystals in urine
23
24 lsopropyl • CNS depression • High osmolar gap
25 alcohol • Disconjugate gaze • No increased anion gap
26
ingestion • Absent ciliary reflex or metabolic acidosis
27 •
Block Time Remaining: 00:42:24
28
29 TIMEDTUTOR

EN • 02: 0 5 �
@: ,j > .,,11 1•• T• l/vl l/1 •
1M
1 •
=: ltem _30of32
:. ? 61 � � , � 0
Question Id: 4511 - ?Mark <J
Previous Next
[:> Tutorial Lab Values Notes Calculator Reverse Color Text Zoom

G]
4
This patient's clinical features are suggestive of ethylene glycol poisoning. Ethylene glycol and methanol

poisonings occur when these compounds (found in antifreeze solutions) are ingested as a substitute for ethanol.
5
6 The initial symptoms of ingestion of these poisons can mimic ethanol inebriation. However, as alcohol
7 dehydrogenase metabolizes ethylene glycol to oxalic acid and glycolic acid, these metabolites cause profound
8 clinical consequences. Glycolic acid injures the renal tubules, and oxalic acid binds calcium, causing
9
hypocalcemia and calcium oxalate crystal deposition in the kidneys. When this occurs, patients develop flank
10
11
pain, hematuria, oliguria, acute kidney injury, and anion gap metabolic acidosis. All of these symptoms are
12 seen in this patient. Administration of fomepizole (a competitive inhibitor of alcohol dehydrogenase) or ethanol
13 prevents further breakdown of ethylene glycol into its toxic metabolites and is an integral part of treatment. Sodium
14 bicarbonate may help alleviate the acidosis, and hemodialysis may be required. Earl y recognition is crucial.

There are very few conditions (eg, diabetic ketoacidosis, lactic acidosis) that cause such a profound anion gap

6
17 metabolic acidosis (with bicarbonate level <8 mEq/L). Diabetic ketoacidosis is not likely in this patient with a
18 glucose level of 80 mg/dl. Severe lactic acidosis is unlikely with a very mild ly elevated lactate level. In cyanide
19
toxicity, for example, lactate levels are usually >10 mEq/L.
20
21 (Choice A) This patient is afebrile, not hypotensive, and has only mildly elevated lactic acid. Sepsis-induced lactic
22 acidosis is unlikely and broad-spectrum antibiotics are not needed.
23
24 (Choice C) Methylene blue is the treatment for methemoglobinemia, seen in the setting of ingestion of certain
25 drugs (eg, dapsone) or anesthetic agents. Methemoglobinemia manifests as cyanosis and respiratory depression
26 as a result of impaired oxygen delivery to the tissues.
27 •
28
Block Time Remaining: 00:42:24
29 TIMEDTUTOR

EN • 02: 0 5 �
@: ,j > .,,II 1•• T• l/vl l/1 •
1M
1 •
=: ltem _30of32
:. ? 61 � � , � 0
Question Id: 4511
- . - - ?
... .
Mark <J
Previous [:>-
Next
-- -- .. Tutorial
-- Lab Values
. - -
Notes
.. Calculator Reverse Color Text Zoom

G]
4
' ,..

drugs (eg, dapsone) or anesthetic agents. Methemoglobinemia manifests as cyanosis and respiratory depression

5 as a result of impaired oxygen delivery to the tissues.


6
7
(Choice 0) N-acetylcysteine is the antidote for acetaminophen overdose. Acetaminophen intoxication presents
8 most often with nonspecifi c symptoms such as nausea, vomiting, or malaise, and can progress to liver damage
9 and failure (eg, encephalopathy, jaundice, elevated prothrombin time). Acute kidney injury and metabolic acidosis
10 can occasional ly occur but typically not to the extent seen in this patient.
11
12 (Choice E) Sodium thiosulfate is the antidote for cyanide poisoning, generally seen in the setting of fires or
13 occupational exposure (eg, mining, pesticides). Patients with cyanide poisoning have markedly elevated lactate
14
levels (typically >10 mEq/L).

Educational objective:

6
17 Ethylene glycol is associated with hypocalcemia and calcium oxalate deposition in the kidneys. This leads to flank
18 pain, hematuria, oliguria, acute kidney injury, and anion gap metabolic acidosis. Treatment involves administration
19
of fomepizole or ethanol to inhibit alcohol dehydrogenase, sodium bicarbonate to alleviate the acidosis, and
20
21 hemodialysis in cases of severe acidosis and/or end-organ damage.
22
23 References
24
25 • Ethylene glycol ingestion treated only with fomepizole.
26
Copyright© UWo�d. All rigllts rese,ved.
27 •
28
Block Time Remaining: 00:42:24
29 TIMEDTUTOR

EN • 02: 0 5 �
@: ,j > .,,II 1•• T• l/vl l/1 •
1M
1 �
=: ltem 31of32
_ :. ? 61 � � , � 0
Question Id: 11417 - ?
Mark <J
Previous Next
[:> Tutorial Lab Values Notes Calculator Reverse Color Text Zoom

G]
4 A 53-year-old man with chronic alcoholism is brought to the emergency department after being found in a park on
5
a cold day in winter. He was unresponsive when found and was intubated in the field by paramedics. On arrival,
6
his body temperature is 31 C (88 F), blood pressure is 77/48 mm Hg, and pulse is 31/min. ECG shows sinus
7
8 bradycardia and QT interval of 560 msec. Which of the following is the best next step in managing this patient's
9 hemodynamic status?
10
11 0 A. Active rewarming
12
13 0 B. Atropine
14
0 C. Levothyroxine

6 0 D. Magnesium
0 E. Transvenous pacing
17
18
19
20
21 Submit
22
23
24
25


26
27
28
Block Time Remaining: 00:42:23
TIMEDTUTOR � Feedback
©
Suspend End Block

..-.
29

02: 0 5 �
r§1 �
EN � r;-;: ,j, .,,ii 1•• T• l/vl l/1 •
1M
1 •
=: ltem 31of32
_ :. ? 61 � � , � 0
Question Id: 11417 - ?
Mark <J
Previous Next
[:> Tutorial Lab Values Notes Calculator Reverse Color Text Zoom

G]
4 A 53-year-old man with chronic alcoholism is brought to the emergency department after being found in a park on

5
a cold day in winter. He was unresponsive when found and was intubated in the field by paramedics. On arrival,
6
his body temperature is 31 C (88 F), blood pressure is 77/48 mm Hg, and pulse is 31/min. ECG shows sinus
7
8 bradycardia and QT interval of 560 msec. Which of the following is the best next step in managing this patient's
9 hemodynamic status?
10
11 �@ A. Active rewarming [76%>]
12
13
B. Atropine [12%]
14
C. Levothyroxine [0%)

D. Magnesium [6%>]

6
17

I
E. Transvenous pacing [4%]
18
19
20

F=l 09/05/2018
21
(1,, 76% ("i'\ 4 Seconds
22 Correcl L!!!. Answered correctly '-:::JTimeSpent l.!::.I Last Updated
23
24


25
Explanation
26
27
28
29
Block Time Remaining: 00:42:20
TIMEDTUTOR � Feedback
©
Suspend End Block

.... r§1
...- �
EN . r;-;: ,j, ..,11 1•• 02: 0 5 �
T• l/vl l/1 •
1M
1 •
=: Item _31of 32
? 61 l:°f" � , � 0
Question Id: 11417 -
:. ?
Mark <J
Previous Next
[:> Tutorial Lab Values Notes Calculator Reverse Color Text om Zo

G]
4 Clinical features of hypothermia

5
6 Classification Mild: 32-35 C (90-95 F)
7 • Tachycardia, tachypnea
8 • Ataxia, dysarthria, increased shivering
9 Moderate: 28-32 C (82-90 F)
10 • Bradycardia, lethargy, hypoventilation, decreased shivering,
11 atrial armythmias
12 Severe: <28 C (82 F)
13 • Coma, cardiovascular collapse, ventricular arrhythmias
14
Treatment General

6 • Warmed (42 C (107 Fl) crystalloid for hypotension
17 • Endotracheal intubation in comatose patients
18
19 Rewarming techniques
20 • Mild hypothermia: Passive external warming (remove wet
21 clothing, cover with blankets)
22 • Moderate hypothermia: Active external warming (warm
23 blankets, heating pads, warm baths)
24 • Severe hypothermia: Active internal rewarming (warmed
25 pleural or peritoneal irrigation, warmed humidified oxygen)


26
©UWorld
27 •
28
29
Block Time Remaining: 00:42:20
TIMEDTUTOR � Feedback
©
Suspend End Block

...-
••. r§1 �
EN . r;-;: ,j, .,,ii 1•• 02: 0 5 �
T• l/vl l/1 •
1M
1 •
=: ltem _31 of32
:. ? 61 � � , � 0
Question Id: 11417 - ?Mark <J Previous Next
[:> Tutorial Lab Values Notes Calculator Reverse Color Text Zoom

G]
4
This patient's unresponsiveness, temperature of 31 C (88 F), and bradycardia are consistent with moderate

hypothermia from cold exposure. Mild hypothermia (32-35 C [90-95 Fl) causes an increase in heart rate with
5
6 peripheral vasoconstriction. Moderate hypothermia (28-32 C [82-90 Fl) causes progressive bradycardia and
7 hypotension due to decreased reactivity of pacemaker cells and salt/water loss from cold-induced diuresis.
8
Primary therapy for moderate hypothermia is active external rewarming, which includes use of warmed blankets,
9
10 warm baths, and warmed intravenous fl uids.
11
(Choices B and E) Due to decreased reactivity of the pacemaker cells, the bradycardia associated with
12
hypothermia is often refractory to treatment with atropine and cardiac pacing. However, it usually wi ll improve with
13
14 treatment of the hypothermia.

(Choice C) Levothyroxine is used to treat hypothermia caused by myxedema.



6
17 (Choice D) Intravenous magnesium sulfate is typically used to treat polymorphic ventri cular tachycardia
18 associated with acquired QT prolongation (torsades de pointes).
19
20 Educational objective:
21 Primary therapy for moderate hypothermia includes active external rewarming and warmed intravenous fluids.
22 Bradycardia associated with hypothermia is often refractory to treatment with atropine and cardiac pacing, but it
23
usually improves with correction of hypothermia.
24
25
26 References
27
• •••• .. - . . . .. . - .. .
Block Time Remaining: 00:42:20
28
29 TIMEDTUTOR

EN • r;:): j > .,,11 02: 0 5 �


, 1•• T• l/vl l/1 •
1M
1 �
=: ltem 32of32
_ :. ? 61 � � , � 0
Question Id: 3856 - ?
Mark <J
Previous Nej
[::?.- Tutorial Lab Values Notes Calculator Reverse Color Text Zoom

G]
4 A 34-year-old male is brought to the emergency department by his neighbor . The neighbor found the man lying on
5
the floor beside an empty bottle of unknown substance. At the hospital, the patient is conscious and alert, but in
6
severe pain. His temperature is 36.8° C (98.2° F), blood pressure is 130/70 mm Hg, pulse is 90/min and
7
8 respirations are 20/min. Abdomen is benign, with normal bowel sounds. Pupils are 5 mm, bilaterally. His tongue
9 is white, heavy drooling of saliva is noted and he is unable to swallow . Which of the following is the most likely
10 explanation for this patient' s symptoms?
11
12 O A. Anticholinergic poisoning
13
14 0 B. Caustic poisoning
0 C. Tricyclic antidepressant poisoning

6
17 0 D. Cyanide poisoning
O E. Alcohol intoxication
18
19
20
21
22 Submit
23
24
25


26
27
28
29
Block Time Remaining: 00:42:20
TIMEDTUTOR � Feedback
©
Suspend End Block

.... r§1
...- �
EN � r;-;: ,j, .,,ii 1•• 02: 0 5 �
T• l/vl l/1 •
1M
1 •
=: ltem 32of32
_ :. ? 61 � � , � 0
Question Id: 3856 - ?
Mark <J
Previous Next
[:> Tutorial Lab Values Notes Calculator Reverse Color Text Zoom

G]
4 A 34-year-old male is brought to the emergency department by his neighbor . The neighbor found the man lying on

5
the floor beside an empty bottle of unknown substance. At the hospital, the patient is conscious and alert, but in
6
severe pain. His temperature is 36.8° C (98.2° F), blood pressure is 130/70 mm Hg, pulse is 90/min and
7
8 respirations are 20/min. Abdomen is benign, with normal bowel sounds. Pupils are 5 mm, bilaterally. His tongue
9 is white, heavy drooling of saliva is noted and he is unable to swallow . Which of the following is the most likely
10 explanation for this patient' s symptoms?
11
12 X@ A. Anticholinergic poisoning [13%]
13
14 � B. Caustic poisoning [73%]

( C. Tricyclic antidepressant poisoning [6%]



6
17 D. Cyanide poisoning [5%]
18

I
19 E. Alcohol intoxication [0%]
20
21
22
lnco�ct II\ 4 Seconds 09/16/2018
(1,, 73% I==!
23 Corre<:t answer L!!!. Answered correctly 1..::.J TimeSpent l.!!!.I Last Updated
24 B


25
26
Explanation
27
28
29
Block Time Remaining: 00:42:16
TIMEDTUTOR � Feedback
©
Suspend End Block

.... r§1
...- �
EN . r;-;: ,j, .,,ii 1•• 02: 0 5 �
T• l/vl l/1 •
1M
1 •
=: ltem 32of32
_ :. ? 61 � � , � 0
Question Id: 3856 - ?
Mark <J
Previous Next
[:> Tutorial Lab Values Notes Calculator Reverse Color Text Zoom

G]
4

This patient's presentation is most consistent with caustic ingestion. He is conscious and alert and of the answer
5
6 choices listed, caustic ingestion is least likely to affect the patient' s cognition. Caustic injury leads to damage of the
7 tissue lining the gastrointestinal tract, manifested by necrosis, edema, scarring and severe pain. The white tongue,
8 heavy salivation and dysphagia are results of this process. Severe esophageal and stomach ulceration may also
9
occur and physicians should monitor for indications of peritonitis and mediastinitis.
10
11 (Choice A) Anticholinergic toxicity causes hyperthermia, mydriasis, delirium, urinary retenti on and decreased
12 bowel sounds. Dry mouth, rather than increased salivation, is seen.
13
14 (Choice C) TCA poisoning presents with CNS depression, cardiac arrhythmias, hypotension and anticholinergic
signs such as hyperthermia, flushing, dilated pupils, decreased bowel sounds and urinary retention.

6
(Choice 0) Cyanide is highly lethal and toxicity presents with headache, vomiting, abdominal pain and flushed
17
18 skin. lnhalational exposure causes a bitter almond odor.
19
(Choice E) Alcohol intoxication usually presents with altered mental status, ataxia and slurred speech.
20
21 Educational Objective:
22 Caustic poisoning does not cause alteration in consciousness. It presents with dysphagia, severe pain, heavy
23
salivation and mouth burns. The damage is the result of necrosis of the ti ssue that lines the gastrointestinal tract.
24
25
In severe cases, perforation of the stomach or esophagus can occur, causing peritonitis or mediastinitis.


26 Copyright© UWo�d. All rigllts reserved.
27 •
28
29
Block Time Remaining: 00:42:16
TIMEDTUTOR � Feedback
©
Suspend End Block

.... r§1
...- �
EN . r;-;: ,j, .,,11 I"•
02: 0 5 �
T• l/vl l/l •

Você também pode gostar